0% found this document useful (0 votes)
410 views173 pages

(9-10-11 2022)

This document contains multiple choice questions about various medical topics including: - Mass spectroscopy identification of amino acids - Treatment for group B strep in labor - Counseling points for warfarin use - Considerations for chewable medications - Acceptable drugs in pregnancy - Causes of ergotism - Antibiotic choices for various infections - Drug choices for hypertension and heart failure - Electrolyte replacement guidelines - Fistulizing Crohn's disease treatment - Aerobic pathogens and epidemics - Consequences of long-term TPN - Pain medication options for menstruation - Duration of osteomyelitis - Geriatric dosing of sim
Copyright
© © All Rights Reserved
We take content rights seriously. If you suspect this is your content, claim it here.
Available Formats
Download as PDF, TXT or read online on Scribd
0% found this document useful (0 votes)
410 views173 pages

(9-10-11 2022)

This document contains multiple choice questions about various medical topics including: - Mass spectroscopy identification of amino acids - Treatment for group B strep in labor - Counseling points for warfarin use - Considerations for chewable medications - Acceptable drugs in pregnancy - Causes of ergotism - Antibiotic choices for various infections - Drug choices for hypertension and heart failure - Electrolyte replacement guidelines - Fistulizing Crohn's disease treatment - Aerobic pathogens and epidemics - Consequences of long-term TPN - Pain medication options for menstruation - Duration of osteomyelitis - Geriatric dosing of sim
Copyright
© © All Rights Reserved
We take content rights seriously. If you suspect this is your content, claim it here.
Available Formats
Download as PDF, TXT or read online on Scribd
You are on page 1/ 173

✔✔ ✔ ✅

✔ ✔
[First parts:]

Tyrosine residue in mass spectroscopy?


A- Leucine
B- Lysine
C- Serine
D- Phenylalanine

What to give a woman during labor who has strepto group B?


A- Ampicillin
B- Cefazoline ‫ﻣﺤﻤﺪ ﺳﻠ)ٮﻤﺎن‬.‫د‬

Counselling of warfarin?
A. Avoid green leaves
B.Take green leaves
C.Separate between green leaves and warfarin with at least 4 hours
D. Separate between green leaves and warfarin with at least 2 hour
‫ اﻷﺻﺢ‬: give fixed amount of green leaves

chewable tab?
A- can only chewable
B- chewable and swallowing
‫ڡﺎل‬3 ‫واذا‬:
Chewable aspirin : ‫ڡ>ى‬9 ‫ڡﻀﻞ‬9‫ٮ )ﻜون ﻣ‬:
Acute coronary syndrome

Warfarin used in pregnant women with (mechanical valve) :


A- not used
B- first trimester
C- second trimester
D- third trimester
‫ڡﺎل‬3‫ ﻟﻮ ﻣﺎ‬mechanical valve ‫ٮﺎر‬3‫ﺤ‬9‫ٮ‬9 A

What is the cause of ergotism?


A.Candida Albicans
B.Aspergillus
C.Claviceps purpure
Case :
Edema + bacterial infection
‫ٮﺬﻛرو‬3‫ٮﺴﻠ)ٮﻦ م ﻣ‬9‫ٮ‬P‫ڡﺎت اﻟ‬3‫ٮ‬3‫ڡﺎوﻣﻪ ﻟواﺣﺪ ﻣﻦ ﻣﺸ‬3‫زريﻊ وﻃﻠﻌﺖ ﻣ‬3‫ﻋﻤﻠﻮ ٮ‬
‫ٮﻮ‬9‫ٮريﺎ دي ﺷ‬3‫ٮﻜ‬P‫ڡﻊ اﻟ‬3‫تﻮ‬3‫ ٮ‬:
MRSA
MSSA
S.pyogens

‫ٮﺪو‬9‫ ﻣريﺾ ﻋ‬HTN ‫ٮﻮ؟‬9‫ٮﺐ اﻟﻜﺴﻞ ﺷ‬P‫ٮ>ى ﻣﻦ ﻛﺴﻞ ﺳﺎل ﻣﻦ ﺳ‬9‫ٮﻌﺎ‬P‫و‬


Hydralazine
Isosorbide
Bisoprolol

Pregnancy in first trimester with HTN, avoided drug?


A- Hydrochorothiazide
B- Ramipril
C- Labetalol
D- Amlodipine

Case with low k level and other electrolyte normal :


A- 20mmol kcl/100 NS polus
B- 90 mmol kcl/250 NS polus
C- 20 kcl/450 NS infusion for 15 minutes
D- 90 kcl/500 NS inf for 5 hours
‫ڡﺎﻟﻮ‬3 ‫ٮﻌﺾ‬P‫ اﻟ‬D ‫ٮﻬﺎ‬9‫ٮﺲ ان ﺷﺎءﷲ ا‬P ‫ڡوق‬9‫ ﺳﺎﻋﺎت و‬٤ ‫ڡﺖ ﻣﻦ‬3‫ﺣﺬ و‬9‫ﺎ‬3‫ ﻻن ٮ‬C

Crhons disease + Fistula :


A- Salfalsalzine
B- Cipro
C- Prednisolone
‫ﺣﻮد‬P‫ اذا ﻣﻮ‬infliximab ‫ٮﺎر‬3‫ﺤ‬9‫ٮ‬9 ‫ﺣﻮد‬P‫ﻫﻮ اﻻﺻﺢ واذا ﻣﻮ ﻣﻮ‬A
‫ﺣﻮد‬P‫ واذا ﻣﻮ‬Cipro+ Metro ‫ٮﺮﺿﻮ ﺻﺢ‬P

What is considered aerobic pathogen and can lead to (epidemic)?


A.Influenza virus
B.Rubella virus (endemic)
B.Rubella virus (endemic)
C.korona virus (pandemic)
D.herps virus

Pt. remain in TPN 6month will cause : (CKD – Acid-base imbalance – bone disease)

Woman come to the clinic with menstrual pain, and give


you 4 structures and asking about the most suitable one?
A.aspirin
B.ibuprofen
C.mefenamic acid
D.celecoxib

How does PEG increase half-life of preparations?


A. Decrease antigenicity
B.Increase solubility
C.Increase particle size

Patient diagnosed with HF with ejection fraction 35% what to give?


Spironolactone 12.5 m
To decrease mortality

Contraindication for Crcl 20


Spironolacton
Metolazone
Fursemide
Torsemide

-Ciprofloxacin major side effect?


A. Pale skin
B. Heartburn
C. Seizures

side effect lamotragine? Blurred vision

-Amitriptyline can cause :


A neuropathy
B/sedation
C/drowsiness )|
-Amitriptyline can treatment or side effect : neuropathy

Mother went to pharmacist her baby 1 month old


suffering from mild fever?
A. Paracetamol nasal metered dose 15 Mg/Kg
B. Ibuprofen syrup dose 12Mg/Kg
C. Naproxen
D. Digoxin

Osteomyelitis duration?
- 1-2 weeks
- 4-6 weeks
- 7 days

Simvastatin max dose in geriatric?


A. 20Mg
B. 60Mg
C. 40Mg
D. 80Mg

Dose of heparin?
A. 80-100 units/Kg
B. 200 units/Kg
C. 10-100 units/Kg
D. 80-500 units/Kg

Maximum infusion rate of vancomycin?


A. 1 MI/Min
B. 2 MI/Min
C. 3 MI/Min
D. 4 MI/Min ‫ﻣﺤﻤﺪ ﺳﻠ)ٮﻤﺎن‬.‫د‬

Maximum dose of phyntoin?


A. 300mg/D
B. 400mg/D
C. 500mg/D
D. 600mg/D

Patient receive morphine 60mg every 12hr Dr will


change to fentanyl. What will be dose?
A. 2mg/D
B. 12Mg/D
C. 1.2Mg/D
D. 5Mg/D

Normal range of HbA1c is?


A. 4-7
B. 5-5.6
C. 4-5.6
D. 4-5

Pyrogen present in containers can be destroyed by


heating the containers at ?
A. 121֯C for 30 Mins
B.210֯C for 1Hr
C.121֯C for 15 Mins
D.180֯C for 4Hr

Vancomycin the trough :


A- after 30 min 4 dose
B- after 30 min 3 dose
*trough : Before 4 dose*

Interleukin I inhibitor?
A. Digoxin
B. Anakinra
C. Methotrexate
D. Warfarin

Nicotine from tobacco is an alkaloid which is?


A.Crystalline
B.Oxygen free Solid
C.Semisolid
D.Oxygen free liqui

According to the Saudia food and drug authority


cough medications are contraindicated in children?
A. Less than one year
B. Less than two years
C. Less than three years
D. Less than six years

Patient with hot flashes and did hysterectomy


What for her
Estrogen
Estrogen + progesterone

A child is taking high dose amoxicillin (99mg/Kg) for Otitis media came to the clinic for
pneumonia and infuenza vaccine?
A. Cancel the vaccines
B. Delay the vaccines for one year
C. Proceed to give the vaccine

Skewed data which central tendency used?


Mean
Mode
Median
Standard deviations

Cause metabolic acidosis:


A- phenytoin
B-phenobarbital
C- valporic acid
D- topiramate

24-year-old pregnant female presents to the urgent care clinic with fever,
frequency, and urgency. She is diagnosed with a urinary tract infection (UTI).
Based on potential harm to the fetus, which of the following medications should be
avoided in treating her UTI?
A. Nitrofurantoin.
B. Amoxicillin.
C. Cephalexin.
D. Tobramycin.

During prescription verification, a pharmacist identified a drug interaction in a 23-year-old


woman diagnosed with urinary tract and prescribed an antibiotic. Pharmacist contacted the
change to a drug that will not harm the patient.
Which of the following antibiotics is the best option for the patient?
A) Cephalexin
B) Ciprofloxacin
C) Nitrofurantoin
D) Trimethoprim-sulfamethoxazole

Which of the following medications can cause hemolytic anemia in the newborn when used
by a pregnant woman at full term?
A. amoxicillin
B. ciprofloxacin
C. azithromycin
D. nitrofurantoin
[Full term nitrofurantion]
[1 trimester : ciprofloxacin]

58-year-old male with a history of hepatitis C, cirrhosis, and ascites


presents with spontaneous bacterial peritonitis. Which of the following antibiotics
requires close monitoring and dosing adjustment in this patient given his liver
disease?
A. Penicillin G.
B. Tobramycin.
C. Erythromycin.
D. Vancomycin

what is the recommended prophylactic therapy for pregnant women colonized with group B
streptococcus in labor with penicillin allergies at high risk for anaphylaxis?
A. Cefazoline
B. Linezolid
C. Penicillin G
D. Clindamycin.
A 28-year-old pregnant female in labor is admitted to the hospital. She tested positive for
Group B streptococcus colonization at her 36-week appointment. She has a penicillin allergy and
her culture are resistant to both clindamycin and erythromycin.
What is the recommended treatment for this patient?
A. Cefazolin
B. Penicillin G
C. Ceftriaxone
D. Vancomycin

36-Which of the following drugs is known to have significant variable


bioavailability due to its extensive hepatic metabolism upon oral administration?
A. Amoxicillin
B. Propranolol
C. Ciprofloxacin
D. Acetaminophen

64-year-old male presents with signs and symptoms of an acute gouty flare.
His doctor wishes to treat him accordingly to improve his symptoms. Which of the
following strategies would be the LEAST likely to acutely improve his gout
symptoms and pain?
A. Naproxen.
B. Colchicine.
C. Probenecid.
D. Prednison

56-year-old female is discovered to have megaloblastic anemia. Her past


medical history is significant for alcoholism. Which of the following would be the
best treatment option for this patient?
A. Oral vitamin B12.
B. Parenteral vitamin B12.
C. Oral folate.
D. Oral vitamin B12 with oral folate.

physostigmine vs pyridostigmine structure and ask about replace


pyridostigmine from the formulary
A) continue physostigmine b/c pyridostigmine can produce amphetamine
B) Replace to pyridostigmine b/c it can produce amphetamine
C) Replace b/c pyridostigmine has lower SE
D) Asa structureCell can modify proteins by adding ubiquitin

# ‫ى ان‬+‫ ﺳؤال ٮ&ﺎ)ٮ‬rivastigmin ‫ٮﻜون‬2‫ى وا)ٮﻪ ﻫ‬+‫ى اﻟﻄﺮف واٮ&ﻜون ﻣﺮﻛﺐ ٮ&ﺎ)ٮ‬+‫ﻞ واٮ&ﻀﺎف ﻟﻪ ﺣﻠ&ڡﻪ ;ٮ)ٮزيﻦ )ڡ‬2 ‫ٮﻪ ٮ&ﻌﺪٮ‬2‫ اٮ&ﻌﻤﻞ ﻟ‬fda
approved ‫ﻤﺮ‬2 ‫ى ﻣﺮﴇ اﻟﺰﻫﺎٮ‬+‫ﻟﻼﺳ&ٮ)ﺤﺪام )ڡ‬
‫ى‬+‫ٮﺪل‬2‫ﻪ ﻛﺼ‬2 ‫ﻚ اٮ‬2 ‫ا)ٮﺖ راٮ‬
A. Avoid because will produce amphetamine
B. Use it because it will produce amphetamine and be dual benifit
C. Use alternative more potent

Which is stimulatery which is inhibtory?!


-platlt activition factor
-Slicing

less sedative antipsychotic??


Queuatpin
Reisperdion

Which has ceiling effect?


Morphine
Ibuprofen
Fentanyl
Oxycodone

Decrease the amount of minimum alveolar concentration (MAC) for anesthetic drug?
Diazepam

Patient with G6PD and has UTI?


Meropenem
Moxifloxacin
levofloxacin
Azithromycin

‫ا)ﺣ&ٮ;ٮﺎر اﻟ)ٮ)ﻄﺮ ﻟﻞ‬DM1 ‫ٮﻦ؟‬2‫ ﺳ)ٮ‬5 ‫كﻞ ﺳ)ٮﺔ وﻻ كﻞ‬


Dm1 ‫ٮﻦ‬2‫ ﺳ)ٮ‬٥ ‫كﻞ‬
DM2 ‫كﻞ ﺳ)ٮﻪ‬
‫ٮ)ٮﻬﺎ )ﻋﻠﻂ‬2‫ٮﺮ )ڡﺎﻫﻤ‬2‫ٮهﺎ اﻟﺪﻛ&ٮور ;ٮﺎ)ٮﻬﺎ )ﻋﻠﻂ وﻛ_ٮ‬2‫ ;ٮﺲ ﻋﻠﻖ ﻋﻠ‬،
‫‪; ،‬ٮﺲ ﻋﻠﻖ ﻋﻠ‪2‬ٮهﺎ اﻟﺪﻛ&ٮور ;ٮﺎ)ٮﻬﺎ )ﻋﻠﻂ وﻛ_ٮ‪2‬ٮﺮ )ڡﺎﻫﻤ‪2‬ٮ)ٮﻬﺎ )ﻋﻠﻂ‬
‫‪:‬اﻟﺼﺢ‬
‫وﻋ‪2‬ٮﻮ)ٮﻪ ﺳﻠ‪2‬ٮﻤﻪ ﻣﺎﻋ)ٮﺪه ﻣﺸكﻠﻪ )ڡ‪+‬ى اﻟﺤﺎﻟﻪ ذي ﻣﺎٮ ‪2‬ﺤ&ٮﺎج ا)ﺣ&ٮ;ٮﺎر‪; ،‬ٮﻌﻤﻞ )ڡﺤﺺ ‪ under control‬ﻣريﺾ اﻟﺴﻜﺮ ﻟﻮ ﻋ)ٮﺪه اﻟﺴﻜﺮ‪-‬‬
‫‪.‬اﻟﻌ‪2‬ٮﻦ كﻞ ﺳ)ٮ&ٮ‪2‬ٮﻦ‬
‫‪.‬او ﻋ)ٮﺪه ﻣﺸكﻠﻪ ;ٮﻌ‪2‬ٮﻮ)ٮﻪ‪ ،‬ٮ ‪;2‬ٮ&ڡﻰ كﻞ ﺳ)ٮﻪ ‪ not under control‬اﻣﺎ ﻟﻮ ﰷن‪-‬‬
‫;ٮﻌﺪ )ﺣﻤﺲ ﺳ)ٮ‪2‬ٮﻦ‪; ،‬ٮﻌﺪ ﻛﺬا ‪ Type1‬اول ﻣﺎٮ&ﻌﺮف ﻋ)ٮﺪه ﺳﻜﺮ ‪ ،‬واول )ڡﺤﺺ ﻟﻤريﺾ ‪; Type2‬ٮﻌﻤﻞ اول )ڡﺤﺺ ﻟﻤريﺾ*‬
‫*اﻟ)ڡﺤﺺ ;ٮيﻜون كﻞ ﺳ)ٮ&ٮ‪2‬ٮﻦ او ﺳ)ٮﻪ‬

‫؟ ‪;cough‬ٮ&ٮ&ڡﻠﻞ ال ‪ dry cough‬اي )ٮ;ٮ&ٮﻪ ﻟﻮ ا)ﺣﺪﻫﺎ واﺣﺪ ﻋ)ٮﺪو‬


‫‪clove oil‬‬
‫‪eucalyptuse oil‬‬

‫‪ adults:‬ﺳؤال اول ﻣﺎ)ٮ;ﺤﻤ)ٮﺖ ﻟﻤريﺾ اﻟﻀ‪r‬ﻌﻂ ل‬


‫‪Aliskiren‬‬
‫‪Nifidipine‬‬
‫‪Thiazid‬‬
‫‪Bb‬‬
‫‪) :‬ڡ‪+‬ى ﺷ‪2‬ٮﻤﺎء &ڡراٮ&ﻪ‬
‫‪Nifedipne for adults‬‬
‫‪Amlodipine for elderly‬‬

‫؟ ‪ tigecycline‬و ‪) propofol‬ڡ‪+‬ى اﻟ)ﺤ‪2‬ٮﺎرات ‪ calories‬اٮ ‪2‬ﺶ اﻟﺪوا ال‪+‬ى )ٮﺤﺴﺐ ﻟﻪ ال ‪ tpn‬ﺳؤال ٮ ‪&2‬ڡول )ڡ‪+‬ى‪-‬‬
‫‪ propofol‬اﻟﺼﺢ‬

‫اٮ ‪2‬ﺶ اﻟﲇ ٮ ‪&2‬ٮ‪r‬ﻌ‪2‬ٮﺮ؟ ‪ solutions,‬اﻟ&ٮﺤﻀ‪2‬ٮرات اﻟﲇ )ٮﺴويهﺎ ﻣﻦ ٮ&ﺎ;ٮﻠﺖ ال‪+‬ى‬


‫‪Side effects‬‬
‫‪Effect of drug‬‬
‫‪Stability of drug‬‬

‫‪ Hydrolysis‬ٮ ‪&2‬ڡول اذا ٮ&ﺮﻛﺖ ﻋﻠ;ٮﻪ اﻻﺳ;ٮريﻦ ;ٮﺎﻟﺤﻤﺎم اٮ ‪2‬ﺶ راح ٮ ‪2‬ﺼ‪2‬ٮﺮ ﻟﻬﺎ ؟‪-‬‬

‫ٮ ‪2‬ﺴ&ٮ)ﺤﺪم ﻟﻼﺷ)ﺤﺎص ﻣﻦ ﻋﻤﺮ ‪-Influenza vaccine‬‬


‫‪A-All from 2 years‬‬
‫‪B-With comorbidity‬‬
‫‪C-All from 6 month‬‬

‫وﻛ‪2‬ٮﺲ ﻣريﺾ ﻋ)ٮﺪو اﻻم )ڡ‪+‬ى اﻟ;ٮﻄﻦ واﻟﻄ;ٮ‪2‬ٮﺐ )ڡﺤﺼﻮ ;ٮﻮﺿﻊ اﺻ;ٮﻌﻮ ﻋﲆ ﻣﲀن اﻻﻟﻢ اٮ ‪2‬ﺶ )ٮوع اﻟ)ڡﺤﺺ ؟‬
Percussion

‫ٮﺲ ;ٮﻌﺪ ﺷ)ﺤﺺ ﻋ)ٮﺪه‬2‫ٮﻪ ﻛ‬2‫ )ڡ‬MRSA ‫ﺴﻦ‬2 ‫ويﺎ)ﺣﺬ _فﺎ)ٮﻜﻮﻣﺎٮ‬، every 12 hours
‫ٮﺎرات‬2‫ واﻟ)ﺤ‬،٥ ‫ڡﻞ ﺣ&ڡﻪ ﺻﺎر‬Ç‫ٮ‬2‫ وﻟﻤﺎ &ڡﺎﺳوا اﻟﻠ‬:
- Stop vancomycin and start linezolid
- Increase the frequency of vancomycin to every 8 hours

‫ ;ﺣراح‬colorectal ‫ى ﻋﻦ ﻣﺼﺪر ﻟﻤﻌﻠﻮﻣﺎت ﻋﻦ ال‬+‫ٮﺪل‬2‫ٮﺮ اﻟﺼ‬2‫ﺴ&ٮﺸ‬2 ‫;ﺣﺎ ٮ‬dvt ‫اﻟﻤﻤﻜﻦ )ٮﺴ&ٮ)ﺤﺪﻣﻬﺎ ﻣﻊ اﻟﻤريﺾ‬
‫ﻣﺎ ﻣ&ٮﺬﻛﺮٮ&ﻬﻢ اﻻﺳﻤﺎء ﻃويﻠﺔ ;ٮﺲ ﻣﻀﻤﻮ)ٮﻬﻢ‬
A-Dvt with orthopaedic
B-Dvt non orthopaedic
C-Parental nutrition and interna

‫ٮريﺎ‬2‫ى ﻃ;ٮﻖ زرﻋﻮ ;ٮﻜ&ٮ‬+‫ )ڡ‬E.Coli ‫ٮﻪ‬2‫ وﺣﻄﻮ )ڡ‬Lacoste


‫ٮﺮ‬2‫ﺼ‬2 ‫ﻚ ٮ‬2 ‫ﺶ راٮ‬2 ‫اٮ‬
A- Damage
B- Nutrition

‫ ﺻﻮت ﺻﻤﺎﻣﺎت اﻟ&ڡﻠﺐ‬dup lub :


A- S1,S2 .mitral valve
B- S1, S2, S3 mitral valve

What is the function of pili in bacteria?


A. bacterial movement
B. attach a bacterial cell to specific surfaces
C. key virulence characteristic
D. all of the above ‫اذا ﻣﻮ;ﺣﻮده‬

Which enzyme inhibit bacterial cell wall ?


Lysosoma
Lyzosome

Pregent women with otitis media give?


Cipro
Levofloxacin
Moxfloxacin
Azithromyacin
7- drug can cause iron deficiency ?
A- Magnesium hydroxide ‫ٮﻤﺎن‬2‫ﻣﺤﻤﺪ ﺳﻠ‬.‫د‬
B- Levothyroxine
C- Metformin

Laxative slow onset of action?


A- psyllium
B- glycerin
C- senna
D- lactoluse
‫ٮﺮ ﻣﻮ;ﺣﻮد )ٮ)ﺤ&ٮﺎر‬2‫ ﻟﻮ )ﻋ‬A

‫ﺶ ﻣﻦ‬2 ‫ اٮ‬phase ‫ﻜون‬2 ‫ ٮ‬pharmacovigilance


Phase 1
Phase 2
Phase 3
Phase 4

‫&ڡول‬2 ‫ ٮ‬potentiate of oral hypoglycemia:


Digitalis
Green tea
Chamomile
Coenzyme Q10

‫واﺣﺪ راح اﻟﻤﺴ&ٮﺸ)ڡﻰ ﻣﺼﺎب ;ٮﺤرق وﺣﺼﻞ‬- toxicity ‫ﺶ؟‬2 ‫ﻣﻦ اٮ‬
Cyanide
Lead
Arsenic

Q38: Inherited genetic ..location..1-22 and x/y ?


A. Autosomal
B. Heterozygous allele
C. Aneuploid
D. Acrocentric
Q39: Chromosomes 13, 14, 15, 21, and 22 are?
A. Acrocentric
B. Heterozygous allele
C. Aneuploid

Straight rods shape


A) bacili
B) cocci

Bacterial flora in skin?


Streptococcus epidermis

Have cell membrane ?


Bacteria
virus

Drug used for horizontal laminar flow?


TPN

Drug mask hypoglycemia? Beta blocker

Medication used with exercise induce asthma? A- Tiotropum


8- Cromlyn
C- Cortisone
D- Salmeterol

Which of the following is not secondary cause of dyslipidmia..


A.hypothyroidism
B. hyperthyroidism
C . diabetes
D.protease inhabite

-Anesthetic with higher chances of cardiotoxicity: nitrous oxide


-Anesthetic with higher chances of hepatic toxicity : isoflurane

Main features of graves’ disease include:


A. diffuse goiter
B. thyroid eye disease
C. pretibial myxoedema
D. thyroid acropathy

Patient female with food poisoning and went to hospital with troponin high NSTMI what is the
management
A- aspirin 81 mg + ticagrelor
B- Aspirin 325 mg + ticagrelor
C- Aspirin 81 mg +Prasugrel
D- Aspirin 325 mg + prasugrel

women take levothyroxcen and She is planning to get pregnant , ask pharmacist about herb an
alternative?
-Tell her that Levothyroxen is safe during pregnancy

Which of the following perspectives incorporates all the costs and regardless of who incurs or
obtains them?
A) Patient
B) Hospital
C) Societal
D) Ministry of Health

Patient got severe head injury due to accident and went to primary and taken to secondary and
2dry refer him to tertiary. What is this?
A)Primary to secondary
B)Secondary to tertiary
C)Primary to specialist ‫ﺸﻮف اﻷﺻﺢ‬2 ‫ﻣﺤﻤﺪ ٮ‬.‫د‬
d)secondary to specialist

The pH of a buffer system can be calculated with the


(A) Noyes–Whitney equation.
(B) Henderson–Hasselbalch equation
(C) Michaelis–Menten equation.
(D) Stokes equation

Acute Heart failure Patient with shortness of breath(SOB) and peripheral edema,
which forrester classification?
A- dry and cold
B- wet and cold
C- dry and warm
D- wet and warm

Horizontal laminar flow:


A- Cifapim
B- Anti cancer
ventrical laminar flow :
Anti cancer or hazard medicine

Warfarn what is the type of bond? Conjugation

Which is volatile alkaloids?


A- Nicotine
B- Theophylline

Drug cause bradykinia and hypersalivation?


Digitalis
ACE inhibitor
Digoxin

UFH cause:
Hypokalemia
Hyperkalemia

-Isoniazid monitoring: Hepatic

-Methotrexate monitoring:
Cbc , Renal and hepatic

-Cyclosporine monitoring trough level?


Before 4th dose

Epileptic patient has skin rash after use of oxcarbazepine what is alternative therapy?
Ethosuximide

Vaccine and dental work what type of demand:


A. Postive
B. Negative

Relief pain dysmenorrhea,


Heat water
Cold water
Reduce exercise

-The gene HER2 was negative we can use:


Letrozole

-The gene HER2 was positive we can use:


Trustzomab

Vaccine give long life immunity?


A-Varicella
B- Measles
C- Pcv
D- Mcv
‫ اﻟﺼﺢ‬،‫ٮﺤﻪ‬2‫ٮﺮ ﺻﺤ‬2‫ٮﺎرات كﻠﻬﺎ )ﻋ‬2‫ ﻫ)ٮﺎ اﻟ)ﺤ‬BCG

Which of the following vaccines is usually


not given until after the child's first
birthday?
a. Influenza vaccine
b. MMR vaccine
c. DTap vaccine
d. Hib vaccine

How to measuring pain in child?


1- numerical rating scale
2- verbal rating scale
3- visual analog scales ‫ٮﻤﺎن‬2‫ﻣﺤﻤﺪ ﺳﻠ‬.‫د‬
4- face pain scale -revised

contraceptive with : breast feeding :


.A-Estrogen pill
B- .Prgesti pill
C-monophisc .
D - Diphisic

Which is teratogenic especially in third trimester :


a)Ibuprofen
b)Lithium
c)Methoxrate

Which of the following statements describes unfractionated heparin?


A. enolic acid derivative
B. propionic acid derivative
acetyl ester of salicylic acid
D. mucopolysaccharide polymers

Lipophilic drug excereted in breast milk?


Diazepam
Dihydroergotamine

Which type of anemia can be treated by chemotherapy anticancer?


A/Non lymphocytic anemia
B/Acute lymphoid anemia
C/Aplastic anemia
D/Hemolytic anemia

Each individual involved in manufacturing of sterile


preparations is required to be well trained to produce sterile preparations, which is the following
tests is required by USP to be done annually to ensure consistent
production of sterile drugs?
A.Sterility test
B.Media fill
C.Viable and non-viable environmental monitoring

primary information resources?


- GUIDELINES FROM CDC
- RCT FROM PubMed ‫ٮﻤﺎن‬2‫ﻣﺤﻤﺪ ﺳﻠ‬.‫د‬
-handbook information Review articl.
‫ٮﺎرات ﰷ)ٮﻬﺎ )ﻋﻠﻂ‬2‫اﻟ)ﺤ‬
Guideline(tertiary)/ RCT(primary),Pubmed(secondary)/Review(tertiary), Article(primary)

Which of the following is not commonly considered to be part of the public health function of
community pharmacists?
a) Health promotion
b) Compounding
c) Diabetes testing
d) 'Sign-posting'

What is the acceptable bioavailability in self life


98%
97%
90% ‫دﻛ&ٮور ;ﺣﺎو;ٮﻬﺎ‬
87% ‫ٮﺤﻪ‬2‫ا)ٮﺎ اﺷﻮف اﻟﺼﺤ‬

Otic condition treated without prescription agent?


A. Vertigo
B. Water clogged ear
C. Tinnitus
D. Temporary hear loss

Sphygmomanometer cuff placed around which part? Arm

To reach glucagon in muscle in diabetic patient he need to


Moderate exercise
Mild Exercise
Hard exercise
Relax muscles

Which of the following counseling points is most important to discus with a


patient who is taking carbidopa/levodopa?
A. Should be taken on an empty stomach
B. High-fat meal may increase the absorption
C. Should not be taken with calcium supplements
D. High-protein meal may decrease the absorption
Patient with infection, hypotension, and high HR. What to give?
a- hypertonic saline
b- 0.9 NS ‫ٮﻤﺎن‬2‫ﻣﺤﻤﺪ ﺳﻠ‬.‫د‬
c- dextran
d- ringer lactate

Treatment of metabolic alkalosis?


Ca carbonate
Saline.9
Ringer lactate. (CI)
Dopamine iv

Question about tablet punching:


A. chipping,
B. capping,
C. binding

powder stay in bottom?


Chipping
Cling
Cepping

45-year-old woman was prescribed a sedative for anxiety to be taken at night. Three days later,
she presented complaining of drowsiness, dysphoria and motor depression every morning.
Which of the following medications was most likely prescribed?
A. zolpidem diazepam
B.ramelteon
D. midazolam

9-year-old boy had nail piercing few days ago, now he has osteomyelitis, which organism?
A. E.coli
B. pseudomonas aeruginosa
C. Streptococcus pyrogens
D. Candida albicans
‫ٮﻪ‬2‫ ﻟﻮ )ڡ‬Staphylococcus ‫)ٮ)ﺤ&ٮﺎرﻫﺎ‬

important of using patches as contraceptive ?


A. To remove compliance
B. For long duration of action
C. To prevent first pass metabolism
‫&ٮﻌﻠﻖ ;ٮﺎﻟﺤراره )ٮ)ﺤ&ٮﺎره‬2 ‫ٮﺎر ٮ‬2‫ٮﻪ )ﺣ‬2‫اذا )ڡ‬

20kg take illicit drug Antidote‫؟‬


Naloxon 400 oral
Naloxone 400 im
Flumazenil oral
Flumazenil im

Child drink bleach what to do ?


Give acid
Give alkaline
Evacuations
‫ٮﺎر‬2‫ٮﻪ )ﺣ‬2‫ اذا )ڡ‬supportive care ‫&ڡﺎﻟﻮ )ٮ)ﺤ&ٮﺎره‬

What are the therapeutic outcomes for a patient with type 2 diabetes mellitus?
a) To keep blood glucose levels above 4 millimole per litre
b) To keep blood glucose between 4 and 9 millimole per litre
c) To prevent hypoglycaemia
d) To prevent hyperglycaemia

Which vitamin affects erythropoietin?


A- Iron deficiency
B- Thiamin deficiency
C- Iodine deficiency
D- B12 deficiency
*‫ٮﻦ ;ٮﺎﻟﺴؤال ٮ&ﻜون اﻻ;ﺣﺎ;ٮﻪ‬2‫ٮ&ٮﺎﻣ‬2‫ اٮ&ﻮ&ڡﻊ ﻟﻮ ﻣﺎﻃﻠﺐ اﻟ)ڡ‬A*

156-●Reaction (NAD _ NADH+) give + 30 energy?


1. exogenic -
2.endogenic -
3.Endothermic
- 4.exothermic

Face to face communication barrier??


A. No elaboration or discussion
B. Disturbs the clinic and patient flow
C. Delay response

Patient taking garlic with warfarin and the pharmacist want to know side effect?
A. Natural product review
B. Micromedex

Oral bioequivalent dose of IV furosemide 40 mg when its bioavailability is 65% ‫? ﺣﺎ;ﺣﺔ ﻛﺪة‬
20
40
60 ‫ٮﻤﺎن اﻻﺻﺢ‬2‫ﻣﺤﻤﺪ ﺳﻠ‬.‫د‬
80
*‫*ﻟﻠ&ٮﻮﺿيﺢ‬
Oral dose = iv dose ÷ bioavailability
40 ÷ 65%( 0.65)

The lup-dup sound results from?


A. S1, S2, S3, and S4 mitral, tricuspid, aortic, and pulmonary valves closure
B. S3, S4 ...
C. S1, S2, S3 mitral, tricuspid, aortic, and pulmonary valves closure
D. S1, S2, S4 murmur ...

Patient had car accident and was taken to the nearest community hospital where he had
surgery and required casting for 8 weeks, after that he needs rehabilitation and he was
??referred
.A. Tertiary to rehabilitative
B. Secondary to rehabilitative

patient came to satellite pharmacy in ER and says his is out of insulin, what you should do?
A. Tell him to buy from retail pharmacy
B. give him one vial
C. tell him you cant give him without prescription
D. tell him to go to ER to assess his case

:The degradation reaction of ASA involves


a.hydrolysis
b.racemization
c.oxidation
d.photolys

Digixin toxity
Patient with hypokalemia and hyponatremia. The nurse was able to open only one line:
a- K 15 mmol in 250 ml NS same line
b- K 15 mmol in 250 ml NS different line
c- Na 15 mmol in 250 ml NS same line

Which found in mitochondria


coenzyme q with iron.
coenzyme will with selinum.
prophylin with iron.3
prophylin with selinum.

What to give for patient take metformin?


Vit B12

Drugs can precibitatee gout :


1. Diuretic
2 Asprin
3.Niacin
4.levodopa
5.cyclosporine

Master budget?
Cash
Sale
Operation
Expensives

QT represent?
Atrioventricular
Ventricular depolarization
Plateau
Ventricular action potential
Value Chain Service Providers offer:
a) Software collaboration between partners
b) Information flows across the value chain
c) Specialist services or functions within the value chain
d) Information brokerage across a sector

Observing evidence of instability during compounding is an important step.


Which of the following is happened should have discarded the product and not
dispense it ?
A. Color intensity
B. Weight Changing
C. Size of the holder
D. Chemical potency

What is the normal immunological role of the CD8+ve T-cell


a) Helps B-lymphocytes to develop into plasma cells.
b) Kills virus infected cells.
c) Secretes antibodies
d) Rejects transplanted tissue.

9- 40 years old women with breast cancer, topoisomerase is elevated, which drug should
be used
A- Cyclophosphamide
B- Melphalan
C- Irinotican
D- Methotrexate

Drug of choice for patient has focal seizure with HLA-B 1502 positive?
A. levetiracetam
B. Carbamazepine
C. Lamotrigine
D. Phenobarbital

pharmacist showed a patient how to use injection by pictures ?


A. Verbal
B. Non verbal
C. Physical
D. Visual

Ampicilin weak acid ..what's fraction will be in lipid form when ampicilin in dudenum (PH 4)??
1%
99%
50%
80%
7-mediators in the mucosa ?
IgG
-IgE
-T-cell
-Macophages
‫ اذا ﻣﻮ;ﺣﻮده‬IgA ‫)ٮ)ﺤ&ٮﺎرﻫﺎ ﻻ)ٮﻬﺎ اﻷﺻﺢ‬

Decrease or increase the dose by *technician* is:


A) Omission error
B) Prescription error
C) Dispensing error
D) Personal error

- A nurse ordered epinephrine from the pharmacy, 15 minutes later she called the
pharmacy and no one answered. Then she took a vial form the crash cart and prepared it
for the patient herself and administered safely to the patient. What did this nurse do
wrong ?
A. prepare the medication herself
B. Used the crash cart for non-emergency cases
C. There was a high chance of error

Anticoagulant take with meal ?


Rivaroxaban
Argatroban

Pertussis management for unvaccinated 1year baby (profen - azithromycin -


amoxicillin - dexamethasone)

Patient came with INR 11 and minor bleeding what to do


A/ oral Vit K ‫اﻻﺻﺢ‬
A/ oral Vit K ‫اﻻﺻﺢ‬
B/ IM Vit K
C/ Prothrombin complex concentrate
D/ Recombinant activated factor VII
*‫*ﻟﻠ&ٮﻮﺿيﺢ‬:
‫ ال‬minor bleeding ‫ ﻟﻮ‬INR ‫ٮﻪ‬2‫ )ٮﻌﻄ‬8 ‫ٮ&ﺤﺖ‬: oral vit k
‫ أﻣﺎ ﻟﻮ‬INR ‫ٮﻪ‬2‫ )ٮﻌﻄ‬8 ‫)ڡوق‬: iv vit k (‫ٮﺮ ﻣﻮ;ﺣﻮده‬2‫)وﻟﻜﻦ )ﻋ‬.
*‫ ال‬im vit K (‫ ;ٮﺤﺎﻟﺔ ال‬neonate) ‫ واذا ﻣﻌﻪ‬coagulopathy ‫ٮﻪ‬2‫ )ٮﻌﻄ‬: iv

Patient has mild pain of ortheoarthrits


Indomethacin
Ibuprofen
Paracetamol
Diclfenac

A patient was administered a neuromuscular blocker (NMB) prior to a surgical procedure to


produce skeletal muscle paralysis. This NMB drug affected small, rapidly contracting muscles of
the face and eyes first and diaphragm muscles last. The effect of this drug was easily reversed
with neostigmine. Which of the following neuromuscular blockers was most likely administered to
this patient?
A. Rocuronium.
B. Succinylcholine.
C. Diazepam.
D. Tubocurarine.

Which of the following could lead to decrease in international normalization ratio (INR) in a
patient of warfarin therapy?
A. decrease the consumption of dietary vitamin K
B. carbamazepine therapy
C. cotrimoxazole therapy
D. metronidazole therapy

Glucosamine for uses ??


A- Osteoporosis
B- Osteomyelitis
C- Osteoarthritis
-Structures of zidovudine antivirals, marked 4 area on structure and asked about
which part to remove that make the antiviral drug not effective ?
A. N3
B. CH3
C. C=O
D. O
*[antiviral effecte. OH]
*[termination N3]

Lithium and Gentamicin can be measured and curved


down after?
A. 8 Hours
B. 7 Hours
C. 5 Hours

lithium dose should be withdrawn after dose by ?


6 hr – 8 hr – 12 hr

Which of the following is the mode of action of Phentolamine?


A. beta-adrenergic blocker
B. beta-adrenergic agonist
C. alpha-adrenergic agonist
D. alpha-adrenergic bloc

- Drug acid Where will be high execrations?


A. Pka 3.3 weak base
B. Pk 3.3 weak acid
C. Pka 6.8 weak base
D. Pka 6.8 weak acid

Progressive of disease?
1- prevalence
2- incidence

What the pulmonary HTN??


A. 10-20
B. 30-50
C. 25-30
D. 5-10

Which of the following proteins is a protease of the blood clotting pathway


a) Factor XII b) Prothrombin c) Fibrin
d) Plasminogen
‫ و;ٮﺮﺿﻮ ﻫ)ٮﺎ اﻻ)ﻋﻠﺐ اٮ&)ڡ&ڡﻮ اﻻ;ﺣﺎ;ٮﻪ‬A

Pt with ischemic stroke when discharge from hospital must refer for which unit
(cardio – psychiatric – occupational - neuroradiologist)

206-Head of pharmacy (decide) to make group to arrange formulary drugs in hospital?


A. organizing
B. planning
C. Leading
‫ٮهﺎ ﻣﺎ;ٮيﻦ‬2‫ ا)ﺣ&ٮﻠ)ڡﻮ )ڡ‬A ‫ و‬b

Child one years ago, had immunodeficiency, he took before 4-month IV Ig, and now it’s time to
take MMR Vaccine?
A- Give vaccine
B- don’t give because he took IVIG
C- don’t give because he has immunodeficiency

99)Patient come with allergic signs after made serological test it was show high titre of IgM and
IgG
both what does it mean??
A. Past infection with same MO
B. New infection.
C. Past vaccine and now get real infection

-512) What to test prior using of methotrexate?? Liver


-513) What to administer prior using of methotrexate?? Folic acid

NSAIDs has the longest duration of action?


Piroxicam
Ibuprofen
Naproxen
Diclofenac

Milk pasteurization:
At 63 c temp for 30 min

A 52-year-old woman is known to have alcoholic liver disease.


Which of the following changes to pharmacokinetics and pharmacodynamics may be important
to consider when prescribing for her?
C. Reduced bioavailability of lipid soluble drugs
D. Reduced hepatic enzyme action

HAM need to ECG monitor ?


A)Daigoxin tab
B)Insulin
C)Kcl infusion ‫اﺻﺢ‬
D)Norm saline

which more OTC poisoning


Infant
Child
Adult
Elderly

Patient went for hiking after that with 48hrs he started to get rash fever what type of
immune reaponse"?
Type 1
Type 2
Type 3
Type 4

27) how is the nomenclature of these plants obtained ( Atropine belladonna and piper negra)?
-Ac, color
-Geography, color
-Discover,Ac
*Ac: active constituent* ‫ﻟﻠ&ٮﻮﺿيﺢ‬
‫ٮ;ٮﺮ ﻋﺸﺎن اﻟﻠون‬2‫اﻻٮ&رو;ٮيﻦ ﻋﺸﺎن اﻻﻛ&ٮﻒ و اﻟ;ٮ‬
Which Amino acid we give for anorexia?
A. Alanine
B. arginine
C. valine
‫ٮﻪ‬2‫ ﻟﻮ )ڡ‬leucine ‫ى اﻷﺻﺢ‬+‫ٮ&)ﺤ&ٮﺎروﻫﺎ ه‬

Nurse received extra vial in the bag what should do?


A. Keep untill next dose
B. Return to pharmac

Dose related Ses of 200mM/L phynetoin?


A. Coma
B. Nystagmus
C. Ataxia
D. Decrease Mental status

What is the cell that produce monoclonal antibodies?


A.T cells
B.B cells
C.Meyloma
D.Hybridoma

What should be monitored at base line of TPN?


A.Glucose in blood
B.Sodium in serum
C.Triglycerides in serum
D.Potassium in seru

Diabetic patient on metformin, pioglitazone and sitaglipting.) Screening shows positive hepatitis
C and elevated liver enzymes. What is the most appropriate action?
A-Do not change
B- Stop pioglitazone
TZD antidiabetics class are hepatotoxic
stop pieglitazon
C- Stop sitag ptin
D- stop pioglitazone and sitagliptin
Which of the following is a function of alcoholdehydrogenase
enzyme?
• Accelerates the reduction of ethanol
• Accelerates the addition of hydrogen to ethanol
• Accelerates the addition of phosphate to ethanol
•Accelerates the removal of hydrogen from ethanol

183- Long term plan use all aspects in organization


A- strategy
B- operation
C- organization

184- pharmacist wrote email to another pharmacist to ask add medication to the formulary in the
hospital this is consider as ?
A. Inter institutional
B. Intra institutional
C. Written communication
D. Non Verbal communication

pharmacist manager screemed on one of the pharmacist who dispensed wrong


medication in front of all the staff what is the good response ?
A. he should punish him in separate area
B. ask for a meeting with the staff
C. cut from his salary
D. try to find the cause of the error and set solutions

Which of the following is the function of the pharmacy and therapeutic committee in a hospital?
A. Responsible for poison management
B. Monitor the adverse reaction reporting
C. Monitor the medication error reporting
D. Responsible for drug formulary management

A 65-year-old man presents to the pharmacy for a refill his medications. On review of his refill
history, the pharmacist noticed that he does not refill his medications regularly. Upon asking him,
the patient started that he has multiple medications that require complex administration.
Which of the following non adherence factors affecting above patient?
A. Patient related factors
B. Provider related factors
C. Health care system factors
D. Medication and condition factor

It is the broad range of health care services provided by in order to optimize therapeutic
outcomes for individual patients
Which of the following is the term used to describe the above- mentioned definition?
A. Pharmaceutical care
B. Medication counseling
C. Medication reconciliation
D. Medication therapy management

drug is used to treat an anaphylactic reaction?


A. Atropine
B. Adrenaline
C. Isoprinosine
D. Phentolamine

Which of the following is a property of amphetamine?


A. It does not cross the blood brain barrier
B. It stimulates the release of stored acetylcholine
C. It stimulates the release of stored norepinephrine
D. It stimulates the release of five hydroxytryptophan (5-

Endotoxin part of gram ?


negative
positive

which is cytokine ?
A. Leukotriene
B. Adrenaline
C. Gonadotropin

55-year-old man presents with an excessive bronchial and oconasal secretions. History
revealed, is exposed to an organophosphate insecticide. Which of the following is the best
management?
A. Atropine IV
B. Diphenhydramine orally
C. Pralidoxime intramuscularly
D. Physostigmine intramuscu

First line treatment COPD :


A/Inhaled corticosteroid
B/Bronchodilators

Women suffer from acute asthma attack what is the best choice for her attack ?
A.ipratropium
B.tiotropium
C.corticosteroids

Status asthmaticus?
1.ipratropium
2. Corticosteroids

Cast about status asthmaticus?


SABA
LABA
Ics
Methylprednisolone

Which of the following Clostridium spp. Is linked to the develop pseudomembranous


colitis?
A) C. tetani
B) C. difficile
C) C. botulinum
D) C. Perfringens

Naproxen adverse affects?


1- cirrhosis
2- renal failure
3- hypotension
4- irritable bowel syndrome

16-month-old baby with bronchopulmonary dysplasia is admitted Pediatric Intensive Care


Unit with respiratory distress requiring in (see lab results)
HEART RATE 86/min
RESPIRATORY RATE 30/min
TEMP. 38.6°C
Nasal swab Positive for respiratory syncytial virus,What is the best management?
A) Cefuroxime
B) Palivizumab
C) Corticosteroid
D) Intravenous fluids and supportive care

An 18-year-old healthy girl admitted to the hospital with a diagnosis of necrotizing


pneumonia. She received vancomycin 1 gm IV q12h and meropenem 1 gm IV q8h. The trough
level of vancomycin was 16 mcg/ml (normal range: 10- 20 mcg/ml).
Which of the following is the next appropriate step in management of this patient?
A. Continue the same treatment
B. Request a drug level for meropenem
C. Increase the dose of vancomycin to 2 gm IV q12h
D. Decrease the dose of vancomycin to 500 mg IV q12h

Which of the following patients are most at risk of suffering from an adverse drug reaction?
a) An 8 month year old infant receiving a prescription for an antibiotic.
b) A 22 year old patient with asthma receiving prescriptions for inhalers to relieve and prevent
their asthma.
c) A 48 year old patient who has hypertension and receives a prescription for an ACE inhibitor.
d) A 68 year old patient who has oedema receiving a prescription for a diuretic.

lactating mother with asthma which can affect milk production ?


A-steroids
B-LABA
C-LABA
D-SABA

Patient with arthritis wants to start on


methotrexate he is 65 years old. When he should take the vaccination??
A. Immediately.
B. Wait until finished from methotrexate course oftherapy.
C. Give the vaccine after 2 month start methotrexate.
A 2 years old child came to clinic for taking hepatitis A vaccine, we know
that he took pneumonia vaccine from month ago, so we should:
a) Give him the vaccine immediately
b) Wait for 3 months
c) Wait for 6 months
d) Wait for 1 year

patient has hypertension and Benign prostatic hyperplasia


take Lisinopril, prazosin another antihypertensive drug but still
uncontroll He is have side effect Orthostatic hypotension
A-Add finstride
B- Stop prazosin
C- Discontinue prazosin and change to Alfuzosin

Old patient has hypertension and multiple medical condition, have recently BPH ،he took
lisinopril and oxybutynin , what should the doctor do:
Start prazosin
start alfluzosin
continue oxybutynin
discontinue oxybutynin

patient with angina and MI who takes ACEI and many other medicines. What
should we give for high blood pressure?
A. spironolactone
B. carvedilol
C. diltiazem
D. ARBs

If patient on warfarin therapy decide to be on vegun diet what will be the effect on INR ?
Increase
Decrease
Not effected
Not related

-Ibuprofen interaction with warfarin?


A)Ibuprofen increase metabolism of warfarin
B)Ibuprofen decrease metabolism of warfarin
C)Ibuprofen displace warfarin from protein binding

What anticoagulant needs bridging by Iv route before oral dose?


A. warfarin
B. Rivaroxaban
C. apixaban
D. dabigatran

First line anticoagulant for metral replacement ?


Warfarin
Colpidogerl
Dapigatran
Enoxaparin

Interaction between warfarin and TCA as amitriptyline?


1. increase bleeding risk
2. increase adverse effects of warfarin
3. decrease warfarin metabolism
4.increase please level

which increase the risk of kidney failure ?


a) age
b) diabetes
c) hypertension
d) radioactive iodine

6- which anticoagulants used after food


A/warfarin
B/apixaban
C/ rivaroxaban
D/ dabigatran

1-Patient was on olanzapine and not well controlled random glucose test was high, what is the
best alternative ?
A) aripiprazole
B) clozapine
C) donepezil
D) haloperidol (1 generation)

2- patient diagnosed with Provoked DVT his weight 70 and hight 170 and age 40 and CrCL 40,
What is the treatment for him?
A) warfrin
B) Apixaban
C) enoxaparin 40mg OD
D) Enoxaparin 70mg BID

The recommended DVT prophylaxis dose for postpartum in women with heigh risk of DVT is?
A. 3 weeks
B. 6 weeks ‫ٮهﺎ‬2‫ٮﻤﺎن واﻟﻬواري اٮ&)ڡ&ڡﻮ ﻋﻠ‬2‫ﻣﺤﻤﺪ ﺳﻠ‬.‫د‬
C. 3 months
D. 6 months
‫ﻟﻠ&ٮﻮﺿيﺢ‬:
*Postpartum:6 week*
*Unprovoked DVT: 6 month*
*Provoked DVT: 3 month*

2 patients taking warfarin with different metabolism the reason may be due to?
A/Pharmacodynamics
B/phramcogenomics

Warfarin with levofloxacin


A/INR dec
B/ INR inc

Affect off Q10 with warfarin Therapy


A/ increase
B/ decrease

Pt use Warfarin 10mg and INR is low why?


A. Low dose
B. Diet
C. Low bioavailability of war
Who can open pharmacy in saudia ‫؟‬
A- Only Saudi citizen
B- Can be non Saudi but register pharmacists should be there to manage shifts
C- Can be non Saudi but manager should be Saudi citizen
D- only pharmacist can open

Expiry of compounded syrup contain water is ?


A-7 days
B-9 days
C-14 days

-Before dispensing of TPN which drug should be discontinued ?


A- Propofol
B- dexmetomidine
C- glycylcycline (tetracycline)
C- Fentanyl

Which the carrier in krebs cycle?


Oxaloacetat
Citrate

- Which one of the following question's types is used to identify a certain piece of information?
A)Leading questions
B)Open ended questions
C)Closed ended questions
D)Process and outcome questions

Which of the following medications can cause pneumonia as a side effect? A) Lisinopril B)
Metformin C) Simvastatin D) Omeprazole

Case patient has hypertension and diabetes and lab results of randomly glucose and HbA1c all
were high , which anti hypertension should take ?
- amlodipine
- Candesartan
-  Bisoprolol+ irbesartan
- Hydrochlorothiazide+ lisinopril
! antidot of opiods
1.naloxon orall
2.naloxon s.c
3. naloxon 400mg iv
4. Naloxon 400(mcg)im

The pKa of acetyl salicylic acid (aspirin) is 3.5. The pH of gastric juice in human stomach is
about 2−3 and the pH in the small intestine is about 8. Aspirin will be:
A-unionised in the small intestine and in the stomach
B-completely ionised in the small intestine and in the stomach
C-ionised in the stomach and almost unionised in the small intestine
D-ionised in the small intestine and almost unionised in the stomach

1) The Drug Enforcement Administration (DEA) regulations require pharmacies to keep


(controlled)substances records, including prescriptions for at least ?
a) two years
b) three years
c) four years
d) five years
[Controlled 3year / Narcotics 5year]

Drug of choice adrenal insufficiency?


Hydrocortisone iv
Cortisone oral

Preventive and curative institution is ?


A. primary
B. secondary
C. tertiary
D. specialty

Which of the following could be stored in a freezing unit ?


A. Insulin
B. Interferon
C. varicella vaccine
D. Human growth hormone
Should drugs be freezing?
A. Interferon
B. Vaccine
C. solution

Phenytoin metabolism reaction is ?


A- Oxidation
B- decarboxylation
C- 4-hydroxyphenyl
D- methylation

case about a diabetic patient type 2 he takes metformin but his blood sugar is so high and
uncontrolled and hb1ac was very high .. what to take?
A- insulin glargine + insulin asprt
B- insulin glargine + glyburide

case E coli with CKD?


A. Ciprofloxacin 400 iv every 12 hrs
B. Ciprofloxacin 400 iv every 24 hrs
C. meropenem 500 iv every 8 hrs
D. meropenem 500 iv every 12 hrs

pt DM2 symptoms ..whent to lose weight..HTN ,heart failure ..do life style change and excirses
drug choice for DM 2 ?
A- Insulin
B- liraglutide ‫اﻻﺻﺢ‬
C- metformin (first line)
D- Glypuride

Which of the following is the result of drug interactions between birth control pill and
erythromycin?
A. The effectiveness of erythromycin may increase
B. The effectiveness of erythromycin may decrease
C. The effectiveness of the birth control pill may increase
D. The effectiveness of the birth control pill may decrease
patient with major depression should receive antidepressant therapy for at least?
A- 2 weeks
B- 6 weeks
C- 2 months
D- 6 months

case about patient with runny nose and complain about and need fast relief of congestion?
A. pheniramine runny nose ‫)ٮ)ڡﻊ ﻣﻊ‬2 ‫ٮ‬
‫)ٮ)ڡﻊ ﻣﻊ ال‬2 ‫ ;ٮﺲ ﻣﺎٮ‬congestion

B. pseudoephedrine $ congestion ‫)ٮ)ڡﻊ ﻣﻊ‬2 ‫ٮ‬


‫)ٮ)ڡﻊ ﻣﻊ ال‬2 ‫ ;ٮﺲ ﻣﺎٮ‬runny nose

C. intranasal cromolyn ‫ٮﻪ‬2‫ٮﻦ اﻻ اذا ﰷن اﻟﺴ;ٮﺐ ﺣﺴﺎﺳ‬2‫)ٮ)ڡﻊ ﻣﻊ اﻻٮ_)ٮ‬2 ‫ﻣﺎٮ‬

D. corticosteroid ‫ٮﻪ‬2‫ٮﻦ اذا ﰷن اﻟﺴ;ٮﺐ ﺣﺴﺎﺳ‬2‫)ٮ)ڡﻊ ﻣﻊ اﻻٮ_)ٮ‬2 ‫ٮ‬

What is the secondary prevention?


A. Vaccination
B. Immunization
C. Do exercise
D. Blood test

Influenza virus ?
A-Sporadic
B-Pandemic
C-Epidemic
-[Covid-19] : pandemic

What's (Z) in statistics ?


A- dependent
B- independent
C- sample size
D-Standard score or standard deviations

To which of the following the major histocompatibility complex unique?


A) Each cell
B) Each organ
C) Each species
D) Each individual

Patient has many comorbidities, liver failure,


renal impairment, gout, and has partial seizure, what do u recommend antiepelitic that works for
his condition??
A. Valproic.
B. Phenytoin.
C. Gabapentin.
D. Ethosuximid.

dizziness when he stands, the responsible drug is?


A/Hydralazine
B/isosorbide dinitrate
C/ prazocin
‫ اذا ﻣﻮ ﻣﻮ;ﺣﻮد )ٮ)ﺤ&ٮﺎر‬A

What is the early sign of breast cancer


- lump
- nipple discharge
- Nipple retracted
- Dimpling

Dr prescribed for pt with open-angle glucoma eye drops one drop/day and told him to
administer?
A. At night
B. At morning

Patient with heart failure NYHA class IV and difficulty breathing, comprehensive history was
given and medication list was
given with doses of lisino and metoprolol and furosemide 10 mg (all were low doses)
A. Patient has suboptimal doses of heart failure medication
Which of the following is evidence-based treatment of aspirin?
B. secondary prevention of patient with MI
C. primary prevention of stroke to a 33-year-old make
D. other choices were primary prevention with specific patient age and disease mentioned
which the fallowing uses in prophylaxis strock
A. anticoagulant
B. thrombolytic drug
C. antiplatele

‫ٮﺲ )ﻋري;ٮﺔ ﺷوي ٮ&&ڡول ﻣريﺾ‬2‫ ;ﺣﺎ;ٮوا ﻛ‬antiarrhythmia ‫&ڡول‬2 ‫ٮﺮ ٮ‬2‫ى اﻻ)ﺣ‬+‫ﻦ )ڡ‬2 ‫ٮﺮ ;ٮﻌﺪٮ‬2‫ وﺳواﻟﻒ ﻛ_ٮ‬how to prevent
stroke ‫؟‬
‫ٮﺎرات‬2‫ اﻟ)ﺤ‬:
Anti platelet
Anti arrhythmia
Anti coagulant

A lactating mother comes to emergency department complaining of nausea and vomiting. Which
of the following agents is most appropriate to give?
A. Cyclizine
B. Domperidone (increase prolactin level)
C. Dimenhydrinate
D. Levomepromazine
*‫ ﻟﻮ‬pregnant : A + B6*

Patient is with difficulty in urine, he was previously with urine incontinence, he is feeling
dizziness and hypertensive, his bp is 140/90. He is taking Lisinopril, HCTZ and Oxybutynin, which
is possible intervention ?
A. DC Oxybutynin
B. DC lisinopril and Oxy
C. DC HCTZ and Oxy
D. Continue all

324- A non-smoker patient suffers from wheezing and chest tightness, what medication
he should use ?
A. B2 agonist
B. corticosteroids
C. omalizumab

Memantine mechanism of action in alzeheimer ?


A. affinity uncompetitive N-methyl-D-aspartate (NMDA) receptor
B. Memantine also blocks the 5-hydroxytryptamine-3 receptor (at
a potency similar to the NMDA receptor
C. And thus preventing loss of neurons

A 6-year-old boy is brought with complaint of uncontrollable spasm-like movements and


excessive eye blinking for four months. Chronic motor disorder was diagnosed.Which of the
following is the best management?
A. pregabalin
B. Haloperidol ‫اﻻﺻﺢ‬
C. Methadone
D. Chlorzoxazone

Which drug should be used for both bone and GIT problems?
A- Ca gluconate
B- Ca carbonate
C- Metoclopramide
D- Omeprazol

tetracycline ‫ى‬+‫ٮﺰم اٮ&ﻮ&ڡﻊ وي&ڡول ﻫﻞ )ڡ‬2‫ٮ&ٮﺎ;ٮﻮﻟ‬2‫ﻟﻤﺎ ﺣﺼﻞ ﻟﻪ ﻣ‬


Acidic medium
Neutral medium
Basic medium

‫ﺐ دوا‬2 ‫ ;ﺣﺎٮ‬amytriptyllin ‫ ﻣﻊ‬entiric coated aspirin ‫ٮﺮﻫﻢ ﻋﲆ ;ٮﻌﺾ‬2_‫ٮﻒ ٮ&ﺄٮ‬2‫ويﺴﺄل ﻛ‬


A- Slow absorption of aspirin
B- Slow absorption of amytriptylin
C- Increase absorption of amtryptillin

Sterile preparation is a very complex manufacturing procedure that


requires -23 attention to the ingredients added in it. Which of the following
ingredients canbe added when preparing a sterile drug?
A. Antioxidants
B. 2% Benzyl alcohol
C. Bactericidal agents
D. 1.9% Sodium Chloride

Which is responsible for coloring the plant ?


A. Tannin
B. flavonoid
C. alkaloids
D. coumarins

Role of tetrahydrofolate ?
Acyl carrier
Nitrogen carrier
Electron carrier
Carbon carrier

Genomes of bacteria ?
A-DNA
B-Rna
C-DNA OR RNA
D-DNA AND RN
-Genome of viral
A-Dna
B-Rna
C-DNA OR RNA
D- DNA AND RN
-HIV ?
A-Dna
B-RNA / Retro virus
C-DNA OR RNA
D- DNA AND RNA

To decrease side effect of metformin on GI?


Divide dose
Take it with PPI

A 28-year-old woman presents with tachycardia and palpitations (ventricular ectopic beats).
History revealed essential hypertension and asthma. Patient wants to conceive (mean become
pregnant)
BLOOD PRESSURE: 147/98mmHg
What is the most likely alternative to the β-blocker in terms of efficacy on pressure, heart rate
and relative safety?
A. Prazosin
B. Enalapril
C. Diltiazem
D. Methyldopa

Drugs cause Oligohydrosis ?


Phenytoin
Topiramate
Zonisanamide
Oxcarbazepine

The action of Digoxin reduced by:


Antacid ‫اﻷﺻﺢ‬
Hyopkalemia
Hyperthyroidism
Verapamil
Hypothyroidism

Maximum dose of ergotamine for day?


3
10
6
7

Captopril contraindication?
In bilateral stenosis
History of angioedema
Pregnant
‫ٮﺎر‬2‫ى ;ٮ)ٮ)ڡﺲ اﻟ)ﺤ‬+‫ٮﺤﻪ ﻣﺎٮ&;ح‬2‫كﻠﻬﺎ ﺻﺤ‬

-carrier in TH ( T helper cell contain) ?


A- cd4-
B- cd8-
-T cytotoxic : cd8

Glargein insulin take?


1- morning
2- evening
3- after meal
4- before meal
*[Long is take night ]
[Rapid is take befor or after food ]
[Short is take befor meal 30 min]

What type of drug leading nightmare in sleep?


-Beta blocker-antihistamine 1gen
-diphenhydramine

⁃ Azathioprine significantly interact with?allopurinol


erythromycin

Saudi MOH allow advertisement of drug in


- magazine
- Social media
- Medical conference
- Private hospitals and clinics

-PUFA need important vitamin?


Vit E
-Vitamin A in ?
A-Fish oil
B-cod liver oil

Side effect of Refeeding syndrome?


Hypophosphatemia
Hypocalcemia

Which of the following statements about digitalisassociated emesis is true?


A.Itis due to Naþ/Kþ-ATPase inhibition within the stomach.
b. Itis due to stimulation of chemoreceptortrigger zones.
c.It is only seen with oral administration.
d.It is only seen with rapid IV administration

Eye preparation sterile ?


hypotonic
hypertonic
isotonic

Essential fatty acid most source from ?


plant or milk or water

Validity of pharmacy permit is?‫ٮﺺ‬2‫ٮﺔ ٮ&ﺮ)ﺣ‬2‫ﺻﻼﺣ‬


-5 years 5 ‫ اﺻ;ٮﺤﺖ اﻻن‬3 ‫ﰷ)ٮﺖ‬

Anticancer activity of Doxorubicin?


Intercalating ‫ﻃﻠﺐ ﻋﻤﻠﻪ‬
Alkylating
Antimetabolite
Antibiotics < ‫ٮﻒ‬2‫ﻟﻮ ﻃﻠﺐ اﻟ&ٮﺼ)ٮ‬

non aqueous preparation?


A. Syrup, colloidal, mucilage
B. Essence, colloid and elixir

-Advantage of pseudoephedrine over ephedrine?


Pseudoephedrine having considerably less effective ‫ٮﺮ‬2‫ٮﻪ ;ٮﻜ_ٮ‬2‫ا&ڡﻞ )ڡﻌﺎﻟ‬

-Ephedrine to psuephdrin??.
To use it in nasal congestive

9) Which of the following is an indication for methotrexate:


A/worts
B/times infection
C/psoriasis
D/acne

8) Which of the following as an indication for clomiphene citrate ?


A.Dysmenorrhoea
B.depression
C.infertility
D.Nausea
D.Nausea

4) Which of the following statement about digitalis is true?


a.Increase heart rate
b.Decrease myocardial contractility
c.Decrease preferences resistance
d.Increased cardiac output

5) Which of the following is correct explanation of tachycardia that occurs after nitroglycerin
ingestion?
A/Reflex sympathetic discharge
B/Decreased intracranial pressure
C/Direct positive chronotropic
D/Increase norepinephrine secretion from intracranial nerve ending

6) Which of the following medications is not effective for the treatment of P.aeruginosa UTI?
a.TMP/SMX
b.Ciprofloxacin
c.Methenamine
d.Norfloxacine

7) What is the rationale for the preferred use of inhaled corticosteroid over oral corticosteroids
and treatment of asthma ?
A.Increase efficacy
B.decrease systemic side-effect
C.increased the ease of use

Complication of liver diseases and cirhosis?


A-Portal hepertension
B-Hepatic encephalopathy
C-Ascites

Gelatin test came positive with?


A-Protein
B-Meconioc acid
C-Carbohydrate

You received a prescription on a plain paper what to do ?


A.Tell the manager
B.Talk to the prescriber and don't dispense

Potent glucocortisone drug ?


Hydrocortisone
Methyprednisolone
Betamethasone

The cause of lower adherence in diabetic patient?


Medication cost
Acid formulation
Alkaline formulation

Which of the following ranges of volume of distribution is deduced for warfarin


(anticoagulant) that is 98% bound to proteins?
A. 3-5 L
B. 40-50 l
C. 50-500 l
D. 5000-10000 l

Crcl ‫ٮهﺎ‬2‫ٮﺲ )ڡ‬2‫ﺶ ا&ڡ‬2 ‫اٮ‬


1- glumuler filtration
2- renal secretion

Used in renal function


1- crcl
2- BUN

-drug has a volume of distribution of 35L in a 70 Kg man.Which of the following best describes
Its distribution?
A. it is bound to DNA
B. it is dissolved in lipids
C. it has low bloavailability
D. it is mostly distributed in plasm

-Thoracotomy indications?
Iodine solution
Or Propylthiouracil
‫ى ﻣﺎذﻛﺮه‬+& ‫اﻟ;ٮﺎڡ‬
‫ى ﻣﺎذﻛﺮه‬+& ‫اﻟ;ٮﺎڡ‬

-Red man syndrome


Vancomycin
Streptomycin

-diphtheriae is caused by what bacteria? Cornebacterium diphtyeria

-Drug that will cause toxicity for the Baby in Breastfeeding mother ? depend on MCQ

myalgia induced by ?
Statins

-Name of drug that if it was combined with statin will result in sever rhabdomyolysis ?
•Gemfibrozil

-Vitamin given with iron to increase non-heme iron absorption ?


•vit C

-Crohn's disease management :


•infliximab

-Class I bio pharmaceutical classification


• high solubility high permeability

-phenobarbitals MOA? acts on GABAA receptors increasing synaptic inhibition

-methotrexate binding ?
Irreversibly
competitively
non-competitive

-Phenytoin?
Linear
non linear

-CHADS2-VAS score is 5 what is the appropriate management? Oral anticoagulant


-CHADS2-VAS score is 5 what is the appropriate management? Oral anticoagulant

-anti arrhythmia drug that cause thyrotoxicosis ?


•Amiodarone or lidocaine ..

•Propranolol MOA ? non-selective beta receptor antagonist

- second mediator?
• serotonin ? or cytokines ..etc

-What type of vaccine is the cervical cancer vaccine? Human Papilloma virus vaccine

- case about newly diagnosed HTN patient asking about the normal range of systolic and
diastolic Bp ?
•115/80 or 170/90 or 140/70

-CI compilations in HTV drugs


•Lisinopril + valsaratan

-alkaloid will cause arrhythmia if it was taken iv large dose or something like that ..
theophylline

-Fungi cell wall content ?


Chitin

?Where you can find the antigens?


A- On the plasma cells
B- T-cells
C- B-cells > immunoglobulin / antibody

Which cells has Antibody on its surface?


B cell
T cell
T helper
T killer

Glycoprotein is found in ?
A-on the surface of the lipid bilayer of cell membranes
B-Mamalian cell
‫اﻻﺻﺢ ﻟﻮ ﻣﻮ;ﺣﻮده )ٮ)ﺤ&ٮﺎرﻫﺎ‬
C-E.coli
D-fungal cell

-showed structures and asked for one structure contained chiral centre ..

- pseudomonas DOC ? Ceftazidime depends on MCQ

-Definition of Category B in pregnancy ? Animal studies have revealed no evidence of harm to


the fetus

-MRSA treatment? Vancomycin

-management of status epilepticus? lorazepam or Phenytoin or valporate all are correct ?

- DOC prophylactic malaria in pregnancy?


Chloroquine and mefloquine all are

-evaluation of COPD
• exacerbation

-treatment of Angle-closure glaucoma ?


• timolol

-Ex on two compartments kinetic ?


Liver
Plasma
Etc

-Amino acids are absorbed in ? the proximal convoluted tubule (PCT)

Volume of distribution (Vd) means ?


•Conc of drug in blood Or
Equivalent distribution between tissue and blood
Conc of drug in tissue
Potent glucocortisone drug ?
Hydrocortisone
Methyprednisolone
Betamethasone

Symmetric distribution ( no skew)


A) median , mood, mean the same
B) median and mean the same different
mood
C) median, mood, mean all different
D) mean and mode the same different
median

Traveller diarrhea treatment :


a)-Ciprofloxacin 500 mg bid for 3 days
B)-azithromycin 500mg single dose

Patient has accident and take anti psychotic with Cimetidine for heart burn and have panic attack
s.e happened ?
A. cimetidine increase metabolism of drug
B. decrease metabolism

Decrease the duration and severity of disease:


A-primary prevention
B-secondary prevention
C-tertiary preventi

Least interaction with grapefruit.


simvastatin
pravastatin
lovastatin
Rosuvastatin

Pilocarpine used in all of the following except:


a)Acute attack of closed angle of
glaucoma
b) Standard drug for close angle glaucoma
c) Used alternatively with mydriatic to separate the iris
d)Standard one for open angle glaucoma

-Side effects of short half life hypnotics ?


Rebound insomnia
Develpment of tolerance

The master budget is a comprehensive financial planning


document. called?
-sales budget
- operation budget
-Organization plan

-Maximum dispensing for prescription of amphetamine from outpatient


department
30 60 90 180

bradycardia and QRS prolongation?


ACEI, diphenhydramine, propranolol , amlodipine

A hospital conducted a pharmacovigilance (PV) activity to detected events after certain


treatment, by asking patients directly and patient records. Which of the following is the type of
PV activity conducted by ?
A) Active PV
B) Passive PV
C) Mandatory PV
D)Spontaneous PV

A 23-year-old drug addict man was brought to the Emergency Room with severe agitation and
excitement. He was administered intravenous diazepam, with no improvement in his condition.
Which of the following is the mostly likely drug the patient was addicted on?
A.Opioids B.Cocaine
C.Pregabalin. D.Cannabinoids

The difference of prescription and herbal?


quality
need safet phase
need RCT

Newly graduated pharmacist hired in multinational.what would be his access of care?


A.General
B.Limited

Clinical pharmacist functions focus on


A. product
B. Hospital
C. Diagnosis
D. Patient

Who is currently responsible for financing and budgeting for healthcare services in KSA?
A. MOH
B. NUPCO

Manager wants to remove medications and wants to formulate a committee to assess and
evaluate he should assign who?
A. Planning B.Organizing
C.Controlling D. Leading

Alternative pathway if there is no glucose?


A. Ketones
B. Cholesterol
C. Triglycerides
D. Protein

Dispense the medications for chronic diseases every?


A. 3 months
B. one month
C.two month

A patient has diarrhea and the doctor wrote him a natural remedy to keep the fluids in order and
so that he does not lose more fluids, what did he advise him to do?
A/increase water intake
B/Sports
C/honey
D/Yoghurt

The presence of blood in vomiting indicates?


A/hemoptysis
B/hemateamesis
C/hematuria
D/hemacheziz

which patient population are at high risk of medication errors?


A- pediatric
B- adolescent
C- adult
D- childbearing age women

Patient with inflammatory acne asks pharmacist about cleansing product, what do you
recommend?
A. hydrocort 1%
B. mild facial soap
C. hydroquinine

-54 female patient on chemo started on cyclophos and anthracyclines what should do for her?
A-echocardiogram (ECG) ovaries preservation
B-acidification of urine alkalinization if urine

What is the main absorptive function of the colon?


A. Iron
B) Triglycerides
C) Water and glucose
D) Sodium, chloride, and water

elderly renal pt ?
A-Ibuprofen
B- sulindac
C- indomethacin
D-codaine+paracetamol
Identify the best definition of planning.
A. An integrated process in which plans are formulated, carried out and controlled
B. Devising ways of achieving the objectives of an organization.
C. Setting an organization’s objectives and the means of reaching them.
D. The core activity of planners and planning departments.

- Immune system works an important role in cancer treatment by ?


A. Macrophage
B. cytotoxic
C. complex

Which of the following is an intervention study?


A)Cohort
B) Ecological
C)Case-Control
D)Randomized Clinical Trial

In pharmacoeconomic studies, the costs (inputs) for types of analyses are measured in dollars
but the pharmacoeconomic method selected will depend on how the outcomes are measured.
Which of the following analysis described the method when patient (preferences for alternative)
treatments are being considered as the outcome?
A.Cost-Minimization B.Cost-Effectiveness
C.Cost-Benefit. D.Cost-Utility

Which of the following terms measures the association between the exposure and the
outcome?
A. P value
B.Risk ratio
C. Standard deviation
D. Confidence interval

On of the following medications is not affecting the potassium level ?


A- salbutamol
B- digoxin
C- insulin
D- sodium polystyrene sulfonate
E- fosinopril
E- fosinopril

Which electrolyte disturbance lead to digoxin toxicity ?


A. Na
B. K
C. Mg
D. Ca
‫ٮﺮ ﻣﻮ;ﺣﻮد )ٮ)ﺤ&ٮﺎر‬2‫ ﻟﻮ )ﻋ‬C

Which of the following cause digoxin toxicity


A. Hypomagnesemia
B. Hypercalcemia
C. Hypophospha

inhibits dihydrofolate reductase, preventing the reduction of dihydrobiopterin (BH2) to


tetrahydrobiopterin ?
A-Adenine
B-Thymine
C-Guanine
D-Cytosine

Which of the following is considered a direct method to measure adherence?


A. Pill count
B. Pharmacy record
C. Patient questions
D. Body drug concentration

Which of the following should not be done for sound alike medications?
A. Use tall man letter
B. Verbal order
C. Non-verbal order

Case of patient with severe allergic rhinitis and needs steroid that given orally?
A. Hydrocortisone
B. Prednisolone
C. Betamethasone dipropionate
D. Beclomethasone
drug cause delay of empty stomach ?
Atropine

Which of the following require monitoring?


A. IV UFH
B. SQ UFH
C. Enoxaparin
D.Fondaprinex

Hypertensive patients counsel?


A. Increase k intake
B.Increase zinc intake
C.decrease k intake
D.decrease zinc intake

advantages of CPOE system:.


1) reduction in medication errors
2) process improvement
3)cost-conscious decisionmaking, clinical decision support, and
efficiency

Which of the following is the best treatment for st.aureus positive mastitis ?
Cephalexin
Nitrofurantoin
Clindamycin

case asthma classification of patient has twice cough/w and no interference with daily activity ?
A. intermediate
B. mild persistent
C. moderate president
D. sever persistent

What is the maximum beyond use date of repackage drug


A. 3 months
B. 6 months
C. 9 months
-Narcotics prescription?
A-7 days
B- 3 days
C-1 month
D - 6 months
-Nom narcotic ? 1 month

Patient came to the pharmacy with prescription that was prescribed 95 days ago, what should
you do?
A. dispense the necessary medication only
B. dispense for one month only
C. ask the patient for the latest prescription
D. refuse dispen

1-A pregnant woman is travelling to a high risk malarial country which antimalarial
prophylactic agent caries the most fetal risk in pregnant women?
A. Mefloquine
B. Doxycycline
C. Chloroquine
D. Atovaquone-proguanil

Which of the following medications may alter the renal function?


A. isoniazid
B. gentamicin
C. acetaminophen
D. cyclophosphamide

64- Which of the following local organization's primary mission is to the safety and quality of
healthcare in health institution?
A) Saudi central board for accreditation of healthcare institutions (CBAHI)
B) Saudi Commission for Health Specialties (SCFHS)
C) Saudi Food and Drug Authority (SFDA)
D) Saudi pharmaceutical Society (SPS)

Polystyrene role ?
A- ca
B- k
C- ca
D- k (hypokalemia)

Rheumatoid arthritis take methotrexate and still not improved after 3 months ?
A. add Adalimumab and abatacept
B. add HCQ (Hydroxychloroquine)
C. continue methotrexate
D. Dswitch to another medication

Methotrexate contraindication in?


A - Pediatric
B - Osteoporosis
C- Breastfeeding
D- Heart Failure

42) Which of following prescriptions may have a potential to cause medication error ?
A. Amlodipine 5.0 mg PO OD
B. Thyroxin 25 mcg PO daily
C. Acetaminophen 500mg PO TID
D. Regular insulin 5 units subcutaneously STAT

To ensure safe and proper disposal of hazardous waste, a color- coded system was created.
Which of the following colors is the color of hazardous waste containers?
A. red
B. blue
C. yellow
D. orange
*[Natcotic Red]
*[Controlled yellow]
*[Hazardous yellow]

Which of the Following is required to consider an adverse drugs event report


A. drugs dose
B. patients tall
C. event name
D. reports phone
-Which of the following drugs require extra precautions during handing to ensure medication
safety?
A. Methotrexate
B. Potassium
C. Dopamine
D. Thiamine

An ambulatory care pharmacist received a prescription for (liraglutide) (REMS), from Bariatric
Endocrine consultant as per medication under (REMS) should be provided to the patient with
documents.Which of the following is the meaning of (REMS) beside the name?
A) This medication has teratogenicity effect
B) This medication is under (restriction and evaluation medication Strategy)
C) This medication is under (Risk Evaluation and Mitigation)
D)This medication is under (Restriction Evaluation Medication)

39)-A patient medication was changed from carbamazepine to phenytoin The technician pulled
the 100 mg tablets of phenobarbital off instead of the 30 mg tablets. The pharmacist did not
notice that when the prescription was checked.The patient received a higher phenobarbital and
was hospitalized for few days for observation. Which of the following is the best statement
concerning this situation?
A) This is considered as adverse drug reaction
B) The physician should have informed the pharmacist about prescription's change
C) The patient should be aware of what he was prescribed and the difference
D) This is preventable medication error if the pharmacy in safety measures

Which of the following medication error is difficult to prevent?


A-Omission error
B-Wrong time error
C-Incorrect duration of treatment
D-Allergic reactions to an unknown patient

hyperkalemia 6.8 mmol/L treatment,


1-Insulin and dextrose
2-Dialysis

which type of data the researchers collect from patients before submitting
the research (propsal)?
A- Survey
B- Primary
C- Secondary
D- Laboratory

Animal reproduction studies have shown an adverse effect on the fetus and there are no
adequate and well-controlled studies in humans,28- p value ?
A- > signficant clinical
B- < signficant statical
C- > signficant statical
D- < signficant clinical

Anti arrhythmic drugs used in supra-ventricular and Ventricular Arrhythmia


Sotalol
phenytoin
Flecainide
Procain

Which IV container need filtered needle?


Vial
Bag
Ampoule
All IV preparation need filtered needle

-clinical pharmacist during the round they ask her to choose one antibiotic for C.diff,and after
searching she found that vancomycin orally bid is the best to choose ,from where she conduct
the information?
-expert opinion
-cohort study
-rct

24-year-old female is diagnosed with genital herpes simplex virus


infection. Which of the following agents is indicated for use in this diagnosis?
A. Valacyclovir.
B. Cidofovir.
C. Ganciclovir.
D. Lamivudine.
D. Lamivudine.

Patient who have history of MI and HTN and he is diagnosed with ostroarthritis and currently
taken acetaminophen 650 mg every 6 hours but still suffering from continuous knee pain . What
is the appropriate management ?
1- celecoxib
2- Naproxen
3- oral GLUCOSAMINE
4- oral hydrocodon

compare the safety of one medicine with another medicine it would be necessary
to:
a) Review the evidence from Phase I clinical trials.
b) Review the evidence from Phase II clinical trials.
c) Review the evidence from a post marking clinical trial.

High sterilized substance can be kept in room temperature for how long ?
A. Up to 24 hrs
B. 3 days
C. 45 days

Drug A stablity in ph ( 1 - 8) in which ph is less stable?


1
5
7
8

According to SFDA, what is the permitted age to dispense cold medications to children?
A. Up to 2 years
B. Up to 6 years
C.Up to 4 years

Drug need incremental dose for elderly?


Clomipramine
Losartan
Thyroxin
Lisinopril
iron interaction with?
mg
levothyroxin
metformin

Eye misaliment in children


- astigmus
- starpismus
- olyopia

Aspirin/ ibuprofen separation?


A-Take ibuprofen 4 hr before apirin dose
B-Take ibuprofen 8 hr before apirin dose

Plan b contraceptive take about?


A. 24 h B. 48 h C. 72 h

Buy inventory then reselling to customer?


Cost of good sell

Which plant is 5Ht3 activity ?


Ginger

Age groups isinactivated polio recommended for?


Children up to 6 years old

Design could be deemed an study ?


Clinical trial

What the main method transmission of brucella species?


Ingestion of unpasteurized milk/milk product

Immunoglobulin activates complement system?


A. IgG
B. IgM

‫ ؟‬immuneglobuline ‫اﻻﻃول ﻣﺪه‬


a)IgG
b)IgE
c)IgM

Drugs exhibits dose-dependent pharmacokinetics at normal therapeutic doses?


Phenytoin

Vaccine give for all family ?


Herd
Spical
Local

All the following diazoxide side effect except?


Diureses
Tachycardia
Hyperglycemia
Hyperuricemia

Used to detemine the significance of the result?


P value

BB with partial agonist ?


Labetalol
Propranolol
Clonidine
Aciputalol

Patient culture is E.coli ( ESBL ) what you will give ?


A. Ciftriaxone 500 mg IV Q12H
B. Ciftriaxone 500 mg IV Q 24 H
C. Meropenem 500 mg IV Q 8 H
D. Meropenem 500 mg IV Q 12 H

Acetyl coA and cetric acid cycle which vitamine is used ?


A. B6
B. B12
C. B2
D. B9

Bond between Carboxyl and amine ?


A. hydrogen bond
B. b- ionic bond
C. vander wal
‫ ﻟﻮ ﻣﻮ;ﺣﻮده‬peptide bond ‫)ٮ)ﺤ&ٮﺎرﻫﺎ‬

A 43-year-old man has abnormal liver function tests noted at a routine work related medical. He
has a history of angina.
Which of the following drugs is most likely to be responsible?
A. Atorvastatin
B. Amlodipine
C. Aspirin
D. Atenolol

What is considered moderate intensity statin?


Atorvastatin 20 mg
Pravastatin 20 m

‫ٮﻦ ا&ڡﻞ‬2‫ ﻣ‬drug drug interaction ?


A- Simva
B- Atrova

Graph presented in the question. Calculate Therapeutic index in graph ED 100 , LD 400:
A) narrow therapeutic index
B) Wide therapeutic index

50(‫ٮﻦ )ٮ&ڡول ﻣ&ٮﯩﻌﻦ اﻻﺳ&ٮﺎٮ&ﺴ&ٮﻚ‬2‫ إ)ٮﻬﻢ اﻟﺪاٮ&ﺎ ﻣﻦ ﻣ;ﺤﻤﻮﻋ&ٮ‬equivalent?


a)if Mean and median and mode are equal
b)if Mean and median and mode are not equal
c)if Mean and median are equal but mode not

cells has role in inflammation?


a)Basophiles
b)Neutrophils
MOA of lactolose?
A- acidify colon and limit ammonia
B- alkalization colon and limit ammonia

which microorganism cause Lyme disease?


• virus
• Fungi
• Bacteria
• Parasite

women take Combination contaceptive and heavy smoker, no family history HF


or VTE
A) add statin
B) continue CC and add statin
C) use barrier contraceptive
D) use only progesterone contraceptive

Which of the following strategies are used with low literacy patients to Encourage them to
change their behavior?
A. Keep information focused and organized
B. Use a normal speaking tone and volume
C. Use briefer educational sessions
D. Use more visual aids

Which of the following strategies are used with older adult patients to encourage them to change
their behavior?
A. Use teach-back more often throughout the session
B. Introduce new concepts more slowly
C. Use briefer educational sessions
D. Use more visual aids
[Second parts:]

Rate control for patient with ischemic heart diseas


*( metapolol, bisoprolo )*

which of the following preparations should be refrigerated regardless of it is ingredient?


A.eye drops
B.suppository
C.oral solution
D.capsules

whats wrong with iron in geriatric patients


*decrease absorption *

‫ﺶ )ٮوع اﻟ)ﺤﻄﺄ ؟‬2 ‫ اٮ‬، ‫ٮﻪ اﻟﻤريﺾ‬2‫ﻌﻄ‬2 ‫ٮﻪ )ﺣﻄﺎ &ڡ;ٮﻞ ﻻٮ‬2‫ى ا ;ٮيﺼﺮ)ڡﻪ ان )ڡ‬+‫ٮﺪل‬2‫ى واﻛ&ٮﺸﻒ اﻟﺼ‬+‫اﻟ)ڡﲏ ﺻﺮف دواء ;ٮﺪل دواء ٮ_ﺎ)ٮ‬
*Dispensing*

Drug need monitoring ?


*theophylline*

Flash tech for which tablet ?


immediate release
sustained release tablets
‫ وﻟﻮ ﰷ)ٮﺖ ﻣﻮ;ﺣﻮده‬fast ‫اﺻﺢ ﳾ )ٮ)ﺤ&ٮﺎرﻫﺎ‬

- Virus associated diarrhea


*lactobacillus*

? Geriatric calcium dose


1200

Ciprofloxacin + metronidazole for treatment ?


*mild to moderate crohn*

Hormone cause uterine contraction during child birth ?


*oxytocin*
*oxytocin*

Drug inhibit xanthine oxidase ?


*Allopurinol*

Clozapin dose for female’s ?


*5 ml*

Antimetabolite drug ?
*anti cancer*

Drug cause miosis and respiratory depression ?


*opioid toxicity*

First sign of breast cancer ?


*painless lump*

Tamoxifen use ?
*treat breast cancer*

HLAB 5702 ?
*abacavir*

Cholinergic antagonist ?
*Ipratropium bromide*

Antiarrhythmic drug cause iodide hypothyroidism ?


*Amiodarone*

Boric acid function in eye drops ?


*cleanse or irrigate the eyes*

Long acting insulin ?


*insulin glargine 24 h*
*insulin degludec 42 h*

.how long the control drug should be administered by IV rout ?


*24 h*
*24 h*

20 weeks pregnant woman suffering from dezzyness and tiredness , lapresults show low
hemoglobin , what to do ?
*Ferrous sucrose iv*

Why give vasopressin with NE in septic shock ?


*To prevent kidney necrosis*

Patient have heart failler + mucuse in lung , diffeculty breathing , what is the condition ?
*Wet and cold*

dissolve thrombus by formation of Fibrinolytic plasmin from plasminogen ?


*Alteplase*
*Streptokinase*

Lowesst potency Glucocorticoids ?


*Hydrocortisone*

Phenytoin decrease IV dose monitor after how long ?


*4 hr*

What not to use in IV ?


*non sterile material*

xl dosege?
slow release
sustaind release

what will happen if you break down sustaind release drug?


toxic
more absorbed

the past 7 years. Can she receive zoster vaccine


(ZOSTAVAX) now?
b. Yes, she needs to receive a dose of the
vaccine now, and an additional dose after
completion of chemotherapy to ensure a full
immunogenic response
c. No, she should not receive the vaccine due to her diagnosis of cancer and because she is
currently receiving chemotherapy
d. No, she does not need to receive the vaccine because it will not help since she has
experienced past outbrea

How long does it take before a flu vaccine


provides you with maximum protection against
the flu?
2 weeks

Healthy children over what age should not


receive the Hib vaccine?
Five years

What part of an immunoglobulin is responsible for recognizing foreign things?


o Fc region
o Heavychain
o Lightchain
*‫ٮﻪ‬2‫ ﻟﻮ )ڡ‬:*‫ ﻟﻠ&ٮﻮﺿيﺢ‬Fab region ‫)ٮ)ﺤ&ٮﺎره‬
*‫ ال‬Antibody ‫ ٮ&ﺮٮ&;ٮﻂ ﻣﻊ‬Fc*
*‫ ال‬light ‫ ٮ&ﺮٮ&;ٮﻂ ﻣﻊ‬Antigen*

! Which one consider as physical examination?


- Regulatory
-Precaution

q- if the 1 mg of drug soluble what could be 20mg ?


freely soluble
very soluble
soluble
sparingly soluble

A 28 year-old woman has been raking fluoxetine for depression and now has worsening
depressive episodes.The resident doctor on the floor has been asked by the consultant ro stop
fluoxetine and initiate phenelzine.
How long should the washout period be before starting phenelzine?
A. 2 weeks ‫ﻪ‬2 ‫ٮﻦ ﻛ)ڡﺎٮ‬2‫;ٮي&ڡﻮﻟﻮ اﺳ;ٮﻮﻋ‬
A. 2 weeks ‫ﻪ‬2 ‫ٮﻦ ﻛ)ڡﺎٮ‬2‫;ٮي&ڡﻮﻟﻮ اﺳ;ٮﻮﻋ‬
B. 3 weeks
C. 4 weeks
D. 5 weeks

Which medication requires negative pregnancy test before dispensing to woman of child bearing
age ??
A. Labetalol
8. Prednisone
C. Ceftriaxone
D. Isotretinoin

What is the pharmacological category for insulin lispro?


A. Long acting insulin
B. Rapid acting insulin
C. Short acting insulin
D. Intermediate acting insulin

Which of the following medications require observing the patient for six hours with hourly heart
rate monitoring after the first dose?
A. Verapamil
B. Bisoprolol
C. Amlodipine
D. Fingolimod

Which of the following indications is appropriate for misoprostol?


A. Misoprostol is ineffective for prevention or treatment
B. Misoprostol is effective for both prevent ion and treatment
C.Misoprostol is effective for preventing NSAIDs induced ulcer
D.Misoprostol is effective for treatment of NSAIDs induced ulcer

A 15 month-old boy is brought to the Emergency Department by his parents with high grade
fever, vomiting, and non-blanching rash. He is started immediately on cefotaxime. The cerebral
spinal fluid culture reveals Neisseria meningitides.
What would be the recommendations regarding prophylaxis of his family and close contacts?
A.His parents should receive rilampicin
B.Antibiotic prophylaxis is not indicated
C.Patient's close contacts should receive rifampicin
D. Siblings under the age of 18 should receive rifampicin

In which of the following patient categories the (gentamicin) extended interval regimens (once
daily dosing) is most suitable?
A. Pregnancy
B. Burns patient
C. Gram Negative Infections
D. Enterococcal endocarditis

A 28 year-old woman with a history of seizures that is well controlled on valproic acid is asking
for advice regarding her medications.She is planning to start a family and is now taking folic acid.
She wants to optimize the medications before her pregnancy.
What would be the best recommendation?
A.Introduce phenytoin to valproic acid
B.Discontinue valproic acid immediately
C.Continue on valproic acid since
she is well controlled
D.Introduce levetiracetam slowly and titrate the dose of valproic acid to discontinue before
pregnancy

Which of the following medications can be teratogenic if a pregnant woman handles crushes or
breaks the tablet?
A. Bosentan
8. Alfuzosin
C. Topiramate
D. Didanosine
‫ٮﻪ‬2‫ ﻟﻮ )ڡ‬Finasteride ‫ﻞ ﻋ)ٮﻪ‬2 ‫ٮﺐ ;ٮﺪٮ‬2‫;ﺤ‬2 ‫ )ٮ)ﺤ&ٮﺎره وﻣﻤﻜﻦ ٮ‬dutasteride

Which of the following represents the immunity acquired by give living organism with
attenuated virus?
A) Local immunity
B)Passive immunity
C) Natural active immunity
D) Artificial active immunity

Attenuated mean?
Active virus
Killed virus
Inactivated virus

A 60 year-old woman has been recently diagnosed with Stage II hypertension.


Which of the following medications would be suitable for her ?
A. Losartan/hydrochlorothiazide ‫اﻻﺻﺢ‬
B. Atenolol/nifedipine
C. Valsartan/furosemide
D. Amlodipine/triamterene

2D6 poor what to give?


Tamoxifen
Raloxefine

Which of the following antibiotics is used in surgical prophylaxis of colorectal surgeries?


Ampicillin
Cefoxitin
Linezolid
Augmentin

moderate physical activti?


swimming
basketball
brisk walking

Which of the following of the hypertensive therapies is preferred to be used


during the acute phase of ischemic stroke if elevated blood pressure should be
treated?
A)lisinopril
B)nicardipine
C)hydrochlorothiazide
D) metoprolol succinate

Erythromycin and birth control interaction


o Increasebirthcontroleffect
o Decrease birth control effect o IncreaseErythromycin
o DecreaseErythromycin
o No interaction

- Mortality of children was 8 in 2012 and 6.4 in 2017, This mean?


A- More hospital open
B- Improve health
C- Nothing, more information need

72 kg Patient diagnosed with a fib and travels a lot do not have time what to give?
A. warfarin 5 mg
B. apixaban 2.5 mg bid
C. enoxaparin 40mg
D. enoxaparin 70 kg bid

Which of the following drugs is associated with progressive multifocal leukoencephalopathy side
effect?
Bisoprolol
Pregabalin
Doxycycline
Natalizumab

Which one of the flowing best represents a direct medical cost?


A-pain
B- Transportation
C- Mortality
D- Medical professional time

Patient with severe gastric pain to right upper quadrant with vomiting also pain to back also get
worse with fatty meals ?
A/pancreatic B/gallstones C/hepatic disease
‫ اﻻ)ﻋﻠﺐ &ڡﺎل‬B ‫ ;ٮﺲ ا)ٮﺎ ﻋﻦ )ٮ)ڡﴘ اﺷﻮف‬A ‫ٮﺤﻪ‬2‫ﺻﺤ‬

Which of the following is the Best and safest laxative for chronic constipation ?
A. Bisacodyl
B. Senna
C. Glycerin suppositry
D. Metamucil (psyllium)
*‫ى ﺣﺎﻟﺔ ال‬+‫ )ڡ‬Acute ‫ )ٮ)ﺤ&ٮﺎر‬C*
*‫ى ﺣﺎﻟﺔ ال‬+‫ )ڡ‬Acute ‫ )ٮ)ﺤ&ٮﺎر‬C*

Pt medications : Genvoya, insulin, amlodipine, spironolactone, Calcium gluconate. What is


theinteraction ?
a- amlodipine with Ca
b- Genvoya with Ca
c- Genvoya with amlodipine

Secondary mediator in type 1 hypersensitivity


A/T cell
B/ B cell
C/ cytotoxic
D/mast cell

which one cause mononucleosis?


A-Epastin
B-hymofylus
C-salmonella

Clinical pharmacist during the round they ask her to choose one antibiotic for C.difficle and after
searching she found that vancomycin orally bid is the best to choose, from where she conducts
the information??
A. Expert opinion.
B. Cohort study.
C. Retrospective.

Which of the following is a B-lactamse enzyme?


A) Plasmid
B) Pyrogens
C) DNA gyrase
D)Carbapenemase

Drug should not use be take with ciprofloxacin?


acei
ferrous sulfate
atorvastatin
metformin
metformin

Which of the following drugs has a high abuse potential?


A) nifedipine
B) allopurinol
C) varenichine
D) methylphenidate

28-year-old woman who is 8 months pregnant is taking naproxen 500 mg twice daily for the past
four weeks to treat severe backache. Which of the following health issues at the time of birth
puts the neonate at higher risk?
A. asthma
B. cardiac arrhythemias ‫ا&ڡﺮب ﳾ‬
C. uncontrolled bleeding ‫ٮﺤﻪ‬2‫;ٮﺮﺿﻮ ﺻﺤ‬
D. gastroesophageal Reflux Disease

which the following treatment Meningitis empiric treatment ?


A. Vancomycin
B. Ceftriaxone 2g 12 h IV 7-14 day
C. Ampicillin
D. Azithromycin

Calculation question: patient take warfarin and stop at 2:00 am warfarin T1/2
is 1.5 hr , what is the time needed to go to surgery?
A) 16:00
B)22:00
C)3:00

What is the best time to do lithium levels after the dose is administered?
A.4 hours
B. 6 hours
C. 8 hours
D. 12 hours (It usually 8-12 h)

muscle numbness on his mouth?


hypophospteamia
hyperphsotamia
hypocalcemia
hypercalcemia

Which of the following is a late adverse


effect of lithium?
(A) nausea
(B) hand tremor
(C) seizures
(D) hypothyroidism

Which of the following factors may increase lithium concentration?


(A) caffeine
(B) osmotic diuretics
(C) increased fl uid intake
(D) nonsteroidal anti-inflammatory drugs

A pregnant woman has sulfa allergy, she suffers from vaginal itching her analysis shows positive
E.coli
what is the antibiotic of choice for her infection?
A-Ciprofloxacin
B- Septazole
C-Nitrofurantoin
D-Tetracycline

morphine refill?
0
1
2
3

3) part of the nerve and has role in covering and protecting the neuro impulse and conducting
it ?
a) Axon
b) Mylen sheath
c) Soma
d) Dendritis

Contraindicated in breastfeeding?
A. warfarin
B. Lithium
C. insulin
D. none of the above

Which of the following drugs is a common cause of interstitial nephritis?


(a) Metformin
(b) Ranitidine
(c) Lithium ‫اٮ&ﻮ&ڡﻊ اﻷﺻﺢ‬
(d) Omeprazole
(e) Ondansetron

What is the Intestine PH range?


A. pH 5 to PH 6
B. pH 6 to pH 7.4
C. PH 7.4 to PH5.7
D. PH 5.7 to pH 6.7

Best diuretic in ADHF ? (Acute decompensated heart failure)


A. furosemide
B. spironolactone
C. HCT

Which of the following is Filamentous anaerobic bacteria ?


A. Psuedomonas aeruginosa
B. Actinomycetes
C. Mycobacterium tuberculosis

Which of the following is the most rapid relief of acidity?


A. Omeprazole
B. Antacid
C. Sucralfate
D. H2 receptor antagonist

Which drug we can use it before niacin to prevent flushing ?


A. Vitamin C
B. Aspirin
C. Ibuprofen

86- A 55-year-old man comes to the pharmacy complaining that has acetaminophen with
codeine he started 2 days ago to manage has post- operative pain does not seem any better
than when he uses acetaminophen alone. After the pharmacist reviews his profile, he noticed
results from pharmacogenomics testing performed 3 years ago that shows he is a CYP2D6 poor
metabolizer.Which is the best explanation why this patient does not seem to benefit from
codeine?
A. Codeine excretion is rapid
B. Codeine absorption is slower
C. Codeine is metabolized faster
D. Codeine dose should be increased
‫ٮﺎرات ﰷ)ٮﻬﺎ )ﺣﻄﺎ‬2‫اﻟ)ﺤ‬
‫ اﻟﺼﺢ‬metabolized slow ‫ ﻻ)ٮﻪ‬cyp2d6 low ‫ وذا اﻟﲇ ;ٮيﻌﻤﻞ‬metabolism

Food not used with amiodarone?


A. Watermelon
B. Grapefruit
C. Green tea
D. Pomegranate

Which cephalosporin has no antipseudomonal activity and should not be used in the
management of pseudomonas
aeruginosa pneumonia?
A. Cephalexin
B. ceftazidim
C. cefipime
D. none of the above

Which pharmacokinetic parameter estimates the renal function?


A. serum creatinine
B. intrinsic ciearance
C. blood urea nitrogen
D.creatinine clearance

TCA act on all of the following receptors expect:


a) Histamine receptors
b) Cholinergic receptors
c) Alpha receptors
d) Beta receptor

Prophylaxis from migraine aura?


A. Ergotamine 4 mg
B. Sumatriptan 50mg BID
C. Propranolol 20mg BID

Case patient take imipramine and asking TCA used in all of the following expect:
a) Panic attack
b) Schizophrenia
c) Migraine
d) Neuropathic pain

What drug should stop before TPN?


Fentanyl
Tetracycline

Which of the following is characteristic of BBB?


A) passive diffusion of glucose-independent insulin
B) allow cross glucose - depend on insulin
C) allow crossing polar drugs
D) prevent non polar drugs

Substance released from the liver in response to IL-6 secretion ?


A- CRP
B- IL-2
C- Immunoglobulin

Which of the following is the major risk associated with the use of over-the- counter drugs?
A. Increased cost without benefit
B. Decreased drug-drug interactions
C. Increased potential for misuse and abuse
D. Increased risk of adverse events

Women newly get pregnant and preform diabetic test in the beginning of her pregnancy
negative, what is your recommendation
regarding gestational diabetic test?
A)No need for testing
B) Do another test between 4 and 12 weeks
C) Do another test between week 20 and 24

sever depression and anxiety can’t give patient ?


A- mirtazepine
B- bupirione
C- Fluoxetine

- Phenobarbital prescription as outpatient for how many days ?


A. 30
B. 60
C. 70
D. 90

-Pt wit metabolic alkalosis and edema use ?


A- ringer
B- isotonic
C- acetazolamide

259- Which of the following prevent renal injury ?


A- Vit C
B- Vit D
C- i.v fluid

-what is the drugs we do not give old people ?


A- amlodipine
B- metoprolol
C- Metoclopramide

what is the recommended dose of IV N-acetyl cysteine for 6 years child ?


A. 2_5mg/kg
B. 20-15mg/kg

harmacological group cause hepatic injury?


• Purine,
• pyrolizidine
dihydropyridin

Women during pregnancy took high doses of ampicillin


and after birth, the baby has high bilirubin. What is your intervention
a) Ceftriaxone (‫)ﻣﻤ)ٮوع ﻟﻠﺮﺿﻊ ا&ڡﻞ ﻣﻦ ﺷﻬﺮ‬
b) Gentamycin
c) No antibiotic needed ‫أﺻﺢ‬

Which ONE of the following is NOT a reason for intentional non-adherence


a. Reducing doses to avoid side effects
b. Forgetfulness
c. Concern about addiction
d. Belief that the medication doesn’t work
e. Unable to afford the prescription charge

What is more effective method to prevent pregnancy ?


A. Oral contraceptives
B. Latex Condoms
C. Spermicides
D. intrauterine device ‫أﺻﺢ‬
‫ &ڡﺎﻟﻮ ان‬d ‫ٮﻢ ;ٮﺮﺿﻮ اﻷ&ڡوى‬2‫أﺻﺢ ﻣﲎ )ٮﺼﺮ واذا ﻣﺎ;ﺣﺎ;ٮﻬﺎ و;ﺣﺎب اﻟ&ٮﻌ&ڡ‬

best way to collect volatile oil ?


A- gas chromatography
B- liquid chromatography

Plague transmitted by
a) House flies
b) Fleas
c) Mosquitoes

Phenytoin Side effects except:


-Gingival hyperplasia
-Acne
-Hirsutism
-Nystagmus
-Nephrotoxicity ‫أﺻﺢ‬

-Steven-Johnson syndrome
Tonic-colonic seizures DOC is:
a) Phenytoin
b) Ethosuximide
c) Valproate

Pharyngitis, penicillin allergic, organism sensible to


bacitracin:
a) Azithromycin
b) Tetracycline
c) Cefaclor
d) Amoxicillin

Used in treatment of AIDS:


a-Acyclovir
b-Rifampicin
c-Zidovudine
d-Prednisolone

Nephrotoxicity is the side effect of:


a- Paracetamol
b-Metronidazole
c- Vancomycin
*d- Gentamycin*

Hepatotoxicity is the side effect of:


*a- Paracetamol*
b-Metronidazole
c- Vancomycin
d- Gentamycin

What is the major complication of morphine overdose?


a – AV-Block
*b- Respiratory depression*
c-Cardiac Arrest
d Liver dysfunction

Which one of the following is considered ultrashort acting barbiturate?


*a- thiopental*
b- secobarbital
c- amobarbital
d- phenobarbaital

Which one of the following is considered long acting barbiturate?


a- thiopental
b- secobarbital
c- amobarbital
*d- Phenobarbital*

Pseudomembranous colitis can be treated by


a- *oral vancomycin plus metronidazole*
b-vancomycin plus tetracycline
c- ampicillin plus metronidazole
d- amoxacyillin plus metronidazole

Which aminoglycoside antibiotic can be taken orally?


a-gentamycin
*b-neomycin*
c-amikacin
d-tobramycin

Patient forget her aspirin tablet in bathroom what will happen to the tablet ?
A/oxidation
B/ Reduction
C/ demethylation
D/ Hydrolysis
‫ اذا ﻣﻮ ﻣﻮ;ﺣﻮده )ٮ)ﺤ&ٮﺎر‬A

Pt has lung infection and use levofloxacine his INR is 5 this fluctuation considered as which
Interaction ?
A/drug disease
B/drug lab
C/ drug -drug

What's the variation in gene called?


Allel
Genotype
Phenotype

Which antiepileptic drug causes nephrolithiasis?


A. Zonisamide
B. Oxcarbazepine
C. Rufinamide

-Vaccine only oral?


Rota

-Vaccine taken orally or Im ?


Polio

New diseaes affect people and specific time?


A. Mortality
B. Morbidity
C. Incidence
D. Prevalnc

New born with fever,dyspnea ,and Jaundice. He can take?


A. Ampicillin and Gentamicin
B. Ceftazidime ang Gentamicin
C. Ampicillin and Ceftriaxone

Which of the following is the best treatment for st.aureus positive mastitis ?
Cephalexin
Nitrofurantoin
Clindamycin

- Before 6-week Pt did surgery and he wanted to lift the hospital, which discharge
vaccine should he give?
A. meningitis
B. influenza
C. pneumococcal

-(Case) diabetic patient needs drug for


hypertriglyceridemia?
A. Statin
B. Glifibrozide
C. Cholestyramine
D. Eztimibe

which of the following drugs passes into mothers milk due to high lipid solubility?
A) diazepam
B) methyldopa
C) promethazine
D) dihydroergotamine

Treatment patient with acute crisi anemia


A. Morphin
B.Hydroxyurea

Metabolic alkalosis + edema treatment?


A. Ringer lactate
B. Isotonic solution
C. Acetazolamide

Reduce of atherosclerosis?
Green tea
Olive oil

QT segment time:
400 seconds
0.4 seconds
0.004 seconds
4 seconds

IM injection:
3ml or 5ml

drug used after trial in hospital ?


A. internal validity
B. external validity
C. reliability

Anti diabetic drug used in pregnancy?


A- rosiglitazon
B- Metoformen
C- glargine is long acting insulin
*glargine CI in pregnancy* ‫ﻟﻠ&ٮﻮﺿيﺢ‬
‫ى )ٮ)ﺤ&ٮﺎره‬+‫ٮﻦ ٮ_ﺎ)ٮ‬2‫ﻟﻮ )ٮوع ا)ٮﺴﻮﻟ‬

- blood pressure in neonate ?


A. same adult
B. Higher than adult
C. Lowed that adult

Morphine to heroine??
A- Carboxylation
B- estrefication

Which medication can cause orthostatic hypotension? A) prazosin B) donepezil


C) rivastigmine D) spironolactone

-Metabolic acidosis what is decrease ?


A- Pco2
B- HCO3
C- CO2
-Metabolic acidosis what is increase ?
A- Pco2
B- CO2
C- COH

Which one of the following questioning techniques uses hints and aid patients in answering
questions?
A) Changing questions
B) Empathic responding
C) Prompting while asking
D) Redirection the questions

Least drug interact with BB??


a) phenytoin
b)phenobarbital
c)rifampicin
d)fluoxetine

Which governmental agency gives approval to generic media enter the drug market in Saudi
Arabia after completing all requirements?
Ministry of Health (MOH)
Saudifood and drug Authority (SFDA)
Saudi Commission for Health Specialists(SCFHS)

Ose of caffeine in neonate where to look?


Drug in pregnancy and lactation
Hospital formulary drug
Harriet lan handbook

-Cell that produces plasma cells?


T cell
B cell
Macrophage
-activator receptor of mast cells ?
B2
Siglic
Platelet activator receptor

-Ceftriaxone:
Adipose tissue, brain, liver
-Ciprofloxacin:
Adipose tissue, liver, brain
Pigggyback mean ?
TPN
iv

Which detoxification pathway is active paracetamol toxicity?


A. Oxidation
B. Reduction
C. Conjugation with glucuronide
D. CYP-450 dependent glutathione

Drugs prolonging QT interval:


Methadone
Amilodepine
Linezolid
‫ٮ&ٮﻪ‬2‫ٮﺎر را;ٮﻊ )ٮﺴ‬2‫)ﺣ‬

Drug prolonging QT interval :


Amilodepine
Citalopram
‫ٮ&ٮﻬﻢ‬2‫ى )ٮﺴ‬+‫ٮﺎريﻦ ٮ&ﺎ)ٮ‬2‫)ﺣ‬

Which contain sugar part and non sugar part ?


Glycoside
Glucose
Protein

-A patient is found to have been lacking in functional CYP2C19 enzyme activity. What would be
your recommendation?
a. He should not receive Clopidogrel ‫اﻻﺻﺢ‬
b. He should not receive any CYPC19 enzyme inhibitors such as omeprazole or fluoxetine
c. If clopidogrel is administered, he will have increases INR
d. He is an ideal candidate to receive Clopidogre

Tablet what type of risk?


Mediums
High
Low
Bioavailability rate limiting step in tablet after reducing particle size-171:
-Absorption rate limiting step
-Dissolution rate limiting step
-Permeability rate limiting step
*‫ٮﻪ‬2‫ ﻟﻮ )ڡ‬Disintegration ‫ٮﺎرات ﺣ&ٮﻜون اﻟﺼﺢ‬2‫*)ڡﺎﻻ)ﺣ&ٮ‬
*B for solutions not tablelt*

Pt with AF and torsade de point which anti arrhythmic use for this pt?
1- amiodarone
2-varpamil
3-defolitde
4-sotalol

A fishing scenario that is not preparing a report on expenses and profits. What type of report is
it?
-Revenue
- Balance sheet
- Income statement
- Cash flow

2 years skin infection with penicillin ?


1-doxycycline
2-Amoxicillin +Clavulanic acid
3-sulphamethoxazol+trimethoprime

Lipinski 5 oral dosage form ?


A - Molecular mass less than 500
B- Hydrogen donar
C- Hydrogen acceptors
D- Partition co efficient

When evaluating patients with angina for possible development of nitrate tolerance, true
statements include
I. Tolerance is rarely associated with isosorbide dinitrate
II. Tolerance doesn’t occur in patients who are concurrently on metoprolol
III. Tolerance is dependent upon the
administration schedule of nitrate medication?a. I only
b. III only
c. I and II only
d. II and III only
e. I, II and III

Parenteral products with an osmotic pressure less than that of blood or 0.9% sodium chloride are
referred to as ?
A-isotonic solutions.
B-hypertonic solutions.
C-hypotonic solutions.
D- iso-osmotic solutions

What is the best hypnotic for geriatric ?


A. Alprazolam
B. Diazepam
C. Phenobarbitone

NSAIDs has the gastrointestinal side effect? Naproxen


Meloxicam
Ibuprofen
Diclofenac

Cockcroft gult use to measure creatinine clearance in?


A-acute kidney disease
B-stable kidney disease

Where in the cell is cytochrome P450 located? a) Mitochondrial inner membrane


b) Cytoplasm
c) Mitochondrial matrix
d) Endoplasmic reticulum

Which of the following proteins transports foreign molecules out of cells?


a) Mixed function oxidase
b) UDP-glucuronosyl transferase
c) Glutathione-S-transferase
d) Multidrug resistance P-glycoprotein
What enzyme, or combination of enzymes, protects cells against superoxide generated in
oxidation reactions?
a) Superoxide dismutase
b) Catalase
c) Superoxide dismutase plus catalase d) Glutathione peroxidase

-Fick’s law ‫ﺶ‬2 ‫? ٮ&ت;ٮﻊ ﻻٮ‬


Diffusion
-Noyes Whitney?
Dissolution
-Henderson Hasselbatch?
PH

Reaction that convert pentazocine to morphine? ‫ﻣﻤﻜﻦ )ٮ&ڡﻞ اﻟﺴؤال )ﻋﻠﻂ‬


‫ٮﻦ‬2‫ٮﻦ ﻟـ ;ٮي)ٮ&ٮﺎزوﺳ‬2‫اﻟﺮﺳﻤﻪ ;ٮﺎﻟﺴﻬﻢ ﻣﻦ ﻣور)ڡ‬:
A.Extention
B.Simplification
C.Rigidifacation
: ‫ٮﻦ )ٮ)ﺤ&ٮﺎر‬2‫ٮﻦ ﻟـ ﻣور)ڡ‬2‫ ﻟﻮ ﻣﻦ اﻟ;ٮ)ٮ&ٮﺎزوﺳ‬:‫* ﻟﻠ&ٮﻮﺿيﺢ‬
C (Rigidifacation)

Which of the following is the type of studies considers data from multiple studies of different
designs to draw conclusions?
o A. Case Series
o B. Double-blind study
o C. Systematic Review
o D. Consensus statement

Researchers compare a group of people who have diabetes with of people who have no diabetes,
and looks back in time to see characteristics of the two groups differ.
Which of the following is the described study design? o A. Prospective cohort
o B. Case-Control
o C. Experimental
o D. Randomized clinical

Which one of the following designs could be deemed an study?


o A. Cohort
o B. Clinical trials
o C. Case-control
o D. Cross-sectional

A 61-year-old was recently diagnosed with parkinson’s disease He was prescribed an anti-
parkinson medication. Later, he was presented to the clinic with mydriasis and narrow angle
glaucoma. Which of the following medicationwas prescribed?
A. Amantadine
B. Bromocriptine
C Trihexyphenidyl
D.Levodopa (L-DOPA)

Before drug be approved we see the cost of safty and non safty which type of cost:-
A.Cost of illness
B.Cost of benefits
C. Cost of effectiveness
D.Cost of utility
*‫*ﻟﻠ&ٮﻮﺿيﺢ‬:
‫ ال‬:A ‫ ﺷﺎف ال‬،‫ٮﻪ‬2‫ ﻣﺎ ﺷﺎف )ڡﻌﺎﻟ‬Safety ، ‫ٮﻪ ﻣ&ڡﺎر)ٮﻪ ﻣﺎ )ٮ)ﺤ&ٮﺎرﻫﺎ‬2‫;ٮﺲ ﻋﺸﺎن )ڡ‬.
‫ ال‬: B ‫ٮﻪ اي ﳾ ﻃﱯ‬2‫ﺪه )ﺣﺪﻣﻪ زيﺎده ﻃ;ٮ‬2 ‫ٮ&ﺪ)ڡﻊ )ڡﻠﻮس ٮ&ﺄ)ﺣﺬ )ڡﺎٮ‬
‫ال‬: C ‫ﺸﻮف ﻫﻞ ﻫﻮ‬2 ‫ٮﻪ ﻣ&ڡﺎر)ٮﻪ ٮ‬2‫ ;ٮﺲ دام )ڡ‬،‫هﻢ ﺣﱴ ﻟﻮ ﰷن ا)ﻋﲆ‬2 ‫ٮﻪ ﻣﺎ ٮ‬2‫ٮﻦ وٮ&ﺎ)ﺣﺬ اﻻﻛ_ٮﺮ )ڡﻌﺎﻟ‬2‫ ﻣ&ڡﺎر)ٮﻪ ;ٮيﻦ دواﺋ‬safe ‫او ﻻ‬
‫)ڡ)ٮ)ﺤ&ٮﺎرﻫﺎ‬.
‫ ال‬:D ‫ٮﻒ )ٮﻌﺮف؟ اذا ٮ&ﺤﺴ)ٮﺖ ﺣﺎﻟﺔ اﻟﻤريﺾ ;ٮﺎﻣراض اﻟﻜرو)ٮﻚ‬2‫ ﻛ‬،‫ﺴ&ٮ)ﺤﺪﻣون ﻟﻠﻜرو)ٮﻚ‬2 ‫ٮ‬

Women newly get pregnant and preform diabetic test in the beginning of her pregnancy
negative, what is your recommendation regarding gestational diabetic test?
A)No need for testing
B) Do another test between 4 and 12 weeks
C) Do another test between week 20 and 24

*UGT1A1*28 ‫ﺶ‬2 ‫ٮﺮه ﻋﲆ اٮ‬2_‫? ٮ&ﺄٮ‬


a--increase irinotecan metabolism
b-- decrease irinotecan metabolism
‫ٮﺔ‬2‫ى ﰷ)ٮﺖ ﻣﺎدة ٮ_ﺎ)ٮ‬+& ‫اﻟ;ٮﺎڡ‬

How is (first) one to prevent medication error?


Pharmacist
Nurse
Doctor
patient

How is (Last) one to prevent medication error?


Pharmacist
Nurse
Doctor
Patient

Where to search for off label drug use :


-usp drug information
-hospital formulary

Skin fold test ‫ﻌﻤﻠﻮ؟‬2 ‫ٮﺶ ٮ‬2‫ٮ&ٮﻪ او ﻟ‬2‫ﺶ اﻫﻤ‬2 ‫ﻫﺬا اٮ‬
Pulging vasculature
Subcutaneous fat

A 55-year-old man brought a prescription for Oxycontin on April year. The script from doctor is
dated December 17 of last year prescription allows five refills. What quantity of fill (s) is\are likely
to be received by the patient?
A) 1 Today+1 Refill
B) 0 Today + 0 Refill
C) 1 Today + 2 Refill
D) 1 Today + 0 Refill

[Third parts:]

Interaction between warfarin and aspirin?


Protein binding displacement
Pt with hypothyroidism taking levothyroxine try to conceive and asked the pharmacist about this
drug ?
Continue levothyroxine
Stop levothyroxine
‫ٮﻪ‬2‫ ﻟﻮ )ڡ‬continue with higher dose ‫ٮ&)ﺤ&ٮﺎروﻫﺎ‬

Elderly with diabetes and another conditions has neuropathic pain?


Gabapentin

Which class from the following can cause diabetes?


Statin
ACEIs

Statin taking in bedtime?


Simvastatin
Atorvastatin

Least statin has drug-drug interaction?


Simvastatin
Atorvastatin
‫ٮﻪ اي واﺣﺪ ﻣﻦ ﻣ;ﺤﻤﻮﻋﺔ‬2‫ ﻟﻜﻦ ﻟﻮ )ڡ‬PPFR ‫ا)ﺣ&ٮﺎروه ﻻ)ٮﻪ ا&ڡﻞ ٮ&)ڡﺎﻋﻞ )ﺣﺎﺻﻪ ﻣﻊ اﻟ;ﺤريﺐ )ڡروت‬

Patient with depression and loss of appetite?


Mirtazapine
Bupropion

Chlorpromazine MOA?
Block of dopamine 2 receptor

Haloperidol side effect?


Extrapyramidal side effect

Mood stabilizer drugs for bipolar disorder ?


Lamotrigine
Phenytoin
Lithium
Mycophenolate stop before pregnancy?
Four weeks
Six weeks
‫ اﺳ;ٮوع ﻟﻠﺬﻛﺮ‬١٢‫ اﺳﺎ;ٮيﻊ ﻟﻼ)ٮﱺ و‬٦‫اﻟﻤ)ڡروض‬

Contrandication in pregnancy?
Finasteride ‫ٮﺮه‬2‫ٮﺮ )ﻋ‬2‫وﻛ_ٮ‬

Class B COPD treatment?


Albuterol
Fluticasone
Ipratropium (or tiotropium )

Anticoagulant that used just before delivery?


Unfractional heparin
Enoxaparin
Wafarin

Hygiene to lower risk of UTI in women?


Urination after sexual intercourse

Infant with psoriasis?


Topical hydrocortisone
Oral hydrocortisone

Rifampicin counseling points ?


‫ﻌﻤﻞ‬2 ‫ اﻟﲇ اﻋﺮ)ڡﻪ ٮ‬check for liver ‫ٮﻪ‬2‫ويﺎ)ﺣﺬ اﻟﺪوا ﻋﲆ ﻣﻌﺪه )ڡﺎﺿ‬

Zolpidem dose for women?


2.5
5
15
20
10 ‫ﻟﻮ &ڡﺎل ﻟﻠﺬﻛﺮ )ٮ)ﺤ&ٮﺎر‬
1.75 :‫ اﻣﺎ اﻻ)ٮﱺ‬، 3.5 :‫ﻟﻮ &ڡﺎل ا&ڡﻞ ;ﺣﺮﻋﻪ ;ٮ&ٮﻜون ﻟﻠﺬﻛﺮ‬

Patient taking codeine and he is cyp2d6 poor metabolizer?


-Codeine overdose side effect
-Insufficient relief of pain ‫كﻠﻬﻢ ﺻﺢ ;ٮﺲ اﻻﺻﺢ‬

Virus cell contain?


-Plasmid
-Peptidoglycan
-nucleic acid genome and protein capsid that cover the genome and lipid envelope

A Doctor noticed a new side effect for a drug, what study should he wrote?
Case cotrol
clinical trial

Hypersensitivity reaction type ll ?


IgG
IgM
IgE
IgA
‫ٮﻪ‬2‫ ﻟﻮ )ڡ‬IgG ‫ و‬IgM ‫ﻣﻊ ;ٮﻌﺾ ا)ﺣ&ٮﺎروﻫﻢ‬

First antihypertensive for adult:


Aliskiren
Clonidine
Doxazoxin
Amlodipine

Patient has acute migraine came to the emergency with a severe migraine headache what
medication will be used?
Sumatriptan

Considered first line therapy for the management of partial seizures?


Carbamazepine
Phenobarbital
Gabapentin
Primidone

Pigion dropping ?
(Cryptococcosis),
*Fungus called: cryptococcus*

Status epilepticus treatment second line if BDZ didn’t work ?


1.Fosphenytion
2.Levetiracetam
‫ ﻟﻮ‬bdz ‫ ﻣﺶ ﻣﻮ;ﺣﻮد )ٮ)ﺤ&ٮﺎر‬Ethoxamide

-patient Hyperthyroidism & antiarrhythmic


A. amiodarone
B. dronedarone

-The first one should be informed if medical error?


Safety gp
manager

the pharmacist offer handshake to the director,type of communication?


1-verbal
2-non verbal
3-physical

85)Which of the following is the most appropriate time to draw a plasma blood
86/sample for digoxin monitoring?
A) 2 hours post-dose
B)4 hours post-dose
C)5 hours post-dose
D)6 hours post-dos

/Which of the following tests should be


performed prior to infliximab?
A) HIV screening
B) Influenza screening
C) Hepatitis C screening
D)Tuberculosis screening

Which of the following conditions is treated with desmopressin?


A. psoriasis
B. endometriosis
C. nocturnal enuresis
D. attention deficit disorder

) A 68-year-old woman with a history of type-2 diabetes and dyslipidemia is


on atenolol and lisinopril. The doctor wants to start statin as her estimated 10-
year ASCVD risk is 10%
Which of the following would be the recommended station in (mg)?
A. Atoravastatin 10mg
B.Atoravastatin 40 mg
C. Simvastatin 10 mg
D. Simvastatin 80 mg

Which diuretic safe pregnant?


Thiazide

Antihypertensive produce tachycardia?


Hydralazine
Reserpine
Methyldopa
Atenolol

How to write enoxaparin prescription ?


a) clexane 80 mg subQ every 12hr
b) enoxaparein 80.0 subQ BID
c) enoxaparin 80.0 subcatanous BID
d) enoxaparin 80 mg subcatanous every 12 hr

119)Monitoring of Bactrim (sulfamethoxazole+trimethoprim) cbc


Because side effect are:
Hyperkalemia
decrease WBCs
decrease platelet count

-[Odds ratio in?


a) Case control
b) cohort
-Relative risk? cohort

: which one for treatment Migrain ?


a)Dihydroergotamine
b)Alpha ergomitrine

Tools used to exam macular degeneration and retinal tear?


A. Snellen chart
B. Retinoscope
C. Ishihara chart

* Doxorubicin mechanism?
A. Alkylating
B. Antimetabolite
C. Metallating
D. ANTHRACYCLINES

Which beta blocker less lipid soluble and less CNS side effect ?
- atenolol
- Propranolol
- metoprolol succinate

Drug used in operation surgery?


A. Enoxaparin
B. warfarin
C. Dabigatran
D. Unfractionated heparin

Vit C used to decrease?


a/viral infection severity
B/Viral infection duration

Natural plant for increase physical activity and reduce stress?


A/chamomile
B/ ginseng ‫اذا ;ﺣﺎ;ٮﻬﻢ ﺳوا )ٮ)ﺤ&ٮﺎر اﻻﺻﺢ‬
increase physical activity : is ginseng
reduce stress : is chamomile
Drug can cross placenta
A/insulin
B/heparin
C/ glyp
‫ٮﻪ‬2‫ ﻟﻮ )ڡ‬Amoxicillin ‫)ٮ)ﺤ&ٮﺎره‬

All patient with UTI what is the best treatment


A/cephalaxin
B/nitrofurantoin
C/TMP/SMX

patient with Crohn's disease has been on the maximum dose of budesonide but still
uncontrolled, presented to the ER with acute symptoms what to give?
A- IV cyclosporin
B- IV methylpred
C- Salazine

Class of bitrybixab?
A/antibiotic
B/corticosteroid
C/Antiplatelet

88-Which of the following is an indication for the use of indapamide?


A. Angina pectoris
B. Cardiac dysrhythmias
C. Essential hypertension
D. Acute myocardial infarction

0) Acute ischemic stroke treated with ?


a- Amilodepine
b– hydralazine

Pt with G6pD and has UTI?


A, meropenam
B, moxiflxacin
C, azithromycin
Which need to calculate CHAD VASC to calculate statin dose
A, pt wih stroke
B, pt with IM
C, pt with DM

To Prophylaxis from stroke Use:


Fibrinolytic
Anti platelets
Anticoagulant

Eye drops can used after opening ?


10 days
12 days
28 days
*21 ‫ٮﻪ‬2‫ ﻟﻮ )ڡ‬days ‫*)ٮ)ﺤ&ٮﺎره‬

2-A post menopause old woman is suffering from facial flushing and vaginal drying.
She has done hysterectomy procedure.Which drug of the following ?
should she use?
A-estrogen
B- Progesteron

Pt has crohn’s disease he takes Sulfasalazine


But the symptoms gets worse what is the best option to ?
A-Cyclosporin
B-Aetaminophen
C-Prednisolo

Patient with crohn disease he cannot take oral which IV is acceptable ?


A/cyclosporine
B/salazin

what is the leads compound?


a-leads to pharmacological activity
b-leads to modification in structure
What is direct agenic category of Isoternin ?
A/ c
B/d
C/x
‫ اﻟﺼﺢ‬Cyp 3A4 ‫ٮﺎر‬2‫;ٮﺲ ﻣﺎﰷن واﺿﺢ ;ٮﺎﻟ)ﺤ‬

Which of the following is most likely to cause extra para middle effect
Mesoridazine
Thioridazine
Haloperidol

3. HIV asymptomatic and CD 4= 500 ask about vaccination ? Hep A/ infulenza /


no vacciene / penomococcal

Patient come with acute pain from osteoarthritis which drug not used for this
patient
A. aspirin
B. ibuprofen
C. cortisone
D. acetaminophen

Which of the proteins below is a protease inhibitor controlling blood clotting?


a) Thrombin
b) Plasmin
c) Antithrombin
d) Tissue plasminogen activator

Which of the following determinants of health is a 'downstream' determinant?


a) Housing
b) Employment conditions
c) Age
d) Education

Patient has accident and take anti psychotic with Cimetidine for heart burn and have panic attack
s.e happened ?
A. cimetidine increase metabolism of drug
B. decrease metabolism
WhT is the change in pedia relative to adult ?
-high volume of distribution , long half life
-low volume of distribution , short half life
-high volume of distribution , short half life
-low volume of distribution, long half life

Which the following hormone suppress due to take oral contraception:


A) FSH
B) LH
C) GnRH

Which of the following parameters is the most appropriate for PQ's self-evaluation of the
effectiveness of levothyroxine therapy?
a. Increased energy
b- Weight loss
c- Improved vision
d- less frequent angina

Which of the following conditions associated with resting membrane potential :


A- Potassium and chloride outside the membrane and sodium inside
B- Potassium and sodium outside, chloride inside
C- Sodium and chloride outside, potassium inside
D- Sodium inside, chlorine and potassium outside

which of the following decreases the effect of phenytoin?


a. alcohol
b. chronic renal toxicity
c. mild renal impairment

Q- ISO 5 IS which ?
Anteroom, laminar flow , buffer area.
Q- ISO 7 IS which ?
Anteroom, laminar flow , buffer area .
Q- ISO 8 IS which ?
Anteroom , laminar flow , buffer area.
A patient (she was an elderly I think 75) has a seizure, a history of arrhythmia and
is diagnosed with depression. What's the best management ?
A- Bupropion
B- Paroxetine
C- imipramin

‫ دواء‬approved to stop ‫ﻣﺮض‬


‫ى ﺻﺤيﺢ ;ٮ)ٮﺴ;ٮﺔ ﻟﻠﻤﺮض؟‬+‫ﻌﺎﻟ;ﺤﻪ اي ﻣﻦ اﻟ&ٮﺎل‬2 ‫وﻟﻜﻦ ﻣﺎٮ‬
incidence decrase
prevlance decrase
prevlance increase
incidence increase

Which of the following anti-arrhythmic drugs can is considered a rate control drug (choices:
satalol, flecainide, amiodarone , propofenone)

Medication Regulations :
MOH
SFDA

which one cause iron deficiency?


Magnesium hydroxide ‫ٮﻤﺎن ا)ﺣ&ٮﺎرﻫﺎ‬2‫ﻣﺤﻤﺪ ﺳﻠ‬.‫د‬
B- Levothyroxine
C- Metformin
Laxative slow onset of action

minimum age limit for giving levocetrezine?


a. after 6 months
b. after 1 year
c. after 4 yrs

Virus infection increase in?


Lymphocyte
Nuitrophil
Basophil
‫ واذا &ڡﺎل‬drcrease ‫ )ٮ)ﺤ&ٮﺎر‬C
Which WBCs responsible for fighting virus infection?
A. lymphocytes
B. Basophils
C. Neutrophils
( note : Neutrophile: bacteria / Lymphocytes: viral / Monocyte: viral/fungi / Esophile: parasite and
inflammatory / Basophile: mediate immunity (histamine))

A 75 yr old women suffering from night bed wetting. Which is the best treatment?
a. darifenacin
b. Oxybutinin
c. Pelvic excercises
d. Tolterodine
A or B and why pls

A 34-year-old man has end stage renal failure and is awaiting a transplant.
Which of the following changes in pharmacokinetics should be considered when
planning drug therapy?
A. Decrease in glomerular filtration rate
B. Increase in active tubular secretion
C. Increase in albumin levels
D. Increase in renal metabolism

- Drug enhanced it’s absorption with ranitidine


A- lorazepam
B- Warfarin
C- Aspirin
D- Naproxen

Which of the following is not a barrier to community pharmacy's involvement in


the new public health agenda?
a) A propensity for pharmacists to focus on the biomedical model of health
b) The operation of community pharmacy in a retail environment
c) A lack of cooperation between pharmacists in the community pharmacy sector
d) The location of many community pharmacies

formulary in the hospital this is consider as ?


Remove Watermark Wondershare
PDFelement
A. Inter institutional
B. Intra institutional
C. Written communication
D. Non Verbal communication

function alpha 1-antitrypsin in lungs


A- work as surfactant
B- enhance alv exchange

Drug cause constipation?


a) Atorvastatin
b) Vrapamil
c) Nitrate

- A 45-year-old patient with atrial fibrillation is treated with amiodarone 400


mg/day and rivaroxaban 20 mg/day with the evening meal. He has concern about his drugs
routine monitoring.
Which of the following would be the best recommended regarding amiodarone
monitoring?
A. Slit-lamp examination at baseline and annually
B. Renal function tests at baseline and annually
C. Cardiac enzymes at baseline and annually
D. Liver function tests at baseline and every 6 months

The authority responsible for the equivalence of the pharmacy certificate coming from
external countries ?
A) Ministry of Education
B) Saudi Commission
C) SFDA
D) MOH

A 2 years old child has taken Hepatitis A vaccine and came to take MMR vaccine:
a) Should take MMR vaccine immediately
b) MMR vaccine should be delayed 3 months
c) MMR vaccine should be delayed 6 months
- Antidepressants have teratogenicity effect on pregnant w
A. SNRIs
B. SSRIs
C. MAOIs
D. TCAS

Who will sign the form that is sent to SFDA for investigation of a drug ?
A) Physician B) Head nurse
C) Investigator D) Chief pharmacist

case about COPD and he asked about the drug that affect the prognosis of the disease:
a) Salmeterol
b) Salbutamol
c) Corticosteroid
d) Ipratropium Br

Sever musel crump , abdominall pain ,nausea , vomiting EXAMINATION reveal hypoadrenal
function immediate ttt?
a) 100mg IV hydrocortisone
b) 5mg prednisolone orally
c) 5% dextrose then NS

Hfa for:
Aerosol
Capsule
Injection

Q)Which of the following best describe medication adherence:


A)Patient follow instructor of the healthcare provider which written and agreed
B)patient follow recommendation of the HCP regarding timing and dosage
medication

- Ciprofloxacin+metronidazole give to :
A- Mild croh’n disease
B- Moderate crohn disease
C- Sever crohn disease
D- Not use
Septic shock pateint unresponsive to fluid and
CKD what will you Give
A-vasopressin
B-Dopamine ‫ﻻن ﻋ)ٮﺪه ﻣﺸكﻠﻪ ;ٮﺎﻟكﲆ‬
C-Norepinephrine

G protein coupled receptor second messenger?


A. AP3
B. CGMP
C. CAMP

A 37-year-old mother breastfeeding her child has developed consistent high blood preassure.
The physician consults the pharmacy to recommend an antihypertensive to control her blood
pressure
Which of the following anti-hypertensive is most hazardous for the child?
A. Atenolol
B. Verapamil
C. Metoprolol
D. Propranolol

Avoidance of consuming tyramine containing food is recommended with which of the following
medications?
Warfarin
Linezolid
Prednisone
Cyclosporine

*Supplement potentiate antidiabetic drugs ?


Aloe vera
Chamomile
Echinacea

*Herbal product can potentiate the effect of oral antidiabetic?


Garlic
Ginseng
Green tea
Echinacea
‫ٮﻪ‬2‫ اذا ﻣﻮ ﻣﻮ;ﺣﻮده وﰷن )ڡ‬aloe vera ‫)ٮ)ﺤ&ٮﺎرﻫﺎ‬
‫ٮﻪ‬2‫ اذا ﻣﻮ ﻣﻮ;ﺣﻮده وﰷن )ڡ‬aloe vera ‫)ٮ)ﺤ&ٮﺎرﻫﺎ‬

Which of the following increase INR?


Garlic
Green tea
St. jonnswort

-Newly graduated pharmacist hired in multinational. what would be his access of care?
A. Loses access to public
B. Doesn’t lose access to public
C. Can get healthcare from certain private hospitals

-Health coverage for pharmacist employed in multinational drug company:


A.Loses access to public
B. Doesn't lose access to public
C.Can get healthcare from certain private hospitals

-Which of the following situation demonstrates a direct association pharmacoepidemiological


study?
A) The study reveals that it is not a false association
B) The regression provides misleading statistical evidence
C) The association between two variables is a result of another
D) The association between the two attributes is not through attributes

hat is the most common side effects of L-dopa that limit its adherence?
1) depression
2)Nausea and vomiting
3)dyskinesia
4)orthostatic hypotension

-Maximum dispensing for prescription of phenobarbital from outpatient departement


30 60 90
-Maximum dispensing for prescription of amphetamine from outpatient department
30 90 180
Benzodiazepines ، barbiturate ‫وﻛﺬﻟﻚ‬

Vancomycin trough level?


5-10
10-20
30-40
Target vancomycin trough is sever infection?
5-10
15-25
15-20

Hypoglycemia side effect?


Glyburide
Metformin
pioglitazone

contraindication in ACEI ?
bilateral renal artery stenosis

Particle size affect which of the following:


a) Uniformity
b) Homogeneity
c) Dissolution

Ramelteon MOA?
Melatonin agonist
Melatonin antagonist

-What is the principle of marketing option are?


Gain customer
Satisfaction
Increase sales
-Main purpose of marketing?
Sell cosmetic products

Gall bladder contraction by which hormone ?


cholecystokinin

Hormone mainly excreted from kidney ‫؟‬


a- hematopoietin b-erythropoietin
‫ٮﺎرات‬2‫ٮﻪ ﻟ)ﺤ;ٮﻄﻪ ;ٮ)ٮ&ڡﻞ اﻟ)ﺤ‬2‫ٮﺤﻪ ﻣﻤﻜﻦ )ڡ‬2‫كﻠﻬﻢ ﺻﺤ‬
‫ٮﺎرات‬2‫ٮﻪ ﻟ)ﺤ;ٮﻄﻪ ;ٮ)ٮ&ڡﻞ اﻟ)ﺤ‬2‫ٮﺤﻪ ﻣﻤﻜﻦ )ڡ‬2‫كﻠﻬﻢ ﺻﺤ‬

DM1 patient, give what?


Sulfonylurea
GLP1 (liraglutide)
DDP
Pioglitazone

Itraconazole capsulesrequireacidic pH optimal dossolution and absorption. Which is appropriate


advice to optimize the oral absorption of drug?
-Take it on empty stomach
-Take it with full glass of water
-Take it with food
-Take it with proton pump inhibitor

what is the conc w/w if 20 ml of water is required to dissolve 2g substance x?


9.09%
10.23%
8.60%
5.7

*Pt take spironolactone and diclofinac what is the result of drug drug interaction ?
Hypokalemia, hypernatremia
Hyperkalemia, hyponatremia
Hyperuricemia, hypercalceima
electrolytes will decrease or increase

‫ٮﻪ ﺳوال ﺣﻖ اﻻرﻣﺎ‬2‫ )ڡ‬wheezing cough ‫؟‬


B1 agonist
B2
Expect the phenytoin level foe pt with low albumin ?
High due to hypo albumin
Low duo to hypo albumin

Drug should decrease when* creatinine increase


- propranolol
- amlodipine
- Levtrectam

*Which of the following is the benefit from adding of -polyethylene glycol (PEG) in some
medications like peginterferon ?
To extend the half-life of medications
To extend the shelf-life of medications To exchange the effect
of the medications
To exchange the elimination of the medications
*Polyethylene glycolin PEG- interferon?
A Increase effect
B Increase solubility

●Bivalirudin monitor ?
( inr - aptt - factor xa-..)

●Enoxaparin patient monitor ?


( inr - factor x )‫ ﻣﺎﰷ)ٮﺖ ﻣﻮ;ﺣﻮده‬aptt

●Surgent want to know about DVT in which text book and ask a pharmacist ( DVT american 2013
update book - Prevention of DVT for orthopedic book - prevention of DVT for non orthopedic
book - book of parentral treatment )

Monophasic contraceptive ?
A) pills deliver the same amount of estrogen and progestin each day for 21 days.
B) deliver the same amount of estrogen each day, but the level of progestin is increased about
halfway through the pill cycle.
C) have 3 different doses of progestin and estrogen that change approximately every 7 days.

1)biphasic contraceptive ?
A) pills deliver the same amount of estrogen and progestin each day for 21 days.
B) deliver the same amount of estrogen each day, but the level of progestin is increased about
halfway through the pill cycle.
C) have 3 different doses of progestin and estrogen that change approximately every 7 days.

2) triphasic contraceptive ?
A) pills deliver the same amount of estrogen and progestin each day for 21 days.
B) deliver the same amount of estrogen each day, but the level of progestin is increased about
halfway through the pill cycle.
C) have 3 different doses of progestin and estrogen that change approximately every 7 days.

Which of the following is the most appropriate advice to give to asthmatic patients as non
pharmacological management?
A. Air purifiers will help ease the patients’ symptoms
B. OB Weight-loss is unlikely to affect their symptoms now
C. Patients should be trained on breathing exercise programs
D. OD.increase physical activity

Who sees pain four hours after eating ?


A) gastric
B) duodenal ulcer

Fixation
IgG or IgM

which of the following systems can be integrated with computerized


physician order entry to guide physician during prescribing medications ?
A) Smart plumbs
B) B. barcode systems
C) Electronic distribution cabinets
D) Clinical decision support system

Which of the following describe glucose uptake and usage by as compared


to other tissues of the body?
A) Brain cell can uptake and use glucose only in the presence
B) Brain cells can uptake and use glucose without being stimul insulin
C)Brain cells can uptake and use glucose when glucose is abol 100 mg/dL in
the blood
D) Brain cells can uptake and use glucose when the insulin is at high level in the
Body

- Which of the following statements provides the best describtic incremental cost-effectiveness
ratio?
A- The cost per extra benefit of a new strategy, independend treatment alternatives
B- The extra cost to obtain an extra benefit when switching alternative to another
C- The cost per quality-adjusted life year gained
D- A summary measurement of efficiency

Inflammatory acne need cleansing:


Hydrocortison 1%
Facial scrub
Mild soap clean
Hydoqunin

virus structure contain:


Cell membrane
Cell wall
Plasmid
Nucleic acid genome

Mycophenolate stop before pregnancy?


Four weeks
Six weeks

The most reliable study to make a clinical decisions?


Systematic review
Case control
Cohort study

Which of the following is the interpretation of a P-value of 0.05?


A) There is 0.05% probability that the result are due to random chance
B) There is 5% probability that the results äre due to random
C) There is 50% probability that the results are due to random
D) There is 95% probability that the results are due to random

Rotavirus contraindication ?
Latex
* Mmr CI with egg allergy*
* IPV CI with neomycin allergy*

which is abase or conjucated group?


phenolate
tertiary amine
quaternary amine

52- which need to calculate CHAD VASE, to calculate statin dose?


- Patient with stroke
- PT with MI
- Pt with DM
- pt with AFIB

Which of the following is volatile liquid?


1) Nicotine
2) Nitrogen

-Characteristic of iron deficiency ?


anemia
Glossitis
Angular stomatitis
Koilonichia

Patient with prolonged qt syndrome should avoid ?


Methadone
Amilodipine
Minocycline
Linzolide

-Drug used to trat hypertenstion and prophylaxis for headache?


A. Hydrlazine
B. phenytoin
C. Dapson
D. minoxidil

Live vaccine and Antibody administration?


A. Same day different sites ‫اﻟﻬواري‬.‫د‬
B.Separate 2weeks
C. Administer live vaccine then after 1 week antibody ‫ٮﻤﺎن‬2‫ﻣﺤﻤﺪ ﺳﻠ‬.‫د‬
D. Separate one month in different sites
*:‫*ﻟﻠ&ٮﻮﺿيﺢ‬
‫ٮﺎر‬2‫ ﺳ;ٮﺐ ا)ﺣ&ٮ‬C : [ If these live vaccines are administered first, it is necessary to wait at least 2 weeks
‫ٮﺎر‬2‫ ﺳ;ٮﺐ ا)ﺣ&ٮ‬C : [ If these live vaccines are administered first, it is necessary to wait at least 2 weeks
before administering the antibody].
*inactive+inactive*‫ٮﻦ ﺷﻬﺮ‬2‫ﻮﻣ‬2 ‫ﻮم ٮ‬2 ‫ٮﺔ ﻟﻠﻮ&ڡﺖ ٮ‬2‫)ڡﺼﻠﻬﺎ ;ٮﺪون اﻫﻤ‬2 ‫ٮهﺎ ;ٮ)ٮ)ڡﺲ اﻟﻮ&ڡﺖ او ٮ‬2‫ﻌﻄ‬2 ‫ٮ‬.
Live+inactive ‫;ٮﺮﺿﻮ )ٮ)ڡﺲ اﻟﴚ‬.
Live+live ‫ﻮم‬2 ‫ ٮ‬٢٨ ‫)ڡﺼﻞ‬2 ‫ٮﻮم ;ٮ)ٮ)ڡﺲ اﻟﻮ&ڡﺖ او ٮ‬2‫ٮﻪ ;ٮ)ٮ)ڡﺲ اﻟ‬2‫ﻌﻄ‬2 ‫ﺎ ٮ‬2 ‫ٮ‬.
Live+Antibody ‫ى‬+‫ اﻟﲇ ه‬immunoglobulin ‫ ﺷﻬور‬٣ ‫;ﺤﺐ ان ٮ&)ڡﺼﻞ اٮ&ﻮ&ڡﻊ‬2 ‫ٮ‬.

Patient with daily cough and chest tightness, with 3 – 4 night attacks per week, with limitations
in his daily activity, what is the classification of this asthma type?
A.Intermittent
B.Mild
C.Moderate persistent
D.Sever persisten

Which of the following large lymphoid organ? A- liver


B-spleen
C- muscle
D- intestine

Medication is an anti-androgen?
Danazol
Tamoxifen
Finasteride
Mifepristone
‫ وي&ڡﻮﻟﻮ اذا‬androgen ‫ )ٮ)ﺤ&ٮﺎر‬A

Doxorubicin mechanism?
Alkylating
Antimetabolite
Metalating
Anthracyclines

The patient didn't take the third dose of Heb B for long period? ‫ٮ&&ڡري;ٮًﺎ اﻟﺴؤال ﻛﺬا‬
A) give the dose
B) give the additional dose after the third dose C)repeat from the start
D) make a Test For Heb B to check if he needs to start again
When taking clozapine you should check?
CBC
‫ٮﻪ‬2‫ وﻟﻮ )ڡ‬ANC ‫اﺻﺢ )ٮ)ﺤ&ٮﺎرﻫﺎ‬

Duration of taking domperidone?


3-5 days (maximum 7 days)

Advantage of cohort study?


Less expensive

respiratory infection what type of IG will be high?


IGM
IGE
IGA
IGG

Mechanism of sitagliptin?
A Dipeptidyl Peptidase IV Inhibitor

Vasopressin extravasation?
Phentolamine

Moderate to sever ashthma ttt?


Omalzumab

How to give medication to breastfeeding mother when we know its toxic to new born?
Take the medication after breastfeeding.

Patient farmer have fever, weight loss and went to the pharmacy?
The pharmacy told him to go to the nearest hospital

Cyproheptadine is
A-antidepressent
b-anti psychotic
c-anti hypertensive
d- anti histamine
Anti-cancer act through mitotic inhibition?
Cyclophosphamide
MTX
Paclitaxel

5th generation ciphalsporin?


Ceftaroline

Ttt of C.difficile?
Metronidazole

Cromolyn MOA?
Mast cell stabilizer

Effect the grouth of fetuse ?


Atenolol

What increase effect of benzodiazepine?


Thiopental

No receptor mechanism?
Deferoxamine

Antihistamine for driver?


Desloratidine

MMR taken first dose?


12 month

DASH diet for?


Hepertenstion

Fentanyl patch counseling?


Prevent increase in body core heat

Advantage of automated dispensing system?


1-Adherence
2-drug quality
3-Decrease error

Mild dehydration for child


Normal saline
apple juice

Phase 4 ?
RMP
‫ﻟﻠ&ٮﻮﺿيﺢ‬:
Phase0: upstroke or rapid depolarization
Phase1: early rapid repolarization
Phase2: plateau
Phase3: final rapid repolarizaion
Phase4: resting membrane potential and diastolic depolarizaion

Boxed warning when using for teeth pain in infant lead to seizure and death?
Clove oil
Ibuprofen suspension
Dextrose and amino acid and lipid

Pseudomonas aerogenosa shape ?


gram negative rod shape

Desmopressin ttt for ?


nocturnal enuresis
biabetic insipidus

Role of CH3 in ibuprofen?


Increase selectivity on cox1 receptor

Refill of narcotic- psychtropic medication valid for?


A- Zero ‫اﺣﺴﻬﺎ ا&ڡﺮب ﳾ‬
B- One day
C- Tow days
D- Three days
according to new evidence what type of Hepatitis did they discover treatment for?
-Hep A
-B
-C
-D

Exacerbate digoxine toxicity?


HypoMg

Antiemetic?
ginger
Epeca

Cyclosporine cause?
Hemolytic cystitis/Hemorrhagic cystitis

Hydrophilic antibiotic?
floroquinolons
polymyxin
rifampin
tetracycline

How to avoid LASA ?


Tall man letters

Brown urine ?
Nitrofurantoin

Colistin affect on? (Renal toxicity)


Kidney

Which of the following is the test of lodine ?


A- starch and glycogen
B- monosaccharide and disaccharide
C - maltose and lactose
D- fructose and maltose
If part of solute needed 10000 parts of solvent to be dissolved, it is prescribed as:
Freely soluble
Soluble
Sparingly soluble
Practically insoluble

what is the maximum volume if injection in glutes ?


1
2
3
5

Which of the following may cause optic neuritis?


Isoniazid
Rifampicin
Pyrazinamide
Ethambutol

[Fourth parts]

(‫ٮﺔ ذي ٮ&ﺄﻛﺪوا ﻣﻦ أ;ﺣو;ٮ&ٮﻬﺎ‬2‫ )اﻟ;ﺤﺰﺋ‬:

1/ Case pt not need to dely live vaccine


-Pt recently done from chemo
-Pt ended course of steroids ‫اٮ&ﻮ&ڡﻊ‬

2/ Type II glass containers are?


A.Most inert Glasses and shows high Hydrolytic Resistance
B.Suitable for most acidic and neutral aqueous preparations
C.Suitable for Alkaline solutions
D.Suitable for non-aqueous preparations

3/-Largest organ in the human body?


Brain
Liver
Skin

4/ Which one of the following statements accurately explain the idea of


‟Diffusion of innovation therapy for health promotion?
A)Psychological process of creativity
B)The way how people develop new idea
C)How new ideas are controlled by a culture
D)The method of disseminating innovative ideas a through a culture

5/ Narcotic IV still effective till ?


A. 12 Hours ‫اٮ&ﻮ&ڡﻊ ذي اﻷﺻﺢ‬
B. 24 Hours
C. 36 Hours
D. 72 Hours
*‫ وﻟﻮ ﻛ&ٮﺐ )ڡﺎﻟﺴؤال‬،‫ ﺳﺎﻋﻪ اذا ﻛﺬا‬١٢ ‫&ڡﻮﻟﻮ‬2 ‫ ٮ‬mixtur ‫ ﺳﺎﻋﻪ‬٢٤ ‫*)ٮ)ﺤ&ٮﺎر‬

6/ What is the Phenothiazine mechanism of action?


A.Dopamine Antagonist
b.Dopamine Agonist
C.Sertonin Reuptake inhibit

7/-Which of the following is the interpretation of the resut of astatistical test denoted P?
A)the null hypothesis HO is rejected if P> 0.05
B) the null hypothesis HO is accepted if P < 0.05
C) the null hypothesis H0 is rejected if p <0.05
D)the null hypothesis HO is accepted if p> 0.05

8/-Which of the following is the interpretation of a P-value of 0.05?


A) There is 0.05% probability that the result are due to random chance
B) There is 5% probability that the results äre due to random
C) There is 50% probability that the results are due to random
D) There is 95% probability that the results are due to random
9/ What is the effect of smoking on theophylline?
A.Inhibition of CYP3A4
B. Inducer of CYP1A2
C. No effect
D.Decrease excretion of theophylline

10/-Maximum day supply a pharmacist can dispense for diazepam prescription?


7 days
30 days
* Dispensed: 30 / Valid: 7 * ‫ﻟﻠ&ٮﻮﺿيﺢ‬
‫ ﻟﻜﻦ ﻣﺪة اﻟﺼﺮف ﺷﻬﺮ‬،‫ﺎم‬2 ‫ اٮ‬٧ ‫ٮﺔ اﻟﻮﺻ)ڡﻪ‬2‫ﺻﻼﺣ‬

11/ ‫)ٮ;ٮ&ٮﺔ ٮ&ﺴ&ٮ)ﺤﺪم ﻟﻌﻼج‬


upper respiratory tract infection?
-echinchae ‫اﻟﲇ ٮ&ﻌﺎﻟﺞ اﻟﺴﻌﺎل اٮ&ﻮ&ڡﻊ‬

12/
-Bajlett test ⸺> Digitals
-Dry Cough —> Thyme
-Dragengrof test :Alkaloids
-PH of stomach: 1.5 to 3.5

13/ Fingolimod
‫ﺶ‬2 ‫ اٮ‬test ‫اﻟﲇ اﻟﻤ)ڡروض )ٮﺴويﻪ &ڡ;ٮﻞ ﻣﺎ )ٮﺼﺮ)ڡﻪ؟‬
Echocardiograph ‫اٮ&ﻮ&ڡﻊ‬
Sensitivity test

14- ‫ﺴ&ٮ)ﺤﺪم‬2 ‫ٮﺎء ;ٮﺲ ٮ‬2‫ٮﺮ او اﺷ‬2‫ ﺳؤال ﻣريﺾ ﻋ)ٮﺪه ا)ٮﻪ ;ٮواﺳ‬lidocaine ‫و;ﺣﺎء ﻟﻪ ٮ&ﺤﺴﺲ ﻣ)ٮﻪ وش )ٮ)ﺤ&ٮﺎر؟‬
Pramoxine

15-‫)ٮ&ڡﻞ دواء ﻣﻦ ﻣﲀن ﻣﻌ&ڡﻢ إﻟﻰ ﻣﲀن ﻣﻌ&ڡﻢ ;ٮﺎداة ﻣﻌ&ڡﻤﻪ وﺳﺄل ﻋﻦ اﻟريﺴﻚ؟‬2 ‫ﺳؤال اﻟﲇ ﻣريﺾ ٮ‬
low risk level

16-‫ﺴ;ٮﺐ ويﻮﻟﺪ )ٮ;ٮﻀﺎت اﻟ&ڡﻠﺐ؟‬2 ‫ٮﻦ ٮ‬2‫ﺳؤال ﻣ‬


SA node
AV node
‫وﺳﺄل اٮ ‪2‬ﺶ آﻟ‪2‬ٮﺔ ﻋﻤﻠﻪ؟ ‪ prednisone‬ﺳؤال‪17-‬‬
‫‪anti-inflammatory‬‬

‫ﺳؤال ﻣ‪2‬ٮﻦ اﻟﲇ ٮ ‪2‬ﺤول‪18-‬‬


‫‪?pepsinogen to pepsin‬‬
‫‪-‬‬ ‫)‪Hydrochloric acid (HCl‬‬
‫*‪*Gastric acid = HCL‬‬

‫ﻟ&ڡ‪2‬ٮﺎس اﻟ‪r‬ﻌﺪة اﻟﺪر&ڡ‪2‬ٮﺔ ‪ gold standerd‬ﺳؤال‪19-‬‬


‫‪TSH‬‬
‫;ٮﺮﺿﻮ ٮ&ﻌ&ٮ;ٮﺮ ﺻﺤ‪2‬ٮﺤﻪ ‪ COPD‬وﻟﻮ )ڡ‪2‬ٮﻪ‬

‫‪ OSTEOPOROSIS AND HEART BURN‬ﺷ)ﺤﺺ ﻋ)ٮﺪه‪20-‬‬


‫‪Calcium carbonate‬‬

‫؟ ‪ Starch‬ﺳؤال ﻣ‪2‬ٮﻦ اﻟﲇ ٮ ‪2‬ﻜﺴﺮ ال‪21-‬‬


‫‪Amylase‬‬

‫اٮ ‪2‬ﺶ )ٮﻌﺎﻟ;ﺤﻬﺎ؟ ‪ red man syndrome‬ﺳؤال ٮ&;ٮﻊ اﻟ)ڡﺎ)ٮﻜﻮﻣﺎٮ ‪2‬ﺴ‪2‬ٮﻦ و&ڡﺎل ا)ٮﻪ ٮ ‪2‬ﺴ;ٮﺐ‪22/-‬‬
‫‪slow rate of vancomycin infusions‬‬

‫)ڡ&ڡﻂ وا)ٮﻪ ﻫﺬا اٮ ‪2‬ﺶ ٮ ‪)2‬ﺤﺺ‪ /‬ٮ ‪&2‬ٮﻌﺎﻣﻞ ﻣﻊ ؟ ‪ relatives‬ﺳؤال ا)ٮﻪ ﺷ)ﺤﺺ ٮ ‪&2‬ٮﻌﺎﻟﺞ ﻣﻊ ﺣ&ڡ&ٮﻪ‪23/-‬‬
‫‪justice‬‬

‫؟‪ agranulocytosus‬ﺳؤال ﻣﻦ اﻟﲇ ٮ ‪2‬ﺴ;ٮﺐ‪24/-‬‬


‫‪PTU‬‬

‫؟‪ Hypothyroidism‬ﺳؤال اٮ ‪2‬ﺶ &ڡ‪2‬ٮﺎس‪25/‬‬


‫‪elevated tsh and decreased t4‬‬

‫ﻋﲆ ﻣﺎ اذﻛﺮ واٮ ‪2‬ﺶ اﻟ)ٮ;ٮ&ٮﻪ اﻟﲇ ٮ&ﻌﺎﻟ;ﺤﻪ؟ ‪ cerebral blood flow‬ﻣريﺾ ﻋ)ٮﺪه اﻋراض ﻛ_ٮ‪2‬ٮﺮه وﻣﻦ ﺿﻤ)ٮﻬﺎ )ٮ&ڡﺺ‪26/-‬‬
‫‪ginkgo‬‬

‫!‪ saw balmetto‬ا;ﺣﺎء ﻻٮ ‪2‬ﺶ ٮ&ﺴ&ٮ)ﺤﺪم‪27/‬‬


‫‪PBH‬‬

‫زي ﻛﺬا ٮ&&ڡري;ٮًﺎ وﰷن )ڡ‪+‬ى أر;ٮﻌﺔ )ﺣ‪2‬ٮﺎرات ﻣﻦ ﺿﻤ)ٮﻬﻢ ‪ ACEI‬ﺳوال ﻣ‪2‬ٮﻦ اﻟ)ڡﺌﺔ اﻟﻤ)ڡروض ﻣﺎ )ٮﻌﻄ‪2‬ٮهﺎ‪28/-‬‬
‫زي ﻛﺬا ٮ&&ڡري;ٮًﺎ وﰷن )ڡ‪+‬ى أر;ٮﻌﺔ )ﺣ‪2‬ٮﺎرات ﻣﻦ ﺿﻤ)ٮﻬﻢ ‪ ACEI‬ﺳوال ﻣ‪2‬ٮﻦ اﻟ)ڡﺌﺔ اﻟﻤ)ڡروض ﻣﺎ )ٮﻌﻄ‪2‬ٮهﺎ‪28/-‬‬
‫‪Bilateral artery setenosis‬‬

‫اﻟﻤ)ڡروض ﻟﻤﺪه ﻛﻢ ‪ zidovidone‬ﺳوال ﻋﻼج ﻣﺮه ٮ&ﺎ)ﺣﺬ‪29/‬‬


‫‪4-6 weeks‬‬

‫؟ ‪ otitis media‬ﺳؤال ﻃ)ڡﻞ ﺻ‪r‬ﻌ‪2‬ٮﺮ وﻋ)ٮﺪه‪30/-‬‬


‫‪high dose of amoxicillin‬‬
‫‪Azythromycin‬‬

‫ﺳؤال ﻣريﺾ ;ٮيﺎ)ﺣﺪ ا&ڡراص )ڡ‪2‬ٮ)ٮ‪2‬ٮ&ٮويﻦ ‪ 100‬ﻣﻠ;ﺤﻢ ‪ 3‬ﻣرات ٮ ‪2‬ﻮﻣ‪2‬ٮﺎ واﻟﺪﻛ&ٮور ﻫ‪2‬ٮﺤﻮﻟﻪ ﻟﺸراب ٮ&ﺮﻛ‪2‬ٮﺰه ‪10‬ﻣﻠ;ﺤﻢ‪ 5/‬ﻣﻞ‪31/-،‬‬
‫ﻫ‪2‬ٮﺎ)ﺣﺪ اد اٮ ‪2‬ﻪ ﻣﻦ اﻟﺸﺮب ٮ ‪2‬ﻮﻣ‪2‬ٮﺎ؟‬
‫ﻣﻠ;ﺤﻢ ‪400‬‬
‫ﻣﻠ;ﺤﻢ ‪200‬‬
‫ﻣﻠ;ﺤﻢ ‪300‬‬
‫ﻣﻠ;ﺤﻢ ‪500‬‬

‫؟‪ MMR‬و;ﺣﺎي ٮ ‪2‬ﺎ)ﺣﺪ ‪ heb a immunoglobin‬ﺳؤال اﻟﻄ)ڡﻞ اﻟﲇ ا)ﺣﺬ‪32/-‬‬


‫)ٮﻌﻄ‪2‬ٮﻪ ﻣ;ٮﺎﺷﺮة‬
‫ٮ ‪)2‬ٮ&ٮ)ﻄﺮ ‪ 3‬اﺷﻬﺮ‬
‫اﺷﻬﺮ ‪6‬‬
‫اﺷﻬﺮ ‪9‬‬
‫*ﻟﻠ&ٮﻮﺿيﺢ‪ :‬ﻻزم ;ٮ)ٮ)ڡﺲ اﻟ‪2‬ٮﻮم ;ٮﻤﲀ)ٮ‪2‬ٮﻦ ﻣ)ﺤ&ٮﻠ)ڡ‪2‬ٮﻦ ;ٮﺎﻟ;ﺤﺴﻢ واذا ٮ&ﺄ)ﺣﺮ ٮ ‪2‬ﻮم ٮ ‪)2‬ﺤﲇ )ڡﺎﺻﻞ ‪ ٢٨‬ٮ ‪2‬ﻮم*‬

‫ﻫﻮ ٮ ‪2‬ﺴ&ٮ)ﺤﺪم اﺳ&ٮ)ﺤﺪام ‪ promethazine‬ﺳؤال‪33/-‬‬


‫‪antihistaminic‬‬
‫‪antipsychtic‬‬

‫ﺳؤال اي واﺣﺪ ٮ ‪&2‬ٮﺎ)ﺣﺬ ﻣﻊ اﻟﲀ)ٮﺴﺮ اﻟﲇ ٮ ‪2‬ﻜون ﺻﻠﺐ ‪34/‬‬


‫?‪Used for solid tumers‬‬
‫‪Dostarlimab‬‬ ‫‪Entrectinib‬‬ ‫‪pembrolizumab‬‬ ‫اﻟﲇ ;ﺣﺎ‬
‫كﻠﻬﻢ ﺻﺢ‬

‫؟‪ hyperkalemia‬ﺳؤال ;ﺣﺎل‪+‬ى واﺣﺪ‪35/‬‬


‫‪ca gluconate‬‬
‫‪Na carbonate‬‬
‫‪Ca resin‬‬
‫؟‪ influenza‬ﻟﻠﺤﺎﻣﻞ ٮ&ﺎ)ﺣﺪه ﻣﻊ كﻞ ﺣﻤﻞ )ﻋ‪2‬ٮﺮ ‪ vaccine‬ﺳؤال ال‪36/‬‬
‫‪- Tdap‬‬

‫وﻣﺶ ﻣ&ٮﻄﻌﻤﺔ؟ ‪ hep b‬ﺳؤال ﻣﻤﺮﺿﺔ ٮ&ﻌﺮﺿﺖ ﻟﺪم ﻣريﺾ‪37/-‬‬


‫‪- ig + vaccination‬‬

‫ٮ ‪2‬ﺎ)ﺣﺬ اٮ ‪2‬ﺶ؟ ‪ nausea‬و;ﺣﺎﻟﻪ ‪ rivastigmine‬ﺳؤال اﻟﺮ;ﺣﻞ اﻟﲇ ٮ ‪2‬ﺎ)ﺣﺪ‪38/‬‬


‫‪donepezil‬‬
‫‪Galantamine‬‬
‫‪Memantine‬‬
‫‪Tacrine‬‬

‫؟‪ Mast cell activator‬ﺳؤال اي اﻟﻌﻼ;ﺣﺎت‪39/‬‬


‫‪-platelet activating factor‬‬

‫؟ )‪ KOH (potassium hydroxide‬ﺳؤال ال‪40/‬‬


‫‪Lead‬‬

‫و;ٮي&ٮﻌﺐ و&ڡﺖ اﻟﺤﻤﺎم وﻟﻮﻣﺎ ٮ ‪2‬ﻤﺸﻂ ﺷﻌﺮه وﻟﻮﻣﺎ ٮ ‪)2‬ٮﺎم ٮ ‪2‬ﺤﻂ ﻣ)ﺤﺪٮ&‪2‬ٮﻦ ٮ&ﺤ&ٮﻪ وي&ٮﻌﺐ ‪ angina‬ﺳؤال اﻋراض ﺷ)ﺤﺺ ﻋ)ٮﺪه‪41/-‬‬
‫‪ attack‬ﺣﱴ و&ڡﺖ اﻟراﺣﻪ ٮ ‪2‬ﻤﻜﻦ ٮ ‪;2‬ﺤ‪2‬ٮﻪ‬
‫؟ ‪angina‬‬
‫‪Class i‬‬
‫‪Class ii‬‬
‫‪Class iii‬‬
‫‪Class iv‬‬

‫‪ alkylating:‬ﻣﻦ ﻋﺎﺋﻠﺔ ‪; anticancer‬ﺣﺎب ﻋﻼج‪42/-‬‬


‫‪cyclophosphamide‬‬

‫وﺣﺪه ﻋ)ٮﺪﻫﺎ ﺿ‪r‬ﻌﻂ وﻣﺸﺎكﻞ أ)ﺣرى ‪; case‬ﺣﺎب‪43/‬‬


‫‪ life style modifications‬ﻋﻤﺮﻫﺎ ‪ 61‬ﺣﺎوﻟﺖ ٮ&ﻌﻤﻞ‬
‫ﻟﻬﺎ؟ ‪ management‬ﻣ)ٮﺬ اﺳ;ٮﻮﻋ‪2‬ٮﻦ وﻣﺎ )ٮ)ڡﻌﻬﺎ ﺿ‪r‬ﻌﻄﻬﺎ اٮ ‪2‬ﺶ ا)ٮﺴﺐ‬
‫‪thiazide‬‬
‫‪Amlodipine‬‬ ‫ﻻ)ٮﻪ ا)ڡﻀﻞ ﳾ ﻟﻜ;ٮﺎر اﻟﺴﻦ‬

‫ﺳؤال اﻋراض وا)ﺣ&ٮ)ٮﺎق وأﺷ‪2‬ٮﺎء ﻛ_ٮ‪2‬ٮﺮة و;ﺣﺎب )ﺣ‪2‬ٮﺎرات ﻣﻦ ﺿﻤ)ٮﻬﺎ؟ ‪44/-‬‬


‫ﺳؤال اﻋراض وا)ﺣ&ٮ)ٮﺎق وأﺷ‪2‬ٮﺎء ﻛ_ٮ‪2‬ٮﺮة و;ﺣﺎب )ﺣ‪2‬ٮﺎرات ﻣﻦ ﺿﻤ)ٮﻬﺎ؟ ‪44/-‬‬
‫‪Pneumonia‬‬
‫‪COPD‬‬

‫‪ copd:‬ﻃ;ٮﻌﺎ )ڡ‪+‬ى اﻟﺼوره ﰷٮ&ﺐ ﻋﻼج ‪ copd،‬وﺳﺄل اٮ ‪2‬ﺶ ﻋﻼج ال ‪; copd‬ﺣﺎب ﺻوره ﻣﻦ ﻣﺮ;ﺣﻊ )ڡ‪2‬ٮهﺎ كﻼم ﺷرح ﻋﻼ;ﺣﺎت‪45/-‬‬
‫‪-Daily inhaled corticosteroids and saba as need‬‬ ‫ﻣﻮ ﻣ&ٮﺎﻛﺪه اذا ذي‬
‫‪-Daily/ inhaled corticosteroids and laba‬‬ ‫‪ LABA‬و;ٮﺮﺿﻮ ذي )ڡ‪2‬ٮﻪ وﺣﺪه &ڡﺎﻟﺖ ﻋ)ٮﻬﺎ ﺻﺢ اﻟﲇ ﻣﻊ‬

‫?‪-Which of the following acts as beta blocker and partial agonist‬‬


‫‪pindolol‬‬
‫‪ :‬اٮ&ﻮ&ڡﻊ اﻟ)ﺤ‪2‬ٮﺎرات ﻛﺬا ٮ&&ڡري;ٮًﺎ أو‬
‫ﻣﺴ&ٮ;ٮﻌﺪ‪1....‬‬
‫ﻣﺴ&ٮ;ٮﻌﺪ‪2....‬‬
‫‪3-inhaled corticosteroids and saba as need‬‬ ‫ﻫﻞ ﺻﺤ‪2‬ٮﺤﻪ؟‬
‫‪4-Saba daily and corticosteroids as needed‬‬

‫اٮ ‪2‬ﺶ ﻣ)ڡﻌﻮﻟﻪ ;ٮيﻜون؟ ‪ phenylephrine‬ﻟﻮﻣﺎ ٮ ‪2‬ﺤﺼﻞ ﻟﻪ ٮ&)ڡﺎﻋﻞ وي&ٮﺤول إﻟﻰ ‪ epinephrine‬ﺳؤال ال‪46/‬‬
‫‪increase alpha 1 selectivity‬‬

‫‪:‬ﺳؤال ﻛ‪2‬ٮﻒ ٮ&ﺴﺎﻋﺪي ﺷ)ﺤﺺ ٮ ‪)2‬ٮﺎم ;ٮﺸكﻞ ﻛويﺲ ويﻀ;ٮﻂ )ٮﻮﻣﻪ و)ڡ‪+‬ى )ﺣ‪2‬ٮﺎرات ﻣﻦ ﺿﻤ)ٮﻬﺎ‪47/-‬‬
‫ٮ ‪2‬ﺄكﻞ ﺷﻮكﻼٮ&ﻪ &ڡ;ٮﻞ ﻣﺎ ٮ ‪2‬روح ٮ ‪)2‬ٮﺎم ‪-‬‬
‫او ٮ ‪2‬ﺸﺮب ﻣويﻪ ﻛ_ٮ‪2‬ٮﺮ &ڡ;ٮﻞ ﻣﺎ ٮ ‪2‬روح ٮ ‪)2‬ٮﺎم ﻫﻮ ٮ&ﻤﺎم ;ٮﺲ ٮ ‪2‬ﺸﺮب ﻣﺎء زيﺎده ﻋﻦ اﻟﻠزوم ﻣﺎ ا)ﺣ&ٮﺮٮ&ﻪ‪-‬‬
‫اﻟ)ﺤ‪2‬ٮﺎر اﻟ_ٮﺎﻟﺚ &ڡﺎل ٮ ‪2‬روح ٮ ‪)2‬ٮﺎم ويﺴ&ٮ‪2‬ٮ&ڡﻆ ;ٮ)ٮ)ڡﺲ اﻟﻤ‪2‬ٮﻌﺎد‪-‬‬

‫;ٮ&ٮﺮﻛﺰ ﻋﲆ اٮ ‪2‬ﺶ؟ ‪phase 3‬ﺳﺄل ال‪48/‬‬


‫‪Efficacy & and therapeutic range‬‬
‫‪Efficacy. safety and therapeutic range‬‬

‫;ﺣﺎٮ ‪2‬ﺐ اٮ ‪2‬ﻪ اﻟﲇ ;ٮي&ڡﻠﻞ ٮ&ﺄٮ_‪2‬ٮﺮ اﻟوار)ڡﺎريﻦ و اﻻ)ﺣ&ٮ‪2‬ٮﺎرات‪49/‬‬


‫‪St John warts‬‬
‫‪Rifampcin‬‬ ‫اﻻٮ_)ٮ‪2‬ٮﻦ ﺻﺢ ;ٮﺲ ;ٮي&ڡﻮﻟﻮ ﻫﺬا أ&ڡوى‬

‫ﺣﻖ اﻟﻌﻼ;ﺣﺎت؟ ‪ Emergency cart cabinet‬اٮ ‪2‬ﺶ )ڡﺎٮ ‪2‬ﺪه ‪50/‬‬


‫‪Allow timely providing emergency drugs‬‬

‫؟ ‪ skin‬اٮ ‪2‬ﺶ )ٮوع اﻟ;ٮﻜ&ٮ‪2‬ٮريﺎ اﻟﲇ ع ‪51/‬‬


‫‪staphylococcus epidermis‬‬
‫؟ ٮ&ﺴ;ٮﺐ اﻟ&ٮﻬﺎب ﻟوز‪52/ Bacteria cause Tonsillitis‬‬
‫‪Streptococcus pyogenes‬‬

‫ٮ ‪2‬ﻌ&ٮ;ٮﺮ ;ﺣﻬﺎز ﻣ)ٮﺎع‪+‬ى ٮ_ﺎ)ٮوي ؟?‪53/ a secondary immune system‬‬


‫‪Tonsillitis‬‬ ‫اﻟﻠوز‬

‫وﻣﺮض ;ﺣﻠﺪي اٮ ‪2‬ﺶ ٮ ‪2‬ﻌﻄﻮ)ٮﻪ؟ );ﺣﻠﺪه ;ﺣﺎف( ‪ dry skin‬واﺣﺪ ﻋ)ٮﺪه ‪54/‬‬
‫‪Cream‬‬
‫‪Lotion‬‬
‫‪Ointment‬‬
‫‪Sloution‬‬

‫؟ اﻛ;ٮﺮ ﻋﻀﻮ‪55/The largest organ in the body‬‬


‫‪skin‬‬
‫‪liver‬‬
‫‪blood‬‬
‫‪brain‬‬
‫؟‪ the largest lymphoid organ‬اﻣﺎ ﻟﻮ &ڡﺎل‬
‫‪spleen‬‬

‫ﻣريﺾ ﻋ)ٮﺪه ﺳﻜﺮ وﺿ‪r‬ﻌﻂ و&ڡﻠﺐ وكﻞ ﳾ ويﺎ)ﺣﺬ وار)ڡﺎريﻦ وﻣ‪2‬ٮ&ٮ)ڡورﻣ‪2‬ٮﻦ و&ڡﻠ‪2‬ٮ;ٮزاٮ ‪2‬ﺪ وﻣﻠ&ٮ‪2‬ٮ)ڡ‪2‬ٮ&ٮﺎﻣ‪2‬ٮﻦ ٮ&ﺤﺎﻟ‪2‬ٮﻠﻪ كﻠﻬﺎ را)ڡﻌﻪ‪56/‬‬
‫ﻣﻌﺎدن وﻛ‪2‬ٮريﺎٮ&‪2‬ٮﻦ‪) ،‬ٮﻮ&ڡﻒ أي دواء؟‬
‫‪ Glipizide‬اﻻﺻﺢ‬ ‫ﻻ)ٮﻪ ﻣﻤ)ٮوع ﻟﻤﺮﴇ اﻟ&ڡﻠﺐ‬

‫اﻻﻣراض اﻟﻤﺴ&ٮﻬﺪف ‪57/ 2030‬‬


‫;ٮي;ﺤ‪2‬ٮﺐ اٮ_)ٮ‪2‬ٮﻦ ويﻄﻠﺐ اﻟ_ٮﺎﻟ_ٮﻪ ‪),‬اﻣراض اﻟ&ڡﻠﺐ واﻟﺴﻜري واﻟﺴﺮﻃﺎن(‬

‫؟ ‪ pharmacokinetics‬ﺳؤال ﻋﻦ‪58/‬‬
‫‪- zero order‬‬
‫‪- first order‬‬

‫‪ :‬ﺷ)ﺤﺺ اٮ&ﺴﻤﻢ ;ٮﺴﻢ )ڡﺌران وﺣﺼﻞ اﻋراض ﻛﺬا و ﻛﺬا ﻣ)ڡ‪2‬ٮﺶ )ڡ‪2‬ٮهﺎ )ٮزيﻒ واﻻ)ﺣ&ٮ‪2‬ٮﺎرات ﰷن )ڡ‪2‬ٮهﺎ‪59/‬‬
‫‪Warfarin‬‬
‫‪Atropine‬‬
‫‪Aluminium phosphide‬‬

‫‪):‬ڡ‪2‬ٮﻪ ﺳؤال ﻣﺸﺎ;ٮﻪ ﻟﻪ اﻟﲇ ﻫﻮ‬


‫ٮﻪ ﺳؤال ﻣﺸﺎ;ٮﻪ ﻟﻪ اﻟﲇ ﻫﻮ‬2‫)ڡ‬:
-About lady ingested a rodenticide in which lab reports were given. Serum potassium seems to
be low. Which among is possible?
Zinc phosphide
Aluminium phosphide
Aluminium hydroxide
Barium carbonate

60/‫ﺎ)ﺣﺬ‬2 ‫ٮﺲ ﻟواﺣﺪ ٮ‬2‫ﺶ ٮ&&ڡ‬2 ‫ اٮ‬Saitagliptin‫؟‬


Live
Creatin clearance

61/‫ﺶ؟ ٮ&&ڡري;ٮﺎ ﻛﺬا‬2 ‫ﻌ&ٮﻤﺪ ﻋﲆ اٮ‬2 ‫ﺪ ٮ‬2 ‫ﺪ ا)ڡﻜﺖ ﻟﺪواء ;ﺣﺪٮ‬2 ‫ﺳﺎٮ‬
Potency
Effenty
Affectiy
Thrputic range

62/‫ٮﺲ ﻣﺴ&ٮوى اﻟﺴﻜﺮ ;ٮﻌﺪ اﻷكﻞ ﻋﺸﺎن )ٮﺸﻮف اذا ;ٮ)ٮﻌﺪل‬2‫ ﻣﱴ )ٮ&ڡ‬aspart dose ‫؟‬
15 min
30 min
1h
2h

63/
Aminoglucosid ‫ﻌ&ٮ;ٮﺮ؟‬2 ‫وش ٮ‬
Time depnded
Conc depede

64/ ‫ﻄﻠﻊ ﻟﻤﺎ )ٮﺤﻂ اﻟ)ڡوار ﻣﻊ اﻟﻤويﺎ؟‬2 ‫ﻌﺎز اﻟﲇ ٮ‬r‫ اﻟ‬Alkaloid
Co2
Oxygen

65/ ‫ ﻣريﺾ ﻋ)ٮﺪه‬meningitis ‫ٮﻪ ﻋﺸﺎن )ٮ&ڡﻠﻞ ال‬2‫ﺶ )ٮﻌﻄ‬2 ‫ اٮ‬complications ‫؟‬
Cortison

66/ ‫ى ال‬+‫ٮﻦ ا&ڡوى )ڡ‬2‫ ﻣ‬evidence ‫؟‬


Systemic review
RCT
Cross sectional
Cohort

67/ ‫ ﻣريﻀﻪ ;ٮ&ٮﺎ)ﺣﺬ‬metoprolol ‫ى ﻣﻦ‬+‫و;ٮ&ٮﻌﺎ)ٮ‬..‫ٮﻪ ﻣﺶ )ڡﺎﻛﺮﻫﺎ‬2‫ وأدويﺔ ٮ&ﺎ)ٮ‬arrhythmia ‫ﻪ؟‬2 ‫ﻣﻤ)ٮوع ٮ&ﺎ)ﺣﺬ اٮ‬
Sotalol
Amiodaron
Flecainide

68/‫;ﺣﺎب رﺳﻤﺔ ال‬action potential ‫ و ﺳﺄل ﻋﻦ ال‬events ‫ى‬+‫ )ڡ‬PR interval


- ventricular depolarization
- atrioventricular conduction
- atrial depolarization
- Ventricular repolarization

69/ ‫ ﺳؤال ﻋﻦ اﻛ&ٮﺮ‬antihyperlipidemia ‫ ;ٮيﻌﻤﻞ‬interactions ‫؟‬


ezetimibe
Statin
Niacin

70/ ?Plant used in inflammation ‫ٮ&&ڡري;ٮًﺎ اﻟﺴؤال‬


Mume fructus

71/
-The major cation in the intracellular fluid?
K+
-The major cation in the extracellular fluid?
Ca+

72/ ‫ ﻟﻮ واﺣﺪ ﻣﺎﳾ ﻋﲆ ال‬enalapril ‫ﺎ)ﺣﺪ اﻟﺪواء اﻟ&ٮﺎ)ٮﻰ ;ٮﻌﺪ ﰷم ﺳﺎﻋﻪ ﻣﻦ ال‬2 ‫ٮﺮﻟﻪ اﻟﺪواء ٮ‬2‫ﻌ‬r‫ٮ‬2‫ و اﻟﺪﻛ&ٮور ﻫ‬enalapril‫؟؟‬
24h
48h
36h

73/ ‫ٮﺮ ;ٮﺎﻟﺴﻦ ﻣﺎﳾ ﻋﲆ‬2‫ٮهﺎ ﻟﻮ واﺣﺪ ﻛ;ٮ‬2‫ﻪ اول ﺣﺎ;ﺣﻪ ٮ&تﻄﻤﻦ ﻋﻠ‬2 ‫ اٮ‬NSAIDs‫؟‬
liver
‫‪Stomach bleeding‬‬
‫‪Renal‬‬ ‫كﻠوي‬

‫;ٮﻌﺪ ﻣﺎﻋﺮف ;ٮﺎﻟﻤ)ٮ)ﻄﺎر اٮ ‪2‬ﻪ اﻟﻌﻼج؟ ‪) esophageal erosins‬ڡ&ٮﺮة ﻃويﻠﻪ و ;ﺣﺎﻟﻪ ‪ NSAIDs‬ﻟﻮ واﺣﺪ ﻣﺎﳾ ﻋﲆ ‪74/‬‬
‫‪Lansoprazole‬‬
‫‪Sacralfate‬‬
‫‪Antodine‬‬

‫واﺣﺪ ;ٮيﺎ)ﺣﺪ ;ﺣ)ٮ&ٮﺎﻣ‪2‬ٮﺴ‪2‬ٮﻦ ﻣﺎٮ ‪)2‬ٮ)ڡﻌﺶ ٮ ‪2‬ﺎ)ﺣﺪ ﻣﻌﺎه اٮ ‪2‬ﻪ؟ ‪75/‬‬


‫‪ ferosamide‬و ﰷن )ڡ‪2‬ٮﻪ‬

‫;ٮﺎٮ ‪2‬ﻦ ;ٮ)ٮﻌﻤﻠﻪ ﻟﻤﺎ ٮ ‪2‬ﻜون اﻟﻌ‪2‬ٮﺎن ﻣﺄكﻠﺶ ﺣﺎ;ﺣﻪ ﻣﻦ اد اٮ ‪2‬ﻪ؟ ‪ tpn‬ال ‪76/‬‬
‫ﻣﺄكﻠﺶ اﻛ&ٮﺮ ﻣﻦ اﺳ;ٮوع‬
‫ﻋﺸﺮ اٮ ‪2‬ﺎم‬
‫)ﺣﻤﺲ اٮ ‪2‬ﺎم‬

‫;ٮيﺸ&ٮ‪r‬ﻌﻞ ﻋﲆ ريﺴ;ٮ&ٮور اﺳﻤﻪ اٮ ‪2‬ﻪ ؟ ‪77/ Phenytoin‬‬


‫‪Na-K-ATPase, the GABAA receptor complex, ionotropic glutamate receptors, calcium channels‬‬
‫‪and sigma binding sites‬‬

‫;ٮ&ٮﻤﺴﻚ ف ريﺴ‪2‬ٮ;ٮ&ٮور اﺳﻤﻪ اٮ ‪2‬ﻪ؟ ‪Mast cell‬‬


‫‪Siglic‬‬

‫;ٮيﻤﺴﻚ ف اٮ ‪2‬ﻪ ;ٮﺎﻟ;ٮﻜ&ٮ‪2‬ٮريﺎ؟ ‪ Antibody‬ال‬


‫‪Antigen‬‬

‫ﰷن ;ٮيﺴﺄل اٮ ‪2‬ﻪ اﻟﺪواء اﻟﲆ ;ٮيﺤ&ٮﺎج ٮ&ﻌﻠ‪2‬ٮﻤﺎت ﻛ&ٮ‪2‬ٮﺮ؟ ‪78/‬‬


‫‪potassium‬‬
‫‪Methotrexate‬‬
‫ﺣﺎ;ﺣﺎت ٮ&ﺎ)ٮ‪2‬ٮﻪ ﻣﺶ )ڡﺎﻛﺮﻫﺎ‬

‫ٮ ‪;2‬ٮ&ڡﺎ ﻣﺎٮ ‪)2‬ٮ)ڡﻌﺶ ٮ ‪2‬ﺴ&ٮ)ﺤﺪم دوا اٮ ‪2‬ﻪ؟ ‪ DHFR‬ﻣﺸكﻠﻪ )ڡ‪+‬ى ال ‪79/‬‬


‫‪methotrexte‬‬
‫*‪ DHFR‬ﻟﻠ&ٮﻮﺿيﺢ‪ :‬دواء اﻟﻤ‪2‬ٮ_ٮﻮ ٮ ‪2‬ﺸ&ٮ‪r‬ﻌﻞ ﻋﲆ اﻻ)ٮزيﻢ*‬
‫*‪ DHFR: dihydrofolate reductase‬ا)ﺣ&ٮﺼﺎر*‬

‫?‪80/ Digoxin main effect‬‬


Decrease preload
Decrease afterload
Increase heart contractlity

81/ First line h1n1?


oseltamivir

82/Test for neutropernia ?


1-ANC
2-WBC

83/ Antimetabolite anticancer ?


Gemcitacbine
Cyclophosphamide

84/Which diuretic listed below would have the [greatest] blood pressure lowering effect then
given in patients with an eGFR below 60 ml/min? ‫ى )ﺣ)ڡﺾ ﺳﻜﺮ اﻟﺪم‬+‫ٮﺮ )ڡ‬2_‫أي ﻣﻦ اﻟﻤﺪرات ﻟﻬﺎ اﻛ;ٮﺮ او اﻋ)ﻄﻢ ٮ&ﺄٮ‬
Chlorthalidone 25 mg daily
Spironolactone 25 mg daily (cI < 20)
Triamterene 25 mg daily
Furosemide 40 mg daily ‫اﻻ&ڡوى‬

85/How to prevent sexual transmitted disease


Condom

86/Urinary clearance is considered the gold standard for measuring GFR?


inulin

87/Who is the most harmful to the stomach? ‫اﻛ_ٮﺮﻫﻢ ﺿرًرا ﻋﲆ اﻟﻤﻌﺪة؟‬


Celocoxibe
Ibuprofen
Naproxen
Diclofenac

88/‫ ﺳؤال ﻋﻦ ﻣريﺾ اﺷ&ٮ)ﺤﺺ‬Perforated colon ‫أول ٮ&ﺪ)ﺣﻞ ﻫﺎم )ڡورا إﻋﻄﺎء؟‬
-‫ٮ;ٮرازول وريﺪ‬2‫اوﻣ‬
-‫ٮﻦ‬2‫ٮﺴ‬2‫)ڡﺎ)ٮﻜﻮﻣ‬
-‫ٮﺪازول او ﺳﻠ)ڡﺎ‬2‫ﻣ&ٮرو)ٮ‬
‫ﻣ&ٮرو)ٮ‪2‬ٮﺪازول او ﺳﻠ)ڡﺎ‪-‬‬
‫ا)ﺣ&ٮ‪2‬ٮﺎر را;ٮﻊ )ٮﺎﺳ‪2‬ٮﻪ‪-‬‬
‫*‪) sucralfate‬ٮ)ﺤ&ٮﺎر ﻣ&ٮرو)ٮ‪2‬ٮﺪازول واذا ﻣﻮ ﻣﻮ;ﺣﻮد )ٮ)ﺤ&ٮﺎر*‬

‫ﻣريﺾ ﻣﻌﺎه أﻟﻢ ﻣﻦ ﺳ)ٮ&ٮ‪2‬ٮﻦ وﻣﺎﻋ)ٮﺪه ﻣﺸﺎكﻞ ٮ_ﺎ)ٮ‪+‬ى اﻟﻢ ;ٮﺸﻌور ﺣرق وﻃﻌﻦ وﻣﺎﺣﺪد ﻣﲀ)ٮﻪ؟ ‪89/‬‬
‫‪ibuprofen‬‬ ‫اٮ&ﻮ&ڡﻊ )ا&ڡﻞ أﻋراض(‬
‫‪gapapentin‬‬

‫؟ ‪ Metformin‬ﻣﱴ ٮ&ﺎ)ﺣﺬ ‪90/‬‬


‫‪A- After meal 2 hr‬‬
‫‪B -Divided the tab during meal‬‬ ‫اٮ&ﻮ&ڡﻊ‬
‫‪C- Before meal 2hrs‬‬
‫‪D -On empty stomach‬‬
‫ه‪+‬ى اﻻﺻﺢ ﺣ;ٮﺔ &ڡ;ٮﻞ اﻟ)ٮﻮم ‪ XL‬واذا ﺣﺼﻠ)ٮﺎ ;ٮﺎﻻ)ﺣ&ٮ‪2‬ٮﺎرات‬

‫اي ا)ٮزيﻢ ﻣﺮٮ&)ڡﻊ ﻣﻦ ا)ٮزيﻤﺎت اﻟ&ڡﻠﺐ؟ ‪ cornary arteri disease‬ﺳؤال ب ‪91/‬‬


‫‪Troponin i‬‬
‫‪Troponin t‬‬

‫؟ ‪ simvastatin‬ﻛﻢ راح ٮ ‪2‬ﺎ)ﺣﺬ ‪ verpamil‬ﺷ)ﺤﺺ ٮ ‪2‬ﺎ)ﺣﺬ ‪92/‬‬


‫‪Simva 10 mg‬‬
‫‪Simva 20 mg‬‬

‫؟‪ wide or narrow‬ﻋﺸﺎن ا&ڡﺪر اﺣﺪد اذا )ﺣﻄﺮ او ﻻ ؟ ٮ ‪2‬ﻌﲏ اذا ﰷن ‪ therapeutic index‬اٮ ‪2‬ﻪ اﻟﺮ&ڡﻢ اﻟ_ٮﺎ;ٮﺖ )ڡ‪+‬ى‪93/‬‬
‫؟ ‪ therapeutic index =4‬ٮ ‪2‬ﻌﲏ ﻟﻮ‬
‫‪narrow‬‬
‫‪wide‬‬
‫‪ narrow‬اذا ﰷن اﻟﺮ&ڡﻢ ﺻ‪r‬ﻌ‪2‬ٮﺮ ٮ ‪2‬ﻌ&ٮ;ٮﺮ*‬
‫‪ wide‬واذا ﰷن ﻛ;ٮ‪2‬ٮﺮ ٮ ‪2‬ﻌ&ٮ;ٮﺮ‬
‫اﻟﺼ‪r‬ﻌ‪2‬ٮﺮ ﻣ_ٮﻼ ﻣﻦ واﺣﺪ ﻟ‪2‬ٮﻦ ا&ڡﻞ ﻣﻦ ﻋﺸﺮه‬
‫*‪ wide‬ﻣﻦ ﻋﺸﺮة و)ڡوق ٮ ‪2‬ﻌ&ٮ;ٮﺮ‬

‫؟ ‪ potasy-sparing diuretic‬ﺳؤال أي ﻣﻦ اﻟﺪٮ ‪2‬وري&ٮﻚ ﻫﺬول‪94/‬‬


‫‪furosemide‬‬
‫‪Spironolactone‬‬

‫ﻛ‪2‬ٮﺲ ﻃويﻠﺔ و)ڡ‪2‬ٮهﺎ ﺻورة ﻻب ريزوﻟﺖ ٮ ‪2‬ﻮم ٮ&)ٮزل ﻻ)ﺣﺮ ﺳﻄﺮ ٮ ‪2‬ﺴﺄل وش اﻟﺪواء ال‪+‬ى ٮ ‪2‬ﺴ;ٮﺐ ﻛﻮ)ٮﺴ&ٮ;ٮ‪2‬ٮﺸﻦ ودارك ﺳ&ٮول ﻣﻦ‪95/‬‬
‫‪..‬اﻻدويﺔ ال‪+‬ى ٮ ‪2‬ﺎ)ﺣﺬﻫﺎ وﻣﻦ ﺿﻤ)ٮﻬﺎ ﺣﺎط )ڡورم ﻣﻦ )ڡورﻣﺎت اﻻٮ ‪2‬رون‬
‫رون‬2 ‫ﺎ)ﺣﺬﻫﺎ وﻣﻦ ﺿﻤ)ٮﻬﺎ ﺣﺎط )ڡورم ﻣﻦ )ڡورﻣﺎت اﻻٮ‬2 ‫ى ٮ‬+‫اﻻدويﺔ ال‬..
iron

96/‫ اﺳﺌﻠﺔ ﻋﲆ‬amino acid ‫ﺶ‬2 ‫ اٮ‬Essential ‫ﺶ‬2 ‫ واٮ‬non essential ‫؟‬
‫ ;ٮ;ﺤﻤﻠﺔ‬٩ ‫ﻋﺪدﻫﻢ‬:
I love lucy very much please try to help arginine
Non essential = 11
Total = 20

97/‫ اﺳﻢ ﻣﻌﺎدﻟﺔ ال‬creatinine clearance for child ‫؟‬


Schwartrz

98/‫ وش اﻟ)ڡورم اﻟﻤ)ٮﺎﺳﺐ ﻣﻦ‬Vit D ‫ﻟﲇ ﻋ)ٮﺪﻫﻢ‬


kidney impairment ‫؟‬
-Calcitriol 1,25 dihydroxycholecalciferol

99/‫ﻌﺔ اﻻﺳ;ٮريﻦ؟‬r‫ٮ‬2‫اﺳ&ٮﺮﻛﺸﺮ و ﺻ‬
C₉H₈O₄

100/ Fat soluble vitamins‫؟‬


Vitamins A, D, E, K ‫كﻠﻬﻢ ﺻﺢ‬

101/ ‫ﺴ;ٮﺐ اﻟﻌﻤﻰ؟‬2 ‫ﺪ ;ٮﺎﻟﻄ)ڡﻮﻟﻪ ٮ‬2 ‫ٮﻦ اﻟﲇ )ٮ&ڡﺼﻪ اﻟﺸﺪٮ‬2‫ٮ&ٮﺎﻣ‬2‫ اﻟ)ڡ‬vitamin cause blindness:
Vitamin A

102/ Antibiogram used for‫؟‬


(is an overall profile of antimicrobial susceptibility testing results of a specific microorganism to a
battery of antimicrobial drugs)

103/ ‫ اﺳ&ٮﺮﻛﺸﺮ‬Ciprofloxacine ‫ﺶ؟‬2 ‫ﻣ;ﺤﻤﻮﻋﺔ اﻟ)ڡﻠور ٮ&ﻌﻤﻞ اٮ‬


(Increase lipophilicity and increase
peneteration of the cell wall).

104/‫ﺶ؟‬2 ‫ٮﺎ اٮ‬2‫ٮﻤ‬2‫;ٮ&ڡﻰ ﺳ;ٮﺐ اﻻ)ٮ‬2 ‫ ٮ‬،‫ٮﻤﻮ;ﺣﻠو;ٮيﻦ )ٮﺎزل‬2‫ٮﻪ ا)ٮﻪ اﻟﻬ‬2‫ﻦ )ڡ‬2 ‫ٮﻞ و;ٮﺎٮ‬2‫ ٮ&ﺤﻠ‬:‫ٮﺲ‬2‫ﻛ‬
Vit B12
Folic acid
‫ﺶ ;ٮﻌﺪ‬2 ‫وﻣﺪري اٮ‬
*‫ )ٮﺤﺪد ﺣﺴﺐ ال‬:‫ ﻟﻠ&ٮﻮﺿيﺢ‬MCV :
*‫ى‬+‫ ;ٮ&ٮ;ٮ&ڡ‬٨٠ ‫ ﻟﻮ ا&ڡﻞ ﻣﻦ‬microcytic ‫ وﺳ;ٮ;ٮﻬﺎ‬:
*‫ى‬+‫ ;ٮ&ٮ;ٮ&ڡ‬٨٠ ‫ ﻟﻮ ا&ڡﻞ ﻣﻦ‬microcytic ‫ وﺳ;ٮ;ٮﻬﺎ‬:
iron deficiency anaemia
*‫ى‬+‫ ;ٮي;ٮ&ڡ‬١٠٠ ‫ وﻟﻮ اﻛ&ٮﺮ ﻣﻦ‬macrocytic ‫ى ﺳ;ٮ;ٮﻬﺎ‬+‫ ;ٮي;ٮ&ڡ‬:
B12 or B9 (folate) deficiency
*‫ ﻟﻮ ﰷن‬Normal (١٠٠-٨٠) ‫;ٮ&ڡﻰ ﺳ;ٮ;ٮﻬﺎ‬2 ‫ٮ‬:
Normcytic anaemia (A B C D)

105/ ‫ دواء ;ٮيﻌﻤﻞ ال‬brown : urine ‫؟‬


nitrofurantoin

106/ The largest capsule size‫؟‬


000
** ‫ ﻟﻮ &ڡﺎل‬:‫ ﻟﻠ&ٮﻮﺿيﺢ‬smallest 5 ‫**)ٮ)ﺤ&ٮﺎر‬

107/ How to make floor stock accessible for nurses‫؟‬


My make it in every floor

108/‫ٮ;ٮﺎ‬2‫ٮﻞ أﻣ‬2‫ٮﻞ و)ﻃﺎﻫﺮ ;ٮﺎﻟ&ٮﺤﻠ‬2‫ ﻋﺎﻣﻠﻪ ٮ&ﺤﻠ‬:‫ٮﺲ‬2‫( ﻛ‬Amoebiasis)، ‫ٮﺎرات‬2‫ﻣﻦ ﺿﻤﻦ اﻟ)ﺤ‬:
epecacanhua /epeca

109/‫ٮﻦ ﻋﲆ اﻟﺤﻤﻞ ٮ&&ڡري;ٮًﺎ ؟‬2‫ٮﺮ اﻟ&ٮﺪ)ﺣ‬2_‫ٮ&ﺄٮ‬


Decrease blood flow of utrine

110/ Side effect of ginkgo‫؟‬


Bleeding

111/‫ ;ﺣﺎب‬Natural product or plant ‫ٮﻪ‬2‫ وﻣﺎﰷن )ڡ‬،‫ ﻣﻤﻜﻦ ٮ&ﺴ&ٮﻌﻤﻠﻪ ﻟﻌﻼج اﻟﺴﻜﺮ‬ginseng ، ‫ٮﺎر‬2‫ﻣﻦ ﺿﻤﻦ اﻟ)ﺤ‬:
Aloe vera

112/‫&ٮﻌﺎﻟﺞ؟‬2 ‫ٮﻒ ٮ‬2‫ ﻛ‬،‫ٮّﻦ ;ٮﺸﺮﻛﻪ‬2‫ﺪ وٮ&ﻌ‬2 ‫ى )ﺣريﺞ ;ﺣﺪٮ‬+‫ٮﺪل‬2‫ﺻ‬


‫ٮﺔ‬2‫ٮﺎت ﺣﻜﻮﻣ‬2‫ﻣﺴ&ٮﺸ)ڡ‬
‫ٮﺎت )ﺣﺎﺻﺔ‬2‫( ﻣﺴ&ٮﺸ)ڡ‬private hospital)

113/‫ ﻟﻮ )ٮﱯ )ٮﻌﻤﻞ‬test ‫ ﻟـ‬liver ‫ٮﻢ‬2‫ٮﺮ ﺳﻠ‬2‫ٮﻢ او )ﻋ‬2‫)ٮﺸﻮف ﺳﻠ‬:


Creatinine
Urine analysis
Albumin
‫؟‪114/ Mechanism of sitagliptin‬‬
‫‪A Dipeptidyl Peptidase IV Inhibitor‬‬

‫‪):‬ٮﻌﻤﻠﻪ &ڡ‪2‬ٮﺎس اﻟ)ٮ)ﻄﺮ ﻣﱴ؟ ﻣﺎﰷن ;ﺣﺎٮ ‪2‬ﺐ ;ٮﻌﺪ ﺳ)ٮﻪ‪ ،‬ﻣﻦ ﺿﻤﻦ اﻟ)ﺤ‪2‬ٮﺎرات ‪ Diabetes type2‬واﺣﺪ ﻋ)ٮﺪه‪115/‬‬
‫ﻟﻠ)ٮ)ﻄﺮ ‪Infestation or check every year‬‬
‫ٮ ‪2‬ﻌﲏ كﻞ ﺳ)ٮﻪ )ڡ‪+‬ى )ٮ)ڡﺲ اﻛ&ٮﺸﺎف اﻟﻤﺮض‬

‫؟ ‪ Monitoring of warfarin‬ﻛ‪2‬ٮﻒ )ٮﻌﻤﻞ‪116/‬‬


‫ٮ&)ٮ)ڡﻊ ‪INR and enoxaparin‬‬
‫‪) :‬ٮ)ﺤ&ٮﺎرﻫﺎ ﻋﲆ ﻃول واذا ﻣﺎ;ﺣﺎب )ٮ)ﺤ&ٮﺎر )‪ (gum bleeding‬اذا ;ﺣﺎب‬
‫‪Xa Factor‬‬
‫‪ warfarin‬ٮ&;ٮﻊ ال ‪: INR‬ﻟﻠ&ٮﻮﺿيﺢ*‬
‫‪ heparin‬ٮ&;ٮﻊ ال ‪ APTT‬اﻣﺎ ال‬

‫‪:‬؟ ﻣﻮ ﻣ&ٮﺎﻛﺪه ﻣﻦ اﻟﺴ;ٮﻠ)ٮﻖ‪ ،‬اﻟ)ﺤ‪2‬ٮﺎر ‪ orange red prostate‬؟ او ‪ orange red precipitate‬ﻛ‪2‬ٮﺲ ٮ&ﺤﻠ‪2‬ٮﻞ‪117/‬‬
‫‪Alkaloid‬‬

‫‪:‬ﺳريﻌﻪ‪ ،‬ﻣﻦ ﺿﻤﻦ اﻟ)ﺤ‪2‬ٮﺎرات ‪ OTC‬وﻋ)ٮﺪه ﺿ‪r‬ﻌﻂ وي;ٮ‪r‬ﻌﻰ ﺣﺎ;ﺣﻪ ‪ runny nose،‬واﺣﺪ ﻋ)ٮﺪه‪118/‬‬
‫‪Paracetamol‬‬
‫‪oxymetazoline‬‬

‫‪:‬وﻛﺤﻪ )ٮﺎﺷ)ڡﻪ‪..‬ﻣﻦ ﺿﻤﻦ )ﺣ‪2‬ٮﺎرات اﻟﻌﻼج ‪ control hypertension‬واﺣﺪ ﻋ)ٮﺪه‪119/‬‬


‫‪dextromethorphan‬‬

‫‪:‬اﻟ)ﺤ‪2‬ٮﺎرات ‪ supportive fluid،‬؟ ﻣﺎﰷن )ڡ‪2‬ٮﻪ ‪ Antiseptic shock‬اول ﺣﺎ;ﺣﻪ )ٮﺴ&ٮ)ﺤﺪﻣﻬﺎ ;ٮـ‪120/‬‬
‫‪Norepinephrine‬‬
‫‪Dopamine‬‬
‫*‪) Dopamine‬ٮ)ﺤ&ٮﺎر ‪ CKD‬ﻟﻠ&ٮﻮﺿيﺢ‪ :‬اذا ﰷن ﻋ)ٮﺪه*‬

‫)ڡ‪2‬ٮﻪ ﻣﺮض )ﻃﻬﺮ ;ٮﺎﻟﻤ)ٮﻄ&ڡﻪ واﻟﺤﺎﻻت اﻟ;ﺤﺪٮ ‪2‬ﺪه اﻟﲇ ٮ&)ﻄﻬﺮ ﻣ)ٮﻪ اﻟﲇ ه‪+‬ى؟‪121/‬‬
‫‪incidence‬‬

‫وا)ٮﺖ ﺻ‪2‬ٮﺪل‪+‬ى ‪ Isotretinoin‬ﻣﻜ&ٮﻮب ﻟﻚ‪122/‬‬


‫؟‪ ethics‬واﻟ;ٮ)ٮﺖ ﻣﻜ&ٮﻮب ﻟﻬﺎ ﺣﺎﻣﻞ‪..‬وا)ٮﺖ ﺻﺮ)ڡ&ٮﻪ ﻟﻬﺎ ﻛﺬا ٮ ‪;2‬ٮ&ڡﻰ ﻣ)ﺤﺎﻟﻒ ﻷي‬
‫‪Non-maleficence‬‬

‫؟ ;ٮﺎﻟ&ٮﺮٮ&‪2‬ٮﺐ ‪) Discovery of drug‬ﺣﻄوات‪123/‬‬


123/‫ )ﺣﻄوات‬Discovery of drug ‫ٮﺐ‬2&‫؟ ;ٮﺎﻟ&ٮﺮٮ‬
Taeget > Bioassays > Lead compound > Pharmacophore study

124/‫ ﻣريﻀﻪ وﻋ)ٮﺪﻫﺎ‬infection ‫ وٮ&ﺎ)ﺣﺬ‬vancomycin ‫ ﻟﻤﺎ ﻋﻤﻠﻮ‬monitoring ‫ ل‬voncomycin level ‫ ﻣﻦ‬,23 ‫ﻟ&ڡﻮه‬
‫ٮﺎرات‬2‫ ﺿﻤﻦ اﻟ)ﺤ‬:
Decrease infusion rate
Decrease dose ‫اٮ&ﻮ&ڡﻊ‬

125/‫ ﻣريﺾ ;ٮيﺎ)ﺣﺬ‬anti-trylipidemia ‫ )ٮﻌﻤﻠﻪ‬،‫ٮﺴ&ٮرول‬2‫ ;ٮيﺎ)ﺣﺬ ﺣﺎ;ﺣﻪ ﻟﻠﻜﻮﻟ‬check or montoring ‫ﺶ؟‬2 ‫;ٮـ اٮ‬
creatine kinase

126/‫ٮﺎرات‬2‫ ﻣﻦ ﺿﻤﻦ اﻟ)ﺤ‬،‫ٮﺐ‬2‫ﺄٮ_ﺮ ;ٮﻬﺮﻣون اﻟﺤﻠ‬2 ‫ﺶ اﻟﲇ ﻣﻤﻜﻦ ٮ‬2 ‫وﺣﺪه ﻣﺮﺿﻌﻪ اٮ‬:
.A.Progesterone
B. Prolactin ‫اٮ&ﻮ&ڡﻊ ذي اﻟﺼﺢ‬
*‫ )ٮﺸﻮف اﻟﺴؤال ﻃﺎﻟﺐ‬:‫ ﻟﻠ&ٮﻮﺿيﺢ‬:
Contraceptive pills ‫ وﻻ‬Hormone !
‫ ﻟﻮ‬pills ‫;ٮ&ڡﻰ‬2 ‫ ٮ‬Progesterone
‫;ٮ&ڡﻰ‬2 ‫ ﻟﻮ ﻫﺮﻣون ٮ‬Estrogen / androgen
**‫;ٮﻄري&ڡﺔ أ)ﺣرى‬: **
? Which contraceptive pills prevent lactation
1- progesterone
2- estrogen
3- combined

127/‫ٮﺮ ﻣ)ٮﻬﺎ‬2‫ ا)ﺣﺬ ادويﻪ ﻛ_ٮ‬:‫ٮﺲ‬2‫ ﻛ‬codiene ‫ و‬proton pump ‫ و‬magnesium carbonate ‫ و;ﺣ&ٮﻪ‬،‫ ٮ&&ڡري;ٮًﺎ‬diarrhea ‫وش‬
‫ﺳ;ٮ;ٮﻬﺎ ؟‬
megnesium
**‫ﻣﻌﻠﻮﻣﺔ‬: **
Aluminium ‫ﻌﻤﻞ‬2 ‫ ٮ‬: constipation (‫)اﻣﺴﺎك‬
*Magnesium ‫ﻌﻤﻞ‬2 ‫ ٮ‬: diarrhea (‫)اﺳﻬﺎل‬

128 / QRS ‫؟‬


Ventricular depolarization

129/‫ ﻣريﺾ ﻋ)ٮﺪه اﻃ)ڡﺎل وﺣ;ﺤزو ;ٮـ‬Hotel، ‫ﺶ؟‬2 ‫ٮ&كﻠ)ڡ&ٮﻪ ٮ&ﻌ&ٮ;ٮﺮ اٮ‬
Direct non medical
*‫ ﻟﻮ &ڡﺎل‬:‫ ﻟﻠ&ٮﻮﺿيﺢ‬hospitalisation : direct medical
130/‫ﻣريﺾ ﻛﺬا دواء‬،anti hypertension ‫ و‬diabetes ‫ وﺣﺼﻞ ﻟﻪ‬ototoxicity ‫ٮﻦ اﻟﻤﺴؤول ع اﻟﻤﻮﺿوع ذا؟‬2‫ﻣ‬
Amikacin

131/‫ اﻟﺤﺎﻣﻞ ل‬finasteride ، ‫ ٮ&ﻜون‬contraindicated ‫ٮهﺎ ﻋﺸﺎن؟‬2‫)ڡ‬


Teratoginicity

132/‫ ;ﺣﺎب‬Ginkgo biloba ‫ﺶ؟‬2 ‫ﻌﻤﻞ اٮ‬2 ‫ ٮ‬side effect ‫ ٮ&;ٮﻌﻪ‬:


Bleeding

133/‫ﺎ)ﺣﺬ‬2 ‫ ﻣريﺾ ٮ‬: ‫ٮﺲ‬2‫ ﻛ‬vancomycin ‫ٮﺲ‬2‫ و;ٮ)ٮ&ڡ‬rough rate ‫ او‬pead ‫ ﻟﻤﺎ &ڡﺴ)ٮﺎه‬،20 ‫ ﻟـ‬10 ‫ﻜون ﻣﻦ‬2 ‫ واﻟﻤ)ڡروض ٮ‬،‫ٮ&;ٮﻌﻪ‬
‫ﻣﺎ اﻟﺤﻞ؟‬..23 ‫ﻃﻠﻊ‬
Decrease the dose of vancomycin

134/ ‫ٮﺐ اﻟﺪاﺋﺮه رﺳﻤﺔ‬2‫;ﺤ‬2 ‫ ٮ‬Pie Chart ‫ ﻟـ‬error category ‫اﻋﺮ)ڡﻮﻫﻢ‬:


No error: A
Error, no harm: B, C, D
Error, harm: E, F, G, H ( H: Near death)
Error, death: I

135/ plant based anticancer ‫؟‬


‫ٮﺎرات ﻻ‬2‫ى اﻟ)ﺤ‬+‫ٮﻪ )ڡ‬2‫ وﻣﺎﰷن )ڡ‬vincristin ‫ وﻻ‬paclitaxel.
Etoposide

136/ who is responsible for solving medication shortage in pharmacies‫؟‬


-saudi health council
-MOH
-SFDA

137/ ptn underwent septic shock, what


should be started immediately ‫ٮﺎرات‬2‫؟ ﻣﻦ اﻟ)ﺤ‬
Dopamine IV
Antibiotic

138/ Vegum IS ‫؟‬


Suspending agent
Emulsifying agent
139/
Q/ ptn history medications: x, y, z
experiencing SE: dizziness, nephrotoxicty, and neutropinea, put them in order of importance
which complication you will adress first ‫ٮﺐ ﻻزم‬2&‫؟;ٮﺎﻟ&ٮﺮٮ‬:
Nephrotoxicity- Neutropenia- Dizziness

140/ Boxed warning when using for teeth pain in infant lead to seizure and death ‫؟‬
Clove oil
Ibuprofen suspension
Gel

141/mother with Hepatitis B in labor,


what to give newborn‫؟‬
Heb B vaccine + HbIg

142/ Best covid prevention‫؟‬


surgical mask
N95 mask ‫ٮﺮ ﻣﻮ;ﺣﻮد )ٮ)ﺤ&ٮﺎر اﻟﻤﺎﺳﻚ‬2‫واذا )ﻋ‬
hand wash

143/ whats the max duration of narcotic parentral given in home care setting‫؟‬
12hr
24hr
35hr
70h

144:/ ?Cyproheptadine is
A-antidepressent
b-anti psychotic
c-anti hypertensive
d- anti histamine

145/ ‫ٮﻦ وﻋ)ٮﺪه‬2‫ ﺳ)ٮ‬4 ‫ ﻃ)ڡﻞ ﻋﻤﺮه‬diarrhea:


A- Loperamide
B- Rehydrate with oral solution

146/‫ٮﻪ واﺣﺪ اﺳ&ٮ)ﺤﺪم ال‬2‫ٮﺲ &ڡﺎﻟﻚ )ڡ‬2‫ﻛ‬cannabinoids ‫ى‬+‫ﺶ اﻟﻤﺎدة اﻟ)ڡﻌﺎﻟﻪ )ڡ‬2 ‫ وﺳﺎل اٮ‬cannabinoids‫؟‬
146/‫ٮﻪ واﺣﺪ اﺳ&ٮ)ﺤﺪم ال‬2‫ٮﺲ &ڡﺎﻟﻚ )ڡ‬2‫ﻛ‬cannabinoids ‫ى‬+‫ﺶ اﻟﻤﺎدة اﻟ)ڡﻌﺎﻟﻪ )ڡ‬2 ‫ وﺳﺎل اٮ‬cannabinoids‫؟‬
Δ9- tetrahydrocannabinol

147/‫ٮﺲ ﻋﻦ وﺣﺪة ﻋ)ٮﺪﻫﺎ‬2‫ ﻛ‬seizure ‫وٮ&ﺴ&ٮ)ﺤﺪم‬


valproic acid ‫ى ﻣﻦ زيﺎدة‬+‫و;ﺣﺖ اﻟﻤﺴ&ٮﺸ)ڡﻰ ٮ&ﺸ&ٮك‬
seizure episodes ‫ٮﺮة واﻟﺪﻛ&ٮور ﻋﻤﻞ‬2‫ى اﻟ)ڡ&ٮﺮة اﻷ)ﺣ‬+‫)ڡ‬
increase of valproic acid dose by 50%
‫ﺶ اﻟﻬﺪف ﻣﻦ ذا اﻟﴚ ؟‬2 ‫اٮ‬
A- Decrease seizure episode by 50%\week
B- Decrease sizure episode by25%
C- To make seizure episode 0%

148/ which not effect on Warfarin ‫؟‬


Green tea
Cyanocobalamin
Garlic
Coenzyme Q10

149/ ?IgM which type of hypersensitivity


Type I
Type II
Type III ‫اٮ&ﻮ&ڡﻊ اﻻﺻﺢ‬
Type IV
‫ﻟﻠ&ٮﻮﺿيﺢ‬: IgM : Type2 and Type3

150/ ?Prevention breast cancer


Vit A 100
Vit D 200
None of them
Vit A + Vit D

151/ ?Treatment thrombocytopenia


UFH
LMWH
Warfarin
Argatroban

152/ ‫ﻣﱴ ا)ڡﻀﻞ و&ڡﺖ ٮ&ﻌﻤﻞ )ڡﺤﺺ‬


152/ ‫ﻣﱴ ا)ڡﻀﻞ و&ڡﺖ ٮ&ﻌﻤﻞ )ڡﺤﺺ‬
?for gastational diabetes
20-24 weeks
24-28 weeks

153/ Which form of vit D in chronic kidney disease ‫؟‬


A. Ergocalciferol
B. Cholecalciferol
C. Vit D 2
D. 25 oh cholecalciferol

154/ What supplements use to decrease Triglycerides ‫؟‬


Co Q10 ‫اٮ&ﻮ&ڡﻊ أﺻﺢ ﳾ‬
Biotin
B6

155/ Colistiin ‫ اﻟﻀرر ﻋﲆ‬:


Kidney ‫( ٮ&ﻌﲏ‬Nephrotoxicity)
Liver
Lung
Heart

156/‫ﺶ اﻟﺼﺢ‬2 ‫ اٮ‬Enoxaparin ‫؟‬


Clexane SQ 80mg every 12 hours-
Enoxaparin SQ 80mg every 12 hours-
Enoxaparin SQ 80mg every 12 hour
Enoxaparin SQ 80mg twice daily ‫اٮ&ﻮ&ڡﻊ اﻻﺻﺢ‬

157/‫ ﺳؤال‬FDA ‫اﻟ;ﺤواب‬..‫ﺼﺮف ;ٮﻮﺻ)ڡﻪ‬2 ‫ﺔ ﻋﲆ دواء ٮ‬2 ‫و&ڡ)ڡﺖ ﻣﺼ)ٮﻊ ﻋﺸﺎن اﻟﻤﺼ)ٮﻊ ﺣﻂ دﻋﺎٮ‬:
‫ ﺳﻌﻮدي‬fda ‫ﻤ)ٮﻊ ٮ&ﺴويﻖ اﻻدويﻪ اﻟﲇ ;ٮﻮﺻ)ڡﻪ‬2 ‫ٮ‬

158/‫ ﺳؤال ﻋﻦ‬hypersensitivity: ‫ اي )ٮوع؟‬،‫ٮﻪ‬2‫;ٮ)ٮﺖ ﻟ;ٮﺴﺖ ﺳﻠﺴﺎل و;ﺣ&ٮﻬﺎ ﺣﺴﺎﺳ‬


Type IV (‫)اﻟرا;ٮﻊ‬
*‫ﺴﻤﻰ اﻟرا;ٮﻊ‬2 ‫ﺶ ٮ‬2 ‫ﻣﻬﻢ )ٮﻌﺮف اٮ‬: delayed*
*‫ و اﻻول‬type I ‫ﺴﻤﻰ‬2 ‫ٮ‬: immediate*

159/‫ ﺳؤال ﻋﻦ ال‬IG ‫ﻄﻠﻊ ﻣﻊ ال‬2 ‫ اﻟﲇ ٮ‬parasite ‫؟‬


IgE

160/‫ ﺳؤال ﻋﻦ ال‬resource ‫ ﻣ_ٮﺎل‬Pupmed :


Secondary

161/‫ ﺳؤال ﻋﻦ ال‬resource : ‫ ﻣ_ٮﺎل‬Review article


Secondary / Tertiary ‫ﺣﺴﺐ‬

162/ which type of vitamin D that is used to prevent breast cancer ?


D2
25-dihyrox..
*‫ٮﻪ‬2‫( ﻟﻮ )ڡ‬D3 (Cholecalciferol ‫ى اﻟﺼﺢ‬+‫ه‬،
25 : ‫ٮﻪ‬2‫(واذا ﻣﺎ)ڡ‬OH)D*

163/‫ﺴوي‬2 ‫ٮهﻢ ٮ‬2‫ ;ﺣﺎب ﻣ;ﺤﻤﻮﻋﺔ ادويﺔ و &ڡﺎل وش اﻟﲇ )ڡ‬Constipation (‫؟)اﻣﺴﺎك‬
Morphine

164/‫ ادويﺔ ال‬TB ‫ﺴوي‬2 ‫ﺶ اﻟﲇ ٮ‬2 ‫ اٮ‬ocular toxicity ‫ ؟‬ethambutol

165 / Dose of ferrous sulfate in pt with iron deficiency anemia‫؟‬


325 mg

166/ Vaginal candidiasis‫؟‬


Clotrimazole

167/ What to give for malignant hypertension‫؟‬


Nitroprusside

168/ How to avoid LASA ‫؟‬


Tell man letters

169/Which Antibiotic are inactivated by surfactants‫؟‬


Daptomycin

170/ Drug cause agranulocytosis ‫؟‬


Clozapine and propylthiouracil

171/ ‫ ﻣريﺾ ﻋ)ٮﺪه‬uncontrolled diabetes ‫ى اﻻﻃراف؟‬+‫ٮﻦ ﺣﺎس ;ٮﺄﻟﻢ )ڡ‬2‫ٮﻦ واﻟﺤ‬2‫ﻣﻦ ﺳ)ٮ‬
171/ ‫ ﻣريﺾ ﻋ)ٮﺪه‬uncontrolled diabetes ‫ى اﻻﻃراف؟‬+‫ٮﻦ ﺣﺎس ;ٮﺄﻟﻢ )ڡ‬2‫ٮﻦ واﻟﺤ‬2‫ﻣﻦ ﺳ)ٮ‬
Gabapentin

172/ Levodopa/carbo counseling‫؟‬


High protein food reduces absorption

173/ Hypnotic drugs dosing is based on ‫؟‬


Dose dependent
Time
Gender

174/ Symotoms of hyperthyroidism‫؟‬


Tachycardia

175/ Anti-hyperthyroidism cause permanent decline in T4‫؟‬


Radioactive iodine

176/ TSH was low and patient diagnosed with goiter ‫؟‬
Iodine deficiency
Iron deficiency

177/ Phenytoin was 100mg/TID, then increased to 200 mg/TID, first symptoms to appear ‫؟‬
Rash
Drowsiness
Gingival hyperplasia

178/ Risk factor cause DVT‫؟‬


Surgery

179/ Medication increase INR‫؟‬


Amiodarone

180/ Effect on Warfarin‫؟‬


Rifampin ‫اﻻﺻﺢ‬
Amiodarone

181/ Why Thiazide not recommended in kidney patient?


Not used in case of kidney imparment

182/ BPH what counseling should tell to the patient?


Cause orthostatic hypotension
Continue to 6 month to give effect

183/ ‫ٮﺲ ﻣريﺾ ا)ٮﻜ&ٮﺐ ﻟﻪ‬2‫ ﻛ‬Finasteride ‫ﺶ ال‬2 ‫ اٮ‬counseling ‫؟‬


5alpha-reductase inhibitors group of BPH (benign prostatic hyperplasia)

184/ Get rid of spore..Autoclave:


Anthrax transmission by [cattle]

185/ Studies could establish the drug shelf-life?


Stability studies

186/ Which correct?


Ergo calcification is D3
Cholecalciferol is D2
Vit D2 and D3 active biological
Vit D2 and D3 inactive biological

187/ ‫ﻜون ﻋ)ٮﺪﻫﻢ‬2 ‫ى ﻟﻤﺎ ٮ‬+‫ اﻟﻄﺎ&ڡﻢ اﻟﺼح‬:‫ ﺳؤال‬low health literacy ‫ﺴوون ﻟﻠﻤﺴ&ٮﺸ)ڡﻰ ؟‬2 ‫وش ٮ‬
low health quality

188/ Category X contraindiaction in breastfeeding?


Methotrexate

189/ ‫(ﻋﻼج ﻫﺸﺎﺷﺔ اﻟﻌ)ﻄﺎم‬osteprosis)


‫ ;ٮﻌﺪ ال‬Ca ‫ و‬VitD‫؟‬
Alendronate ‫ٮﺎرات‬2‫ﻣﻦ ﺿﻤﻦ اﻟ)ﺤ‬

190/ ‫ﻌ&ٮ;ٮﺮ‬2 ‫ وش اﻟﲇ ٮ‬High alert (HAM) ‫؟‬


Dextrose 50 ‫ﻣﻦ ﺿﻤ)ٮﻬﺎ‬

191/ Antibiotic interact with pulmonary surfactant?


Dapromycin
192/ Vinca labeling Avoid inteathecal :
Iv use only

193/ Using an iv :
Mini Bag

194/ Hemorrhoids and is allergic to lidocaine, which he can give?


Pramoxine

195/ Patient take levothyroxine when to monitor?


3 months
3 weeks
4 months
4 weeks
*take the morning on empty stomach, at least 30 min before breakfast*

196/ Patient with Crohn’s disease has been on the maximum dose of budesonide but still
uncontrolled, presented to the ER with acute symptoms, what to give?
IV cyclosporin
IV methylpred
Salazine

197/ Which cephalosporins are (not) active against pseudomonas species?


Cefazolin (first generation)-
Cefepime-
Ceftobiprole-
Ceftazidime-

198/ Rifampcin have problem in vision, what the drug resbonsibilty for this side effect?
Ethambutol

199/‫ ﺳؤال‬Horizontal laminar airflow ‫ﺶ؟‬2 ‫ٮﺮ ﻣﺮﻛ;ٮﺎت اٮ‬2‫ﺴ&ٮ)ﺤﺪم ﻟ&ٮﺤﻀ‬2 ‫ٮ‬
Antibiotics
Chemotherpy
*‫ أﻣﺎ‬Vertical flow ‫ ;ٮ&ٮﻜون‬: chemotherapy*

200/Mechanism of action aliskiren?


acts as a renin inhibitor, blocks the conversion of angiotensinogen to angiotensin I

201/‫ ﻣريﺾ ﰷن ;ٮيﺎ)ﺣﺪ‬parenteral nutrition ‫ٮ&ٮﺲ‬2‫ٮﻮم واﻟﻤﻌﺎدن واﻻﻟﻜ&ٮروﻟ‬2‫ﻌ)ٮﺴ‬r‫ٮﻮم واﻟﻤ‬2‫ى )ٮﺴﺐ اﻟ;ٮﻮٮ&ﺎﺳ‬+‫وﺣﺼﻠﻪ ا)ﺣ&ٮﻼل )ڡ‬
‫وﺳﺎل ﻋﻦ اﻟﺴ;ٮﺐ‬.
‫ اﻋ&ٮ&ڡﺪ ا)ٮﻬﺎ‬refeeding syndrome
** ‫ٮﺸ)ٮﺖ‬2‫ اذا اﻟ;ٮ‬، ‫ )ﻋﺎﻟ;ٮا ً اﻹ;ﺣﺎ;ٮﺔ ﺻﺢ‬:**‫ﻟﻠ&ٮﻮﺿيﺢ‬
Lactic acido PO4,K,Mg abnormalities
Thiamine Deficiency ( ‫) ﻣﻮ ﻻزم كﻠﻬﺎ ﻣﻊ ;ٮﻌﺾ‬
‫ﻜون اﻟﻤريﺾ دا)ﺣﻞ‬2 ‫ ٮ‬refeeding Syndrome

202/‫ٮﺔ وٮ&ﺴﺄﻟﻚ ﻋﻦ‬2‫ٮﺪﻟ‬2‫ أم ;ﺣﺖ اﻟﺼ‬:‫ ﺳؤال‬phenytoin ‫ وا)ٮﺖ كﻠﻤﺖ‬،‫ى ﺷﺎ)ڡﺖ ا)ٮﻪ ﻣﻜ&ٮﻮب ﻟﻠﺼر ع واﻟﻄ)ڡﻞ ﻣﺎﻋ)ٮﺪه ﺻر ع‬+‫ه‬
‫ و&ڡﺎﻟﻚ ﻋﺸﺎن‬،‫ اﻟﺪﻛ&ٮور واﺳ&ٮ)ڡﺴﺮت ﻣ)ٮﻪ ﻋﻦ ﺳ;ٮﺐ اﻟﺼﺮف‬enzyme inducer ‫وش ﻣﱴ اﻷم‬..‫وﺷﺮﺣﺖ ﻟﻸم و)ڡﻬﻤﺖ‬
( literacy ) ‫؟‬
A.law ‫اﻟﻬواري‬.‫د‬
B.intermedate
C.proficient ‫ٮﻤﺎن‬2‫ﻣﺤﻤﺪ ﺳﻠ‬.‫د‬
D.under law

203/ ‫ﺶ‬2 ‫ اٮ‬Anti-cancer ‫ى‬+‫ﻌط‬r2 ‫ اﻟﲇ ٮ‬Mitotic ‫؟‬


M phase
*‫ اﻻدويﻪ‬:*‫ ﻟﻠ&ٮﻮﺿيﺢ‬mitotic :
Vinca alkaloids ( vincristine)
Taxans (paclitaxel)
Etoposide
‫ﻌﻞ كﻞ‬r‫ اﻷدويﺔ اﻟﲇ ;ٮﺸ&ٮ‬anti mitotic

204/‫ٮﻪ ﻋﲆ‬2‫ ﺳؤال ﻟﻤريﻀﻪ ﺣﺎﻣﻞ و ﰷ)ٮﺖ ﻣﺎﺷ‬valproate ‫ وﰷن ﰷٮ&ﺐ ﻋ)ٮﺪﻫﺎ‬seizure ‫ﺶ )ٮﺴوي‬2 ‫ﻣﺎ ﺣﺪد )ٮﻮﻋﻪ و ﻃﺎﻟﺐ اٮ‬
‫ﻣﻌﻬﺎ ؟‬
D/C valproate and start levetiracetam
‫ ا)ڡﻀﻞ‬: ‫ٮﻪ‬2‫ﻟﻮ )ڡ‬:
Lower starting dose of lamotrigine and titrate slowly

205/ Interaction between rifampin and alprazolam?


Rifampin will decrease the level or effect of Alprazolam by affecting hepatic/intestinal
enzyme CYP3A4 metabolism
‫;ٮﻤﻌﲎ‬:
Rifampicin is enzyme inducer so decrease alprazolam conc
206/ MOA of isoproternol ‫؟‬
B1 B2 agonist/ ‫ ;ٮﻤﻌﲎ‬:
Stimulates both B1 and B2 adrenergic receptors

207/ ‫ ﺳؤال ا)ٮﻪ اﻟﻤرأة اﻟﺤﺎﻣﻞ كﻞ ﻣﺎ ٮ&&ڡﺪﻣﺖ ;ٮﺎﻟﺤﻤﻞ ;ٮزيﺪ ال‬Vd ‫ٮﺶ؟‬2‫ﻟ‬
Delay gastric empety
Increase surface area of placenta
Decrease gastric motility
Increase plasma volume

208/ Amino acid in hair and keratin?


Arginin
cysteine

209/ ‫ﺶ؟‬2 ‫ اٮ‬،‫ٮًرا‬2‫اﻟﻤﺮض اﻟﲇ ﺣﺼﻠﻮ ﻟﻪ ﻋﻼج ا)ﺣ‬


HepB ‫ﻣﺎﻟﻪ ﻋﻼج‬
HepA
HepC
HepD

210/ ‫ وﺣﺪه ٮ&ﺎ)ﺣﺬ‬:‫ٮﺲ‬2‫ ﻛ‬Digoxin ‫ٮﻦ( ﻣﻦ ا)ﺣﺮ ;ﺣﺮﻋﺔ وﺻﺎر‬2‫ٮﺎﺳﺎٮ&ﻬﺎ ;ٮﻌﺪ )ﺳﺎﻋ&ٮ‬2‫ اﻋ&ٮ&ڡﺪ وﻫﻤﺎ ا)ﺣﺬو &ڡ‬2-1 ‫ى‬+‫ٮع‬2‫واﻟريﺖ اﻟﻄ;ٮ‬
‫&ٮﻌﻤﻞ؟‬2 ‫ﺶ ﻣ)ڡ&ٮﺮض ٮ‬2 ‫ اٮ‬,2.5 ‫ٮﺰ‬2‫اﻟ&ٮﺮﻛ‬
Digoxin fab
Saline
Lower the dose
Nothing to do
‫ٮﺎراٮ&ﻪ‬2‫او )ﺣ‬:
Use Antidote
Nothing to do (‫ٮﻦ‬2‫ ﺳﺎﻋﺎت ﻣﻮ ﺳﺎﻋ&ٮ‬٦ ‫ٮﻤﺎن )ﻻن اﻟﻤ)ڡروض ;ٮﻌﺪ‬2‫ﻣﺤﻤﺪ ﺳﻠ‬.‫د‬
Lower the dose

211/ ‫ رﺳﻤﺔ ا)ٮﻪ‬lipid ‫ ﻋﻤﻠ)ٮﺎه‬cyclization ‫ٮﺮ‬2‫ﺼ‬2 ‫ ﺣﱴ ٮ‬fit with receptor :


Rigidity ‫ٮﻪ‬2‫اﺣﺪ اﻛﺪ ﻋﻠ‬
Stability
Flixablity

212/‫ﺆٮ_ﺮ ﻋﲆ اﺳ&ٮﻤراره؟‬2 ‫ وش اﻟﲇ ٮ‬،‫ٮﺪ وي&ڡﻠﻞ اﻻ)ﺣﻄﺎء‬2‫ٮﻮٮ&ﺮ ﻣ)ڡ‬2‫ اﻟﻜﻤ;ٮ‬:‫ﺳؤال‬


‫ﺳؤال‪ :‬اﻟﻜﻤ;ٮ‪2‬ٮﻮٮ&ﺮ ﻣ)ڡ‪2‬ٮﺪ وي&ڡﻠﻞ اﻻ)ﺣﻄﺎء‪ ،‬وش اﻟﲇ ٮ ‪2‬ﺆٮ_ﺮ ﻋﲆ اﺳ&ٮﻤراره؟‪212/‬‬
‫‪System down‬‬

‫ﺳؤال ﻋﻦ اﻟﴚء اﻟﲇ ٮ ‪2‬ﺴﻬﻞ وﺻول اﻟﺪواء ﻟﻠﻤﺮﴇ؟‪213/‬‬


‫‪pyxis‬‬

‫؟ ‪ zidovudine‬ﻣﺪة ﻋﻼج‪214/-‬‬
‫‪4-6 weeks‬‬
‫؟‪ fluoxetine‬ﻣﺪة‪-‬‬
‫‪2-12 weeks‬‬
‫‪2 days-12 days‬‬
‫‪2-12 month‬‬

‫;ٮﺲ ﻣﺶ )ڡﺎٮ ‪2‬ﺪة‪ ،‬اﻟﺤﻞ ٮ ‪2‬ﺎ)ﺣﺬ اٮ ‪2‬ﺶ؟ ‪ non pharmacological‬وﺣﺎول ٮ ‪&2‬ٮﻌﺎﻟﺞ ‪ osteoarthititis‬ﻛ‪2‬ٮﺲ ;ٮ&ٮ&ڡول ا)ٮﻪ ﻋ)ٮﺪه‪215/‬‬
‫‪Oral NSAIDS‬‬
‫‪Topical NSAID‬‬
‫‪Topical glucosamine‬‬

‫ود)ﺣﻞ ﻣﻄﻌﻢ ﺻ‪2‬ٮﲏ ٮ ‪&2‬ٮ‪r‬ﻌﺪا و;ٮﻌﺪﻫﺎ ﺣﺲ ;ٮﻀ‪2‬ٮﻖ فﻰ اﻟ&ٮ)ٮ)ڡﺲ و ارٮ&‪2‬ٮﲀريﺎ ‪ allergy from egg‬ﻛ‪2‬ٮﺲ ;ٮي&ڡول ﺷ)ﺤﺺ ﻋ)ٮﺪه‪216/‬‬
‫وﺣﺲ ;ٮﺎﺣﻤرار فﻰ ;ﺣﺴﻤﻪ )ﺣﺼﻮﺻﺎ فﻰ اﻟﻮ;ﺣﻪ وا)ٮ&ٮ)ڡﺎ)ﺣﺎت ﺣول اﻟﻌ‪2‬ٮﻦ واﻟ)ڡﻢ‪ ،‬ٮ&ﻌﻤﻞ ﻣﻌﻪ اٮ ‪2‬ﺶ؟‬
‫‪epinephrine‬‬ ‫اٮ&ﻮ&ڡﻊ‬
‫‪Anti histamine‬‬
‫‪Iv corticosteroids‬‬

‫?‪217/ Aspirin used in mi for‬‬


‫‪1 year‬‬
‫‪2 year‬‬
‫‪3 year‬‬
‫‪Indefinite‬‬

‫‪Five parts:‬‬
*1) Pregnant women on levothyroxine for hypothyroidism presented with complaints of fatigue,
what is your recommendation?*
• Double levothyroxine dose
• Adjust levothyroxine dose based on TSH level during the first trimester ‫ﻣﻤﻜﻦ ذي اﺻﺢ‬
• Nothing, normal for pregnant women to experience fatigue

*2) Female with hypothyroidism on levothyroxine 100 mg is presenting with fatigue and weigh
loss, know that she is planning to get pregnant.What is your recommendation? (TSH and free T4
levels were provided*
• Decrease dose to 50 mg
• Decrease dose to 75 mg
• Increase dose to 150 mg
• Increase dose to 200 mg

*3) Patient with bacteremia, started empiric therapy, culture came back positive for MRSA
resistant for all antibiotics (including Vancomycin) except for Linezolid, so the team decided to
start her on it. The pharmacist looked it up and found that linezolid has no FDA approval for
bactermia. What do you suggest?*
• Give high dose Vancomycin
• Do not start Linezolid since it is not approved for this indication
• Linezolid can be used as off-label since it’s the only option
• Repeat culture

*5) Patient is treated with Vancomycin 15mg/kg Q12h infused over 30 minutes, cultures are
susceptible to vancomycin, after starting vancomycin he developed rash in his face, neck, upper
torso. What the cause of this drug therapy problem?*
• Vancomycin dose is high
• Vancomycin dose is low
• Wrong infusion duration
• Drug-Drug Interaction

*6) First-line hypoglycemic agent after metformin for patients with DM + HF?*
- Empagliflozin or canagliflozin
- Liraglutide ( Victoza ) approved to risk of CV events

*7)Contraindicated hypoglycemic agent in patients with DM + HF? *


Pioglitazone CI
(Rosiglitazone Risk of MI )

*8) Safest antibiotic for pregnant women with UTI in her first trimester?*
- Nitrofurantoin - Cefuroxime

*10) Patient to be started on Azathioprine, genetic testing done and showed negative TMPT
enzyme activity and homogenous TMPT*3 allele, what is the resultant?*
• Increased hepatotoxicity
• Decreased Azathioprine activity
‫ﺎ)ﺣﺬ‬2 ‫)ٮﻌﻄﻰ اﻟﺪواء ﻣﻦ اﻷﺳﺎس او ﻟﻮ ا)ٮﻌﻄﻰ ﻣﺎ ٮ‬2 ‫ اﻟﻤ)ڡروض ﻣﺎ ٮ‬Low dose ‫ٮﻦ‬2‫ﻋﺸﺎن ﻣﺎﻋ)ٮﺪه اﻟ;ﺤ‬
• Azathioprine SE are not related to TMPT

*11) Symptoms of Shwachman(?) Diamond Syndrome?*


• Tachycardia
• Tachypnea
• Weight loss

*12) Recommended INR for patient with mechanical mitral valve?*


- 2.5-3.5

*13) Patient with history of HTN and DM, complaining of runny nose and sneezing. What to give?
*
• Diclofenac + Loratadine
• Diclofenac + Chlorpheniramine
• Acetaminophen + Oxymetazoline
• Acetaminophen + Pseudoephedrine(?)

*14) Patient has iron-deficiency anemia, he is a heavy smoker for 15 years, endoscopy found GI
bleeding. What is the cause of iron-deficiency anemia in this case?*
• Smoking
• Stomach ulcers

*15) While waiting for INR results for a patient on warfarin, to asses safety you will ask a patients
if they are experiencing what side effect?*
• Weight loss
• Bleeding gums
*16) (case) Before starting this patient on fingolimod, what parameters needs to be assessed for
fingolimod dosing?*
• ECG
• Genetics testing

*17) In which of the following scenarios high intensity statins are indicated for primary
prevention?*
• Post MI with LDL of …
• DM type 2 with LDL of …. ( Any type of DM and LDL 70-189 mg/dl )
• Healthy individual with LDL of …( if LDL MORE THAN 190mg/dl
• Angina pectoris with LDL of …

*18) Which of the following is a limitation of CPOE?*


• Time consuming
• System downtime

*19) What is the FDA-approved alternative for heparin in pulmonary embolism if patient has
heparin-induced thrombocytopenia ?*
Argatroban

*20) When a patient is diagnosed with invasive fungal infection for the first time, what important
question you should ask the patient?*
• History of HIV infection
• The use of Amiodarone
• Family history

*22) Which type of hepatitis is curable with recent approved drugs?*


Hep C

- ‫ٮ)ڡﻪ؟‬2‫ﺼﻠﺢ ﻟﻠﺤﺎﻣﻞ او ﻟﲇ ﻣ)ٮﺎﻋ&ٮﻬﻢ ﺿﻌ‬2 ‫هﻢ ﻣﺎ ٮ‬2 ‫ٮﻦ أٮ‬2‫ڡﺎﻛﺴ‬Ç‫اﻟ‬


most live attenuated vaccines, including the (MMR) and varicella vaccines

-41 ‫ٮﻦ وﻋﻤﺮﻫﺎ‬2‫ٮهﺎ؟ ﻣﻊ اﻟﻌﻠﻢ ﻣﺎ ا)ﺣﺬت وﻻ )ڡﺎﻛﺴ‬2‫ى ٮ&ﻌﻄ‬+‫ٮﻦ ال‬2‫ وش اﻟ)ڡﺎﻛﺴ‬٢ ‫وﺣﺪه ﻋ)ٮﺪﻫﺎ اﻟﺴﻜﺮ اﻟ)ٮوع‬
Mmr + tdap
Ha+Hb
Inf + ppsv23 ‫اٮ&ﻮ&ڡﻊ ذي‬
Varicall + ‫ٮﺖ ;ٮﺲ )ﻋﻠﻂ‬2‫)ٮﺴ‬
‫)ٮﺴ‪2‬ٮﺖ ;ٮﺲ )ﻋﻠﻂ ‪Varicall +‬‬

‫؟ ‪ Hepatic Encephalopathy‬ﻛ‪2‬ٮﻒ )ٮ&ڡ‪2‬ٮﺲ‪-‬‬


‫‪Albumin‬‬
‫‪lipase‬‬
‫‪Direct bilirubin‬‬
‫‪Ammonia‬‬
‫؟ ‪ Encephalopathy‬ٮ ‪&2‬ٮ)ﺤﻠﺺ ﻣﻦ اﻷﻣﻮ)ٮ‪2‬ٮﺎ اﻟﲇ )ڡ‪+‬ى ‪ lactulose‬ﺳؤال ٮ_ﺎ)ٮ‪+‬ى ﻋﻠ‪2‬ٮﻪ‪ :‬ﻛ‪2‬ٮﻒ*‬
‫‪-Decrease colon amonia absorption‬‬
‫‪-Increase colon amonia absorption‬‬
‫‪.‬ٮ ‪2‬ﻌﲏ ٮ ‪2‬ﺤﻮﻟﻪ اﻟﻰ أﻣﻮ)ٮ‪2‬ٮﻮم ويﻄﻠﻊ ﻣﻊ اﻟ&ڡﻮﻟون‬

‫وش اﻟ)ڡ‪2‬ٮ&ٮﺎﻣ‪2‬ٮﻦ اﻟﲇ )ٮ&ڡﺼﻪ ﻣﻤﻜﻦ ٮ ‪2‬ﺴ;ٮﺐ ا)ٮ‪2‬ٮﻤ‪2‬ٮﺎ واﺣﻤرار وﺣﻜﻪ ويﻜون &ڡريﺐ ﻣﻦ اﻟ)ڡﻢ؟‪-‬‬
‫‪B1‬‬
‫‪B2‬‬
‫‪B3‬‬
‫‪B7‬‬

‫_ف‪2‬ٮ&ٮﺎﻣ‪2‬ٮﻦ ﻟﻠﺤروق ‪?Vitamin used for burns‬‬


‫‪C‬‬
‫‪A‬‬
‫‪B‬‬
‫‪D‬‬

‫اٮ ‪2‬ﺶ ٮ ‪2‬ﻌﺎﻟ;ﺤﻪ؟ ‪ Pulmonary Embolism‬ﻣريﺾ ﻋ)ٮﺪه‪-‬‬


‫‪Warfarin‬‬
‫‪Aspirin‬‬
‫‪Enoxa‬‬
‫‪Clopidogrel‬‬

‫وﺣﻤﻠﺖ اٮ ‪2‬ﺶ ا)ڡﻀﻞ ﳾ ٮ&ﺴويﻪ؟ ‪ warfarin‬وﺣﺪه ٮ&ﺴ&ٮﻌﻤﻞ‪-‬‬


‫‪ warfarin‬ٮ&ﻮ&ڡﻒ‬
‫‪ warfarin‬ٮ&ﻜﻤﻞ ﻋﲆ‬
‫‪) enoxp‬ٮ‪r‬ﻌ‪2‬ٮﺮ ل‪+‬ى‬
‫واﻟرا;ٮﻌﻪ )ﻋﻠﻂ‬

‫اٮ ‪2‬ﺶ ﻣﻦ ﻫﺬي اﻟ)ٮ;ٮﺎٮ&ﺎت ٮ&&ڡﻠﻞ ﻣﻦ )ڡﻌﺎﻟ‪2‬ٮﻪ اﻟﺪواء؟ ‪ warfarin,‬و ‪ amiodarone‬ﻣريﺾ ﻋ)ٮﻪ أﻣراض وي&ٮﺴﻌﻤﻞ‪-‬‬
‫‪Grape‬‬
‫‪blueberry‬‬
‫‪Green leaves‬‬

‫؟ ‪ INR‬اٮ ‪2‬ﺶ ال‪+‬ى ٮ ‪&2‬ڡﻠﻞ‪-‬‬


‫‪Amiodarone‬‬
‫‪Carba‬‬
‫‪Rifampin‬‬ ‫اﺻﺢ‬
‫‪phenytoin‬‬

‫؟‪ Acetaminophen‬وﺣﺪه ﺣﺎﻣﻞ وٮ&ﺴﺄل ﻋﻦ‪-‬‬


‫ٮ&ﺴ&ٮﻌﻤﻠﻪ اي و&ڡﺖ‬
‫او )ڡ‪+‬ى اﻟ;ﺤﺰء اﻟ_ٮﺎﻟﺚ ﻣﻦ اﻟﺤﻤﻞ‬
‫او ٮ ‪&2‬ٮﻌﺎرض‬

‫؟‪ cost‬اﻋ&ٮ&ڡﺪ ﻛﺬا ويﻦ ٮ ‪2‬ﻜون )ڡ‪+‬ى أي ‪ humanistic‬و;ﺣﺎ)ٮ‪+‬ى ال‪-‬‬


‫‪Cost utility analysis‬‬

‫ﻣ‪2‬ٮﻦ اﻟ)ٮﺎس )ٮﺴوي ﻟﻬﻢ ٮ&ﺤﻠ‪2‬ٮﻞ؟ ‪ carbamazepine‬اﻟ)ٮﺎس ال‪+‬ى ٮ&ﺴ&ٮﻌﻤﻞ‬


‫اﺳ‪2‬ٮﺎ‬
‫أور;ٮﺎ‬
‫ا)ڡري&ڡ‪2‬ٮﺎ‬

‫وﺣﺪه ﻋ)ٮﺪﻫﺎ ﺿ‪r‬ﻌﻂ وﻣﺮض ٮ_ﺎ)ٮ‪+‬ى )ٮﺎﺳ‪2‬ٮﻪ‪ ،‬ا;ٮﻮﻫﺎ ﻋ)ٮﺪه ﺳﻜﺮ اﻟ)ٮوع اﻟ_ٮﺎ)ٮ‪+‬ى وﻫ‪2‬ٮﺎ ﻣﺎﻋ)ٮﺪﻫﺎ ﺳﻜﺮ‪ ،‬ﻛ‪2‬ٮﻒ ٮ&ﻤ)ٮﻊ ﻣﺎ ٮ ‪;2‬ﺤ‪2‬ٮهﺎ ﺳﻜﺮ؟‪-‬‬
‫ٮ&ﺴوي ٮ&ﻤﺎريﻦ ا&ڡﻞ ﳾ ٮ_ﻼث ﻣرات )ڡ‪+‬ى اﻷﺳ;ٮوع‬
‫ﻣ‪2‬ٮ&ٮ)ڡورﻣ‪2‬ٮﻦ ‪500‬‬
‫اﺳ;ٮريﻦ‬
‫و اﻟرا;ٮﻊ )ﻋﻠﻂ‬

‫اٮ ‪2‬ﺶ )ٮﺼ‪2‬ٮﺤﻪ اﻟﺼ‪2‬ٮﺪل‪+‬ى ﻟﻬﺎ؟ ‪ Mycophenolate‬وﺣﺪه راﺣﺖ ﻋ)ٮﺪ اﻟﺼ‪2‬ٮﺪل‪+‬ى وٮ&;ٮ‪r‬ﻌﻰ ٮ&ﺤﻤﻞ وﻫ‪2‬ٮﺎ ٮ&ﺴ&ٮﻌﻤﻞ‬
‫ﻻزم ٮ&ﻮ&ڡﻒ ‪ ٦‬ﺷﻬور ;ٮﻌﺪٮ ‪2‬ﻦ ٮ&ﺤﻤﻞ‬

‫؟‪ Fingolimod‬اٮ ‪2‬ﺶ اﻻ)ﺣ&ٮ;ٮﺎر ال‪+‬ى ٮ&ﺴويﻪ ﻟﺸ)ﺤﺺ ;ٮيﺄ)ﺣﺬ‬


‫‪ECG‬‬
‫‪Nephro‬‬
‫‪Her2‬‬
‫‪Cyp2D6‬‬

‫اﻟﲇ ﻋ)ٮﺪﻫﺎ ﺳﺮﻃﺎن )ڡ‪+‬ى اﻟ_ٮﺪي اٮ ‪2‬ﺶ ٮ&ﻌﻄ‪2‬ٮهﺎ ﻋ)ٮﺪﻫﺎ اﻻﺳ&ٮرو;ﺣ‪2‬ٮﻦ ﻋﺎل‪+‬ى‪ ،‬اﻟﺪواء ال‪+‬ى ٮ&ﺄ)ﺣﺬه )ڡ‪+‬ى ﻫﺬي اﻟﺤﺎﻟﻪ؟‬
‫ى ﻫﺬي اﻟﺤﺎﻟﻪ؟‬+‫ى ٮ&ﺄ)ﺣﺬه )ڡ‬+‫ اﻟﺪواء ال‬،‫ى‬+‫ٮﻦ ﻋﺎل‬2‫ٮهﺎ ﻋ)ٮﺪﻫﺎ اﻻﺳ&ٮرو;ﺣ‬2‫ﺶ ٮ&ﻌﻄ‬2 ‫ى اﻟ_ٮﺪي اٮ‬+‫اﻟﲇ ﻋ)ٮﺪﻫﺎ ﺳﺮﻃﺎن )ڡ‬
tamoxifen

Alzheimer ‫ﻋﻼ;ﺣﻪ؟‬
Donepzil
‫ واﻟ)ٮ;ٮ&ٮﻪ اﻟﲇ ٮ&ﻌﺎﻟ;ﺤﻪ ؟‬gingko biloba

-‫ ﺳؤال‬INR ‫ى‬+‫ﻜون )ڡ‬2 ‫ ﻛﻢ ٮ‬DVT ‫؟‬


٢-١
٣-٢
٣.٥ - ٢.٥
٦ -٥

How to write a prescription in board accreditation?


A.HCTZ 100 mg orally twice daily
B.HCTZ 1g orally every 12 hour's
C. hydrochlorothiazide 100 mg orally every 12 hours.
D.hydrochlorothiazide 100 mg orally twice daily

Which of the following vaccines can be


administered either intramuscularly or
subcutaneously?
a. Influenza vaccine
b. PPSV23
c. PCV13
d. MCV

Vaccine take oral ?


Rota
Vaccine CI in pragnancy?
MMR

Name of VitK?
Phytonadione

Case and the pharmacist realised the consultant mistake and consultant shouted at pharmacist
“ you pharmacists
pass your lines .. mind your business” and the pharmacist did not respond What was the
pharmacist’s response?
- Passive
- aggressive

Preparations non sterilizing?


Oral ‫اٮ&ﻮ&ڡﻊ‬
Iv
Sc
Eye

MOA of entacapone?
COMT

What supplements use to decrease Triglycerides ?


Co Q10
biotin
b6
Fish oil = omega-3

Structure of Paracetamol (Acetaminophen) ? ‫ﻌﻪ‬r‫ٮ‬2‫ﻃﻠﺐ اﻟﺼ‬:


C8H9NO2

Which of the following is considered plant origin!?


Digoxin
Propranolol
Enalapril
Theophylline

Which of the following is made of sugar part and non sugar part?
Glycosides
Glucose
Fructose

Converting prontosil to active form(sulfonamide) ? ‫ﺤ)ڡﺰه‬2 ‫ﺶ اﻟﲇ ٮ‬2 ‫واٮ‬


Oxidase
Reductase
particles dissolved in 10 ? ‫ﻌ&ٮ;ٮﺮ‬2 ‫ﻌﲏ ;ٮي&ڡول ٮ‬2 ‫ٮ‬
5 solute
100 particles dissolved in 5 ‫اٮ&ﻮ&ڡﻊ‬
1000 particles
10000 particles

Treatment of atherosclerosis?
Olive oil

Vitc (ascorbic acid) reduce?


Duration of cold
Severity

MOA of Bisacodyl?
Stimulation of enteric nerves to cause colonic contraction

Cause infectious mononucleosis?


Epstein-Barr virus
Haemophilus influenza
Salmonella

Causes ototoxicity:
ethacrynic acid, furosemide, bumetanide
‫ٮﺐ وﺣﺪه ﻣ)ٮﻬﻢ‬2‫كﻠﻬﻢ ﺻﺢ ;ٮي;ﺤ‬

A tablet is scored twice(has 4 quadrants) what does that mean?


It is an Immediate release tab
It can be cut in 4
It can be cut in 2

-INR in DVT:
2-3
-INR in mitral valve :
2.5-3.5

Definition of prodrome?
-An early sign or symptom (or set of signs and symptoms) that often indicates the onset of a
disease before more diagnostically specific signs and symptoms develop.
-Period from infection to appear symptoms
‫ٮﻪ اٮ&ﻮ&ڡﻊ‬2‫ﺎ ﻛﺬا او ﻛﺬا كﻠﻬﻢ ﺻﺢ ;ٮﺲ ;ﺣﺎب اﻟ_ٮﺎ)ٮ‬2 ‫ٮ‬

-[Patient receive 100mg of 4 drugs A,B,C,D and initial conc of drugs were 20,5,0.5,4 and
mesured volume of distribution is 5L
A
B
X
D
‫ وﰷٮ&ﺐ‬zero conc ‫ﻟكﻞ دواء ٮ&&ڡري;ٮًﺎ‬
20
0.5
5
4
‫ وي&ڡول ا)ٮﻪ ﻟﻤﺎ &ڡﺎس‬vd ‫ ٮ&;ٮ&ڡﻰ أي‬،‫ ﻟ&ٮﺮ‬5 ‫ ﻟ&ڡﺎﻫﺎ‬drug‫ ؟ اﻻ;ﺣﺎ;ٮﺔ‬A
*‫ٮﻖ اﻟ&ڡﺎ)ٮون‬2‫ٮ&ﻄ;ٮ‬: D/C0
100/20 = 5 ‫]اﻟﲇ ;ٮﺎﻟﻤﺴﺄﻟﺔ‬

-‫ ﻣريﺾ ;ٮيﺎ)ﺣﺪ‬cortisone ‫ﺶ ﻣﻌﺎه؟‬2 ‫ﺎ)ﺣﺬ اٮ‬2 ‫ٮ&)ٮﺼﺤﻪ ٮ‬


‫ٮﺎرات‬2‫وﰷن ﻣﻦ ﺿﻤﻦ اﻻ)ﺣ&ٮ‬:
vit D + Ca

‫ال‬Hydroxychloroquine ‫ﺶ ؟‬2 ‫)ﺤرج ﻋﻦ ﻃريﻖ اٮ‬2 ‫ٮ‬


Renal (kidney)
Hepatic
Pulmonary
‫اﻟرا;ٮﻊ ﻣﺪري‬

-‫ٮ;ﺤﺔ‬2‫ واﺣﺪة ﻋ)ٮﺪﻫﺎ أﻟﻢ )ٮ&ٮ‬dusmenorrhea ‫)ٮﻌﺎﻟﺞ‬


‫ى‬+‫ وﻟﻼﺳﻒ ﻣﺎ)ڡ‬,‫ﺐ اﺳ&ٮﺮﻛﺸرات‬2 ‫ﺶ؟ و;ﺣﺎٮ‬2 ‫ اﻻﻟﻢ ;ٮﺎٮ‬Mefenamic, ‫ٮﺎرات ﰷ)ٮﺖ‬2‫ اﻻ)ﺣ&ٮ‬:
Aspirin
Naproxen
Diclofenac
*‫ٮﻪ‬2‫ ﻟﻮ )ڡ‬Mefenamic ‫ﻫﻮ اﻻﺻﺢ )ٮ)ﺤ&ٮﺎره‬ *

‫ ﺳؤال‬dissolution equations‫؟‬
‫ ﺳؤال‬dissolution equations‫؟‬
Noyes whitney

‫ ﺳؤال‬RCT ‫ٮﺔ؟‬2‫ى اﻟﻤ‬+‫ٮﺤﺔ ;ٮ)ٮﺴ;ٮﺔ ﰷم )ڡ‬2‫ى ﺻﺤ‬+‫;ٮ&ٮ;ٮ&ڡ‬


60
75
85
90

‫ﺎ)ﺣﺬه وﻫﻮ )ﺣﺎرج ﻣﻦ اﻟﻤﺴ&ٮﺸ)ڡﻰ؟‬2 ‫ٮﻦ اﻟﲇ ٮ‬2‫ﺶ اﻟ)ڡﺎﻛﺴ‬2 ‫ى اﻟﻤﺴ&ٮﺸ)ڡﻰ اٮ‬+‫ﺳؤال ﻣريﺾ )ڡ‬
Hep B,hpv,influenza
Influenza,hep A,varicella
Influenca,pneumococcal,mcv
Influenza,hov,pneococal

‫ ا)ﺣﺮ ﻣﺎدة ٮ&تﻜون‬metabolism of Uracil ‫؟‬


Ammonia
Uracil
Xanthine
Alanine

‫ﺸرح اﻟ)ڡرق ;ٮيﻦ‬2 ‫ٮﺮة ٮ‬2‫ٮﺲ ﻛ;ٮ‬2‫ ;ﺣﺎب ﻛ‬tablet and oral form ‫&ڡول‬2 ‫ويﺴﺎل ٮ‬:
What the rate limiting step of the oral liquid from tablet dosage form
‫ﻜون‬2 ‫ اﻟ;ﺤواب ٮ‬:
Dossolution
Disintegration
*‫ﻟﻠ&ٮﻮﺿيﺢ‬:*
Disintegrating = Solid / Tablet
Dissolution = Liquid

‫ٮﻪ ﻃﺎﻟﺐ ;ٮي&ٮﻌﻠﻢ ﻋﲇ ﻣريﺾ ﻋ)ٮﺪه‬2‫ٮﺲ ﻃويﻠﺔ ;ٮ&ٮ&ڡول ان )ڡ‬2‫ ﻛ‬pseudomembraneous colitis ‫و)ٮﺼﺢ اﻟﻤريﺾ ا)ٮﻪ‬
‫ﺴ&ٮ)ﺤﺪم‬2 ‫ ٮ‬alcohol and gloves
‫ى )ٮ)ڡﺴﻪ؟‬+‫ﺤم‬2 ‫ﺶ ﻋﺸﺎن ٮ‬2 ‫اﻟﺴؤال ;ٮي&ڡول ٮ&)ٮﺼﺢ اﻟﻄﺎﻟﺐ ;ٮﺎٮ‬
Use alcohol
Give metronidazole
Wash hand with water and soap

‫ ﺳؤال ﻣريﺾ ا)ﺣﺬ‬paracetamol ‫ وا)ﺣﺪ ﻣﻌﺎه‬,‫ٮﻮم‬2‫ ﻣرات فﻰ اﻟ‬٣ ‫;ﺣﻢ‬١ ibuprofen ‫ﺸ&ٮكﻰ ﻣﻦ‬2 ‫ٮﻮم و;ﺣﺎي ٮ‬2‫ى اﻟ‬+‫ ﻣرات )ڡ‬٣
‫ ﺳؤال ﻣريﺾ ا)ﺣﺬ‬paracetamol ‫ وا)ﺣﺪ ﻣﻌﺎه‬,‫ٮﻮم‬2‫ ﻣرات فﻰ اﻟ‬٣ ‫;ﺣﻢ‬١ ibuprofen ‫ﺸ&ٮكﻰ ﻣﻦ‬2 ‫ٮﻮم و;ﺣﺎي ٮ‬2‫ى اﻟ‬+‫ ﻣرات )ڡ‬٣
dark stool and GI bleeding ‫وش اﻟﺴ;ٮﺐ‬
Dru drug interaction
Gi bleeding from brufen ‫اٮ&ﻮ&ڡﻊ ذي‬
Gi bleeding from paracetamol

‫ٮﺴ)ٮﺎ ﻟﻪ‬2‫ ﺳؤال ;ٮي&ڡول ﻣريﺾ &ڡ‬GFR ٥٤ ‫ وﻃﻠﻊ‬stage of kidney failure


Stage 1 :
> 90
2:
89-60
3:
59-30

Which of the following drugs used for ventricular and supraventricular arrhythmia!?
A. Sotalol
B. Doflitide ‫ٮﻤﺎن‬2‫ﻣﺤﻤﺪ ﺳﻠ‬.‫د‬
C. Procainamide
D. ‫ اﻋ&ٮ&ڡﺪ ﰷن‬Phenytoin
‫ٮﻪ‬2‫ وﻟﻮ )ڡ‬Flec+prpafenone ‫اﻻﺻﺢ‬

1)‫ اﻟ)ٮ&ٮ;ٮﻪ اﻟﲇ ٮ&ﺴ&ٮ)ﺤﺪم‬Depression ‫؟‬


St. John's

2)‫ٮ)ڡﺔ اﻟ&ٮ)ڡﺎﻋﻞ‬2‫ و)ﻃ‬NAD-NADP ‫ٮﻪ؟‬2‫دا)ﺣﻞ اﻟ)ﺤﻠ‬


‫ٮﻦ‬2‫ٮ&ٮرو;ﺣ‬2‫)ٮ&ڡﻞ اﻟ)ٮ‬
Electron carrier

3) ?Drug interaction with Verapamil


Digoxin

‫ٮﻜﺖ ؟‬2‫ﺪ ا)ڡ‬2 ‫ﺴ&ٮ)ﺤﺪم ﻟﺴﺎٮ‬2 ‫اﺳﻢ اﻟﺴﻜور اﻟﲇ ٮ‬


Naranjo scale

4)‫ﺎ)ﺣﺬ‬2 ‫ ﻣريﺾ ٮ‬cortisone ‫ﺎ)ﺣﺬه؟‬2 ‫ٮﻦ اﻟﲇ ٮ&)ٮﺼﺤﻪ ٮ‬2‫ٮ&ٮﺎﻣ‬2‫ﺶ اﻟ)ڡ‬2 ‫ﻟ)ڡ&ٮﺮة ﻃويﻠﻪ اٮ‬
Iron
Ca and D
Vit c

5) ?Toxicity of lithium
Na
Mg

6)?Dose of calcium for Elderly


1200

7)PH of Stomach?
1.5-3.5

8) ?Vaccine for pregnant in Third Tri


Influenza

9) BB decrease mortality?
Carvidolol

10) ٪١٠٠ ‫ اي )ٮوع‬bioavailability


Iv
-‫ وآ)ﺣﺮ )ٮوع‬: Oral

11) phenytoin enzyme‫؟‬


Cyp2c19

12)‫ ﻣريﺾ ﻋ)ٮﺪه‬renal failure ‫ٮﺎرات‬2‫ اﻟ)ﺤ‬,‫ٮﺮ‬2‫ وكﻼم ﻛ_ٮ‬:


-‫&ڡول‬2 ‫ ٮ‬h1/2 ‫ و‬vd ‫زيﺪون‬2 ‫ٮ‬
-‫)ٮ&ڡﺼون‬2 ‫او ٮ‬
-‫زيﺪ‬2 ‫ او ٮ‬h1/2 ‫ وي)ٮ&ڡﺺ‬vd

13)‫)ٮﺎ&ڡﺶ‬2 ‫ﺴوي ا;ﺣ&ٮﻤﺎع ٮ‬2 ‫ﱯ ٮ‬2 ‫ٮﻪ ٮ‬2‫ٮﺪﻟ‬2‫ﺮ اﻟﺼ‬2 ‫ٮ)ٮﺎريﻮ ا)ٮﻬﻢ ;ٮيزودون اﻻﺳﺮه ;ٮﺎﻟﻤﺴ&ٮﺸ)ڡﻰ وﻣﺪٮ‬2‫ ﺳ‬:
-‫ٮﻪ‬2‫ٮﺪﻟ‬2‫زود ﺳ&ٮﺎف اﻟﺼ‬2 ‫ٮ‬
‫زودون اﻟﺴ&ٮﻮك‬2 ‫ٮ‬
‫ى‬+‫ٮﺎريﻦ ﻣﺎﻟﻬﻢ داع‬2‫و)ﺣ‬

14)? Intrasynovial means


Spinal
Muscle
Joint

1 ?(15 dexteose kcal


3
3.4
4

16)?Used for uti infection


Garlic
Cranberry
Echinacea
Ginko

17)?Use in the preventtion of RSV in neonate


Plvizumab
Osltumab

18)Aspartame added to preparation as :


Sweeting agent

19) ‫ وﻣﻌﻪ‬٧٥ ‫ٮﺲ ﻣريﺾ ﻋﻤﺮه‬2‫ ﻛ‬pulmonary embolism ‫ﻛﻢ ;ﺣﺮﻋﺔ اﻟوار)ڡﺎريﻦ؟‬
2.5
5
7.5
10

20)10 ‫ ﺳؤال‬fats ‫ ﻛﻢ ٮ&ﺴﺎوي‬calories ‫؟‬


90 calories

21) isonazid ‫ﺎ)ﺣﺬه وش ٮ&)ٮﺼﺤﻪ؟‬2 ‫ﻣريﺾ ٮ‬


Sunscreen and hydration

‫ ﺳؤال ﻋﻦ‬Valine deficiency :


‫)ﺤﺺ ال‬2 ‫ ﳾ ٮ‬Muscle

Aminothyophyline % of theophylline?
70
85
80

1)-Human suppressor gene?


P53

2)-PC?
After food

3)-old patient have renal failure he wants to take vaccine?


HepA ‫ ﺷﻬﺮ‬٢٤-١٨ ‫ٮﻦ وﻋﻤﺮه‬2‫اﻟﻤ)ڡروض ا)ﺣﺬ اﻟ;ﺤﺮﻋ&ٮ‬
, Varcilla
, yellow fever
Zoster ‫ﻣﻤﻜﻦ‬
influenza
Dtap ‫اﻟﻤ)ڡروض ا)ﺣﺬﻫﺎ وﻫﻮ ﻃ)ڡﻞ‬
Pneumococcal

4)-vaccine can be given to a child age between 6-12 month?


BCG
DTap

5)-Monitor of Argatroban?
ApTT

6)-case and we want to give him warfarin what will we measure when we give it to him?
PT/INR , renal functions

7)-Ciprofloxacin structure and what does piperazole chain do?


Increase against G-ve

8)-child have otitis media for how many days we give him Antibiotic?
3
5
7 (childeren ‫)اذا &ڡﺎل‬
10 if (less than 2 years)
9)- 1 years old child have otitis media what to give?
Amoxicillin
Azithromycin

10)-child have whooping cough (pertussis)ttt?


Vaccine? Dtap
Azithromycin ‫اﻻﺻﺢ‬

11)-G6PD deficiency?
Tmp/Smx

12)-eclampsia ttt?
Magnesium sulfate

13)-patient have liver and renal failure what type of vitamineD can he take?
Ergocalciferol
Cholecalciferol
calcitriol
Calcitonin

14)-chlamydia first choice ttt?


Azithromycin

15)-UTI infection ttt?


Tmp/Smx

16)-Amiodarone SE?
Hyperthyroidism or Hypo both
Pulmonary toxicity
Hepatotoxicity
Skin photosensitivity
‫ٮﺎرات‬2‫ى اﻟ)ﺤ‬+& ‫ﻣﺪري ;ٮﺎڡ‬

17)- Crohn disease ttt?


- Acute ? Oral or IV steroid ( Prednisone or Budesonide 9mg OD upto 8 weeks.
- Maintenance of remission?
Infliximab

18)-Hashimoto’s disease ttt?


Levothyoxine

19)-cushing’s syndrome?
Adrenal insufficiency
Adrenal gland overactive ‫&ڡﻮﻟﻮ ذي اﺻﺢ‬2 ‫ٮ‬
Hyperthyroidism
Hypothyroidism

20/ ‫ﺶ ﺳ;ٮ;ٮﻠﻪ‬2 ‫ٮهﺎ ﻋﺪة ادويﺔ واٮ‬2‫ٮﺲ ﻃويﻠﻪ و)ڡ‬2‫ ﻛ‬constipation and dark stool‫؟‬
Ferrous sulfate

21)-mycobacteria?
Acid fast bacillus

22)-clostridium?
G+ve bacilli anaerobic
G+ve cocci anaerobic
G+ve bacilli aerobics
G+ve cocci aerobic

23)-pseudomonas DOC?
Ceftriaxone / ceftazidim

24)-vitamin deficiency cause cracks and sores in mouth and lips?


Riboflavin B2

25)-what’s the right thing about vitamin D?


D2 and D3 both considered biological inactive

26)-direct and indirect and intangible cost

27)-cost benefit analysis?


Monetary outcome Dollars
28)-cost analysis Humanistic out come?
QALT

29)-cost effectiveness analysis calculated by?


ICER ( Incremental cost-effective ration )
‫ﺎ‬2 ‫ٮﻪ ٮ‬2‫ ﻣﻮ ﻣ&ٮﺬﻛﺮه ﰷن )ڡ‬ICER ‫ﺎ‬2 ‫ ٮ‬ACER
‫ٮﺎرات‬2‫ٮﺐ وﺣﺪه ﻣ)ٮﻬﻢ ;ٮﺎﻟ)ﺤ‬2‫اﻟﻤﻬﻢ ;ٮي;ﺤ‬

30)-determine the official role of each HCP?


MOH

1- supplement give with iron ?


Vit c

2- dose of calcium for pregnant & elderly


1200

3- supplement give with long term corticosteroid??


Ca + D

4-pregnant with nausea ?


Ginger or vitB6

5- which antidepressant cause constipation + urinary retention + sleeping?


-St john
-TCA
-MAO
-SSRi

6- structure of chlorpromazine and 4 options ask about which one is the metabolite?

7- give 4 different structure ask about which used for anti arrhythmia ?
Quinidine

8- women come to pharmacy want pregnancy test and ask about how to use?
(Deep in urine)
9- antidote iron ?
Deferoxamine

10- antidote paracetamol ?


N-acetylcysteine

11- type of aspirin lispro

12- heartburn and osteoporosis?


Ca carbonate

13- essential amino acids or fatty acid

14- drug for MAC ?


Diazepam

15- similar affect to propranolol?


All choices BB
‫ٮﻜ&ٮﻒ اٮ&ﻮ&ڡﻊ‬2‫ٮﺎر اﻟ)ٮون ﺳﻠ‬2‫;ٮيﻜون اﻟ)ﺤ‬

16- beta blocker least lipid?


Atenolol or sotalol ‫ﻣﻮ ﻣ&ٮﺎﻛﺪه‬
Nadolol

17- pregnant with HIV ?


Zidovudine

18- target INR for matrial value ?


2.5–3.5

19- SE of lamotrigine ?
Alopecia sever rash

20- fingo … ?
ECG

21- test name of vit b12 ?


Schilling test

22- when diagnose DM if ?


6.4 - 6.4 - 6.8
6.5

23- refeeding syndrome ?


Hypophosphatemia

24- treatment of crons disease?


Infuximab

25- functions of kidney?


Serum creatinine
Cr cl
BUN

26- most bioavailability


Rectal
Inhalation
Sublingual
Buccal

27- hypotonic solution?


D5 with water
D5 with Na 0.9
Na 4.5 ‫اٮ&ﻮ&ڡﻊ‬
0.45%
0.225%
‫;ٮﻮ‬2 ‫ذوﻻ ﺻﺢ كﻠﻬﻢ ﻫﺎٮ‬

28- water in Injection ?


Sterile water for injection

29-Which of the following medications is an osmotic diuretic?


A. mannitol
B. furosemide
C. spironolactone
D. Hydrochlorothiazide

30- Patient female with food poisoning and went to hospital with troponin high NSTMI what is the
management
A. aspirin 81 mg + ticagrelor
B. Aspirin 325 mg + ticagrelor
C. Aspirin 81 mg +Prasugrel
D. Aspirin 325 mg + prasugrel

31- What is the age range of a neonate?


A. Birth to one week
B. Birth to one month
C. One month to six months
D. One month to twelve months

32- Vit B12 name


cobalamin

33- An elderly man with hypothyroidism was started on thyroxin. The drug has a half - life of
72hours. How long will it take to produce full effect?
A. two days
B. one week
C. two weeks
D. two months

34- Which of the following describes the best patient instruction regarding the proper way to
dispose of needles?
A. Discard the needles in garbage
B. Return needles to their original package
C. Discard the needles in a rigid and resistant container
D. If the needle is for personal use, you can use it for two times to reduce environment
contamination

35- amount of water in neonate ?


A. 60%
B. 75%
C. 80%
D. 90%

36- Drug acts on non-receptor mechanism ?


A. Deferoxamine
B. Desloratadine
C. Atenolol
D. None of the above

37- HIV, TB , AIDS


A- Chronic disease
B- Communicable disease
C- Non communicable

38- iso 5?
100

41- Public health is the science of protecting the health of a community Which population from
the following list is their target?
A) People at risk of being sick
B) Injured and sick people
C) Mothers and children
D) Entire community

42- What resource should be used when searching for the most current clinical trials on the use
of novel anticoagulants for pulmonary embolism?
A. Pharmacotherapy, the pathophysiologic approach
B. PubMed
C. UpToDate
D. Micromedex

43-Which of the following genetic testing might derive a potential benefit from Trastuzumab
therapy in a breast cancer patient?
A. TPMT positive activity
B. VKORC1 genotype positive
C. HER2/gene genotype positive
D. HLA-B *5801 genotype positive
‫‪ cm‬ﰷن ;ﺣﺎٮ ‪2‬ﺐ ﻣرأة ﻃﻮﻟﻬﺎ ‪44)165‬‬
‫؟ ‪ IBW‬وي&ڡول ﻛﻢ ال‬
‫‪50‬‬
‫‪55‬‬ ‫ٮ&&ڡري;ٮًﺎ‬
‫‪61‬‬
‫‪65‬‬
‫ﻟﻠ&ٮﻮﺿيﺢ*‪ :‬اﻟﻤ)ڡروض ‪ 56.77‬ﻫﻮ اﻻٮ ‪2‬ﺪٮ ‪2‬ﺎل*‬
‫ﻃﻮﻟﻬﺎ ;ٮﺎﻟ&ٮﺤويﻞ ﻟﻼ)ٮﺶ = ‪64.9‬‬
‫اﻟ&ڡﺎ)ٮون ٮ ‪&2‬ڡول ﻟﻠ)ڡ‪2‬ٮﻤ‪2‬ٮﻞ‬
‫) ‪45.5 + ( 2.3 * inches more than 60‬‬
‫‪45.5 + ( 2.3 * 4.9 ) = 56.77‬‬

‫)ڡ‪2‬ٮﻪ ﺳؤال )ﻣﺎﻋﺮ)ڡ)ٮﺎ ;ﺣوا;ٮﻪ(‪ :‬واﺣﺪ ٮ ‪;2‬ﺤ‪2‬ٮﻪ)‪45‬‬


‫‪Asthma exacerbation‬‬
‫ويﺴﺄل ﻟ‪2‬ٮﻪ وﻣﻌﻄ‪2‬ٮﲏ &ڡراءات‬
‫اﻟ&ڡﻠﺐ ﻋﺎل‪+‬ى‬
‫واﻟ&ٮ)ٮ)ڡﺲ ‪٤٠‬‬
‫‪ FEV1‬و ‪ FVC‬وال‬
‫‪ %‬واﺣﺪ ﻣ)ٮﻬﻢ ‪ %0.5‬واﻟ_ٮﺎ)ٮ‪+‬ى ‪35‬‬
‫‪:‬واﻟ)ﺤ‪2‬ٮﺎرات ﰷ)ٮﺖ‬
‫ﻣ)ٮ)ﺤ)ڡﺾ ‪ FVc‬ان اﻟ&ڡﻠﺐ واﻟ&ٮ)ٮ)ڡﺲ ﻣﺮٮ&)ڡﻌ‪2‬ٮﻦ و‬
‫وﻻ اﻟﻌﻜﺲ زي ذا‬
‫‪ FEV1‬و ‪ FvC‬كﻠﻬﺎ ﰷ)ٮﺖ اﻟ&ڡﻠﺐ واﻟ&ٮ)ٮ)ڡﺲ ﻣﺮٮ&)ڡﻌ‪2‬ٮﻦ وي‪r‬ﻌ‪2‬ٮﺮ ب‬

‫وﺳؤال واﺣﺪ ﻋ)ٮﻪ كﻠﻪ ;ٮﺸكﻞ داٮ ‪2‬ﻢ وﺿ‪2‬ٮﻖ ٮ&)ٮ)ڡﺲ وﻫﺎﻻﻋراض و;ٮ)ٮﻬﺎٮ ‪2‬ﺔ اﻟﻜ‪2‬ٮﺲ ﰷٮ&ﺐ ا)ٮﻪ ٮ ‪2‬ﺪ)ﺣﻦ ﻣﻦ )ڡ&ٮﺮه ﻃويﻠﻪ)‪46‬‬
‫وش اﻟ&ٮﺸ)ﺤ‪2‬ٮﺺ؟‬
‫‪Pneumonia‬‬
‫‪COPD‬‬
‫واﺷ‪2‬ٮﺎء ٮ_ﺎ)ٮ‪+‬ى كﻠﻬﺎ ;ٮﺎﻟ&ٮ)ٮ)ڡﺲ‬
‫ﻋﺸﺎ)ٮﻪ ﻣﺪ)ﺣﻦ ‪ COPD‬ﺣﻄ‪2‬ٮﺖ‬

You might also like